STUDY-MODE - Practice Exam 6 Flashcards

1
Q

Some research suggests that racial identity has a greater impact than race itself on the therapeutic process. For instance, studies on African American therapy clients suggest they are most likely to prefer an African American therapist when they are in which stage of identity development?
Select one:

A. pre-encounter
B. immersion
C. internalization
D. autonomy

A

The stages listed in responses a, b, and c are included in Cross’s Black Racial Identity Development Model.

a. Incorrect During the pre-encounter stage, race has low salience.
b. CORRECT The first half of the immersion-emersion phase is characterized by immersion in African American (ethnic) culture. Not surprisingly, people in this phase usually prefer a therapist of their own ethnicity.
c. Incorrect Internalization-commitment is the final stage of identity development. During this phase, other factors in addition to ethnicity are considered when choosing a therapist.
d. Incorrect Autonomy is not a stage in Cross’s model and, in other models, is most similar to Cross’s internalization phase.

The correct answer is: immersion

How well did you know this?
1
Not at all
2
3
4
5
Perfectly
2
Q

Freud argued that the “work of the mental apparatus is directed toward keeping the quantity of excitation low.” If the mental apparatus is unsuccessful in doing so, the result is likely to be which of the following?
Select one:

A. anxiety
B. ego decompensation
C. psychosis
D. object-cathexis

A

Excessive excitation by the id’s impulses, according to Freud, leads to anxiety.

a. CORRECT The function of the ego’s defense mechanisms, for example, is to keep the conflicts that produce anxiety out of consciousness.
b. Incorrect This is a “made up” term.
c. Incorrect While unresolved anxiety can, from a Freudian perspective, lead to psychosis, this is not necessarily the case and, therefore, is not the best answer.
d. Incorrect Although object cathexis is a psychoanalytic term, it refers to the id’s investment of energy into an object that will satisfy an instinctual need and is not the correct answer to this question.

The correct answer is: anxiety

How well did you know this?
1
Not at all
2
3
4
5
Perfectly
3
Q

Sue and his colleagues (2007) distinguish between three forms of racial microaggression. As described by these investigators, __________ is occurring when a White person says to an African-American individual, “When I look at you, I don’t see color” or “There’s only one race, the human race.”
Select one:

A. microassault
B. microinsult
C. microinvalidation
D. microinequity

A

Microassault, microinsult, and microinvalidation are the three forms of microaggression described by D. W. Sue et al. (Racial microaggressions in everyday life: Implications for clinical practice, American Psychologist, 62, 271-286, 2007).

a. Incorrect Microassaults are “explicit racial derogations characterized primarily by a violent verbal or nonverbal attack meant to hurt the intended victim through name-calling, avoidant behavior or purposeful discriminatory actions” (p. 278).
b. Incorrect Microinsults are “remarks or comments that convey rudeness, insensitivity and demean a person’s racial heritage or identity” (p. 278).
c. CORRECT Microinvalidation refers to “verbal comments or behaviors that exclude, negate, or nullify the psychological thoughts, feelings, or experiential reality of a person of color” (p. 278). “Color blindness” is a type of microinvalidation.
d. Incorrect Microinequity is not one of the three forms of microaggression described by Sue et al.

The correct answer is: microinvalidation

How well did you know this?
1
Not at all
2
3
4
5
Perfectly
4
Q

When conducting Seligman’s Positive Psychotherapy (PPT), a client is asked to keep a daily journal in which he/she lists:
Select one:

A. three good things that happened each day.
B. three good and three bad things that happened each day.
C. his/her thoughts about the good and bad things that happened each day.
D. at least one positive activity that he/she will engage in the following day.

A

Knowing that the name of the daily journal is the blessings (gratitude) journal may have helped you identify the correct answer to this question. See, e.g., M. E. P. Seligman, T. Rashid, and A. C. Parks, Positive psychotherapy, American Psychologist, 61, 774-788, 2006.

a. CORRECT Although Positive Psychotherapy does not entirely ignore negative events, as its name implies, it focuses primarily on positive events.
b. Incorrect See explanation above.
c. Incorrect See explanation above.
d. Incorrect See explanation above.

The correct answer is: three good things that happened each day.

How well did you know this?
1
Not at all
2
3
4
5
Perfectly
5
Q

As defined by Alfred Adler, __________ refers to a person’s goals and the ways in which he or she attempts to achieve those goals.
Select one:

A. homeostasis
B. success identity
C. self-actualization
D. style of life

A

When preparing for exam questions from the area of Clinical Psychology, a good strategy is to associate key concepts and terms with each of the major theories and theorists.

a. Incorrect Homeostasis refers to the tendency for a system (e.g., a family system) to maintain its current level of functioning.
b. Incorrect In reality therapy, a person has adopted a success identity when he or she fulfills his or her needs in responsible way.
c. Incorrect The term “self-actualization” is used by several theorists. As defined by Rogers, it refers to an internal biological force that serves as a major source of motivation and guides the individual toward positive, healthy growth.
d. CORRECT Adler believed that each person has a unique set of goals and ways for achieving goals, which he referred to as the person’s style of life. According to Adler, a person’s style of life is fairly well established by 4 or 5 years of age.

The correct answer is: style of life

How well did you know this?
1
Not at all
2
3
4
5
Perfectly
6
Q

With regard to power in the client-therapist relationship, a feminist therapist would most likely:
Select one:

A. recognize that her power stems primarily from her ability to serve as a role-model.
B. acknowledge the power differential inherent in the relationship but view it as a temporary condition.
C. initially foster her position of power so as to maximize her influence.
D. regard power a “non-issue” because feminist therapists consider the therapeutic relationship to be egalitarian.

A

A key feature of feminist therapy is its promotion of an egalitarian approach.

a. Incorrect Although the therapist is viewed as a role model by many feminist therapists (and their clients), feminist therapists attempt to minimize the power differential inherent in the therapeutic relationship.
b. CORRECT This issue is addressed by M. A. Douglas in her chapter entitled “The Role of Power in Feminist Therapy: A Reformulation” [in L. B. Rosewater and L. E. A. Walker (Eds.), Handbook of feminist therapy: Woman’s issues in psychotherapy, New York: Springer, 1985]. According to Douglas, the power differential inherent in the therapeutic relationship should, first of all, be acknowledged. It should also be viewed as temporary since the goal of therapy will be to terminate the relationship and/or to readjust the relationship so that it is more egalitarian. Douglas stresses the importance of minimizing the power differential inherent in the therapeutic relationship.
c. Incorrect See explanation for response b.
d. Incorrect Feminist therapists acknowledge the power differential inherent in the therapeutic relationship and view it as an issue that must be dealt with.

The correct answer is: acknowledge the power differential inherent in the relationship but view it as a temporary condition.

How well did you know this?
1
Not at all
2
3
4
5
Perfectly
7
Q

A therapist is most likely to say that which of the following is the most stressful type of client behavior?
Select one:

A. threats of a malpractice suit
B. demands for time and attention
C. suicidal statements
D. a lack of gratitude

A

Therapists experience stress and burnout for several reasons. Not surprisingly, the most difficult situation–i.e., the one requiring an immediate response–is the one that is associated with the greatest stress.

a. Incorrect See explanation for response c.
b. Incorrect See explanation for response c.
c. CORRECT Surveys of therapists have revealed that suicidal ideation and threats are considered the most stress-producing client behaviors.
d. Incorrect See explanation for response c.

The correct answer is: suicidal statements

How well did you know this?
1
Not at all
2
3
4
5
Perfectly
8
Q

Which of the following approaches in family therapy encourages the therapist to develop a “therapeutic triangle” with family members?
Select one:

A. extended family systems therapy
B. communication/interaction family therapy
C. adaptive family therapy
D. strategic family therapy

A

In extended family systems theory, one therapeutic technique involves creating a therapeutic triangle in which the therapist actively interacts with two family members during the treatment process. The goal is to shed light on and reduce the fusion that exists between family members.

a. CORRECT Bowen’s extended family systems therapy encourages the therapist to become a member in a therapeutic triangle with two family members (usually the spouses/partners).
b. Incorrect In communication/interaction family therapy, the therapist serves as a facilitator and teacher.
c. Incorrect Adaptive family therapy is a “made up” term.
d. Incorrect Although strategic family therapy involves an active therapist, the role of the therapist is to assess the problems, not to become a member of the family’s triangles.

The correct answer is: extended family systems therapy

How well did you know this?
1
Not at all
2
3
4
5
Perfectly
9
Q

According to Atkinson, Morten, and Sue’s (1993) Racial/Cultural Identity Development Model, the resistance and immersion stage is characterized by:
Select one:

A. a preference for the dominant group and resistance to being considered a member of a minority group.
B. conflicting attitudes toward both the dominant group and one’s own group.
C. rejection of the dominant group and preference for one’s own group.
D. active participation in efforts to promote social change that will benefit one’s group.

A

The Racial/Cultural Identity Development Model distinguishes between five stages of identity development: conformity, dissonance, resistance and immersion, introspection, and integrative awareness.

a. Incorrect Preference for the dominant group is characteristic of the conformity stage.
b. Incorrect This best describes the dissonance stage.
c. CORRECT During the resistance and immersion stage, the individual actively resists the dominant cultural group and becomes immersed in his/her own culture.
d. Incorrect This describes the internalization stage of Cross’s Nigrescence model.

The correct answer is: rejection of the dominant group and preference for one’s own group.

How well did you know this?
1
Not at all
2
3
4
5
Perfectly
10
Q

Research on the use of cognitive-behavioral therapy with older adults has found that it is:
Select one:

A. less effective than cognitive-behavioral therapy for younger adults and, consequently, is generally contraindicated.
B. usually most effective when it is conducted at a slower pace and at higher levels of abstraction.
C. usually most effective when it is short- (versus long) term and conducted at lower levels of abstraction.
D. usually most effective when it focuses on replacing negative cognitions with positive ones and on the present rather than on the past.

A

The research has generally found that older adults benefit from cognitive-behavioral therapies and other forms of therapy to the same degree as do younger adults. However, the effectiveness of therapy with older adults may be enhanced when certain modifications to therapy are made. See, e.g., B. G. Knight and D. Satre, Cognitive behavioral therapy with older adults, Clinical Psychology: Science & Practice, 62(2), 188-203, 1999.

a. Incorrect Cognitive-behavioral therapy has been found to be an effective treatment for older adults for a variety of disorders including depression, anxiety, and insomnia and for coping with physical disability, chronic pain, and grief.
b. CORRECT Knight and Satre note that older adults generally benefit most from cognitive-behavior therapies when therapy is presented at a slower pace and is conducted at an abstract level (i.e., at a level that focuses on schemas and relationship scripts rather than on simple cognitions).
c. Incorrect This is the opposite of what is true. Because many of the problems faced by older adults are chronic, they are more likely to benefit from long-term interventions.
d. Incorrect This is also the opposite of what is true. The emotions of older adults are generally more complex than those of younger adults, and emotional reactions to events often consist of a mixture of positive and negative emotions. Consequently, it is usually best to focus on both the positive and negative connotations of emotions in therapy. In addition, allotting some time in therapy for older adults to reminisce about the past (rather than focusing only on the present) is beneficial.

The correct answer is: usually most effective when it is conducted at a slower pace and at higher levels of abstraction.

How well did you know this?
1
Not at all
2
3
4
5
Perfectly
11
Q

From the perspective of Prochaska and DiClemente’s (1984) transtheoretical model of change, the failures of treatment programs for cigarette smoking are often due to the fact that:
Select one:

A. the majority of smokers are not ready to change.
B. cigarette-smoking is self-reinforcing and difficult to extinguish.
C. nicotine addiction has a genetic component.
D. interventions do not address the environmental cues that trigger relapse.

A

The Prochaska and DiClemente model describes six stages of change: precontemplation, contemplation, preparation, action, maintenance, and termination. Additional information about these stages is provided in the Clinical Psychology chapter of the written study materials.

a. CORRECT Research by Prochaska and colleagues has shown that a majority of individuals with addictions are in the precontemplation or contemplation stage and, therefore, are ambivalent about change. For these individuals, most interventions will be unsuccessful.
b. Incorrect See explanation for response a.
c. Incorrect See explanation for response a.
d. Incorrect See explanation for response a.

The correct answer is: the majority of smokers are not ready to change.

How well did you know this?
1
Not at all
2
3
4
5
Perfectly
12
Q

According to Cross (2001), an African American adult in which of the following stages of identity development is likely to say that racial discrimination is not a contributor to his problems and that he prefers to see a White therapist?
Select one:

A. disintegration
B. pre-encounter
C. emersion
D. pseudo-independence

A

Cross’s Black Racial (Nigrescence) Identity Development Model distinguishes between four stages: pre-encounter, encounter, immersion/emersion, and internalization.

a. Incorrect See explanation for response b.
b. CORRECT According to Cross, race has low salience for individuals in the pre-encounter phase. People in this stage are likely to deny the impact of racial oppression and prefer a White therapist.
c. Incorrect See explanation for response b.
d. Incorrect See explanation for response b.

The correct answer is: pre-encounter

How well did you know this?
1
Not at all
2
3
4
5
Perfectly
13
Q

A therapist relying on a solution-focused family therapy approach would be most interested in which of the following?
Select one:

A. miracle and scaling questions
B. boundary disturbances
C. joining and mimesis
D. schism and skew

A

If you are unfamiliar with solution-focused therapy, you may have been able to choose the correct answer to this question through the process of elimination.

a. CORRECT Solution-focused therapists use various types of questions to identify solutions to a client’s problems. The miracle question involves asking what the individual would notice first if, as the result of a miracle, his/her problem is suddenly gone. Scaling questions are used to determine how a client currently feels.
b. Incorrect Boundary disturbances are of concern to Gestalt psychologists.
c. Incorrect Joining and mimesis are techniques used by structural family therapists.
d. Incorrect Marital schism and skew are terms used by Theodore Lidz to describe dysfunctional marital relationships.

The correct answer is: miracle and scaling questio

How well did you know this?
1
Not at all
2
3
4
5
Perfectly
14
Q

A man who smokes two packs a day says he has decided to stop smoking the day after his birthday, which is two weeks away. According to Prochaska and DiClemente (1992), this man is in which of the following stages of the change process?
Select one:

A. precontemplation
B. contemplation
C. preparation
D. action

A

Prochaska and DiClemente (1992) distinguish between six stages in the change process. These include the four listed in the responses plus maintenance and termination stages.

a. Incorrect During this stage, the individual has little insight into the need for change.
b. Incorrect In the contemplation stage, the person is aware of the need for change and is considering change but has not yet committed to it.
c. CORRECT In the preparation stage, the person has a clear intent to take action in the near future.
d. Incorrect In the action stage, the person actually takes steps to bring about change. (In the maintenance stage, he/she consolidates the change and takes steps to prevent relapse.)

The correct answer is: preparation

How well did you know this?
1
Not at all
2
3
4
5
Perfectly
15
Q

In discussing a depressed client, a therapist says the client’s problems are due to the fact that she did not experience normal grief following the death of her mother three years ago. He says that the focus of treatment will be on helping the client go through the grieving process and restore her interests and friendships so that she can overcome her loss. Apparently, this therapist is a practitioner of:
Select one:

A. rational-emotive therapy.
B. interpersonal therapy.
C. object-relations therapy.
D. reality therapy.

A

Although a client’s unresolved grief would be a focus of many forms of treatment, only one of the therapies listed explicitly defines grief as a treatment target.

a. Incorrect See explanation for response b.
b. CORRECT Interpersonal therapy (IPT) defines four key problem areas presented by depressed patients:

grief
interpersonal role disputes
role transition and
interpersonal deficits.

These areas are the primary targets of therapy.

c. Incorrect See explanation for response b.
d. Incorrect See explanation for response b.

The correct answer is: interpersonal therapy.

How well did you know this?
1
Not at all
2
3
4
5
Perfectly
16
Q

For gay and lesbian adolescents, the primary presenting problem at social service agencies is:
Select one:

A. suicidal ideation.
B. drug or alcohol abuse.
C. social and emotional isolation.
D. academic problems.

A

Gay and lesbian adolescents face similar problems as their heterosexual peers. However, their identity development is also complicated by stigmatization and a lack of role models.

a. Incorrect See explanation for response c.
b. Incorrect See explanation for response c.
c. CORRECT Isolation is frequently cited as a primary problem for gay and lesbian adolescents, and one study found it to be the single-most frequent presenting problem (A. D. Martin and E. S. Hetrick, The stigmatization of the gay and lesbian adolescent, Journal of Homosexuality, 15(1-2), 163-183, 1988).
d. Incorrect See explanation for response c.

The correct answer is: social and emotional isolation.

How well did you know this?
1
Not at all
2
3
4
5
Perfectly
17
Q

Members of an outpatient therapy group are likely to rank which of Yalom’s therapeutic factors as most important?
Select one:

A. interpersonal input, catharsis, cohesiveness, and self-understanding
B. self-understanding, universality, identification, and instillation of hope
C. family re-enactment, catharsis, altruism, and interpersonal input
D. interpersonal input, interpersonal output, universality, and identification

A

In his book, The Theory and Practice of Group Therapy, Yalom (1970) distinguishes between 12 therapeutic (curative) factors and discusses the results of research examining the importance of these factors for group therapists and for group therapy members.

a. CORRECT Although the results vary somewhat from study to study, these four factors are usually the most highly rated ones by group therapy members.
b. Incorrect Identification is usually among the factors that are rated least important.
c. Incorrect Family re-enactment is also rated as less important by group members.
d. Incorrect As noted above, identification is not rated as being particularly important by group members.

The correct answer is: interpersonal input, catharsis, cohesiveness, and self-understanding

How well did you know this?
1
Not at all
2
3
4
5
Perfectly
18
Q

According to the Health Belief Model, health behavior is related to:
Select one:

A. a combination of information, motivation, and behavioral skills.
B. perceived severity, perceived susceptibility, and perceived benefits.
C. habit.
D. intentions and perceived norms.

A

Several models have been developed to predict when people will act in ways that benefit their health.

a. Incorrect These are components of the Information-Motivation-Behavioral Skills model of AIDS prevention.
b. CORRECT The Health Belief Model is based on the premise that perceived severity of consequences, perceived susceptibility to consequences, and perceived benefits of preventive behavior all contribute to health behavior.
c. Incorrect Habit is not a component of the Health Belief Model.
d. Incorrect This sounds more like the Theory of Reasoned Action.

The correct answer is: perceived severity, perceived susceptibility, and perceived benefits.

How well did you know this?
1
Not at all
2
3
4
5
Perfectly
19
Q

An existential therapist is most likely to describe “existential anxiety” as:
Select one:

A. the result of unresolved intrapsychic conflicts.
B. a potential stimulus for growth.
C. the result of a boundary disturbance.
D. a manifestation of incongruence between self and experience.

A

Existential therapists view anxiety as an inevitable condition of life and distinguish between normal and neurotic anxiety.

a. Incorrect This is more consistent with the Freudian view of anxiety.
b. CORRECT For existential therapists, neurotic anxiety is out of proportion to the situation and can be immobilizing. In contrast, normal (existential) anxiety can serve as a source of motivation to change.
c. Incorrect According to Gestalt psychologists, boundary disturbances are a source of neurotic behavior.
d. Incorrect This answer is consistent with Carl Rogers’s view of anxiety.

The correct answer is: a potential stimulus for growth.

How well did you know this?
1
Not at all
2
3
4
5
Perfectly
20
Q

A practitioner of Rogers’s client-centered therapy would use the Q-sort technique to:
Select one:

A. facilitate identifying a client’s diagnosis.
B. help the client identify specific therapy goals.
C. help identify solutions to a client’s presenting problems.
D. evaluate a clients progress in therapy.

A

The Q-sort technique involves rank-ordering descriptive statements that are printed on separate cards. It requires the sorter to distribute the cards into a fixed number of piles along a continuum (for example, a continuum ranging from “not at all characteristic of me” to “extremely characteristic of me”) and to do so in a way so that the piles create a normal distribution. In therapy, this technique is often used to evaluate a client’s self-perceptions.

a. Incorrect See explanation for response d.
b. Incorrect See explanation for response d.
c. Incorrect See explanation for response d.

d. CORRECT Rogers used the Q-sort technique to evaluate therapy progress by evaluating a client’s degree of congruence between his/her self and ideal self. This involved having the client first sort the cards in terms of how well they describe the client’s current self and then in terms of his/her ideal self. The more similar the two sorts, the greater the congruence between the self and ideal self.

The correct answer is: evaluate a clients progress in therapy.

How well did you know this?
1
Not at all
2
3
4
5
Perfectly
21
Q

In treating a client, a practitioner of Reality Therapy would most likely be interested in which of the following?
Select one:

A. the client’s sense of responsibility and needs for power, freedom, and belongingness
B. the client’s cognitive distortions (e.g., selective inattention, misperceptions)
C. the nature of the client’s ego state pathology
D. the client’s level of “response-ability”

A

Reality Therapy emphasizes fulfilling one’s needs in a responsible way.

a. CORRECT For practitioners of Reality Therapy, behavior is motivated primary by biological and psychological needs (e.g., needs for belongingness, power, and freedom). The ability to fulfill one’s needs in a way that doesn’t deprive others of the ability to do the same (i.e., in a responsible way) is the goal of therapy.
b. Incorrect This sounds more like cognitive therapy.
c. Incorrect This sounds more like Transactional Analysis.
d. Incorrect This is a goal of Gestalt therapy. It refers to the ability to act, decide, choose, and otherwise respond.

The correct answer is: the client’s sense of responsibility and needs for power, freedom, and belongingness

How well did you know this?
1
Not at all
2
3
4
5
Perfectly
22
Q

Which of the following information-processing strategies is characteristic of the reintegration status of Helms’s (1995) White Racial Identity Development Model?
Select one:

A. suppression of information and ambivalence
B. obliviousness and denial
C. flexibility and complexity
D. selective perception and negative out-group distortion

A

Helms’s (1995) White Racial Identity Development Model distinguishes between six stages or statuses:

  1. contact (Obliviousness and denial are characteristic of the contact status.)
  2. disintegration (Suppression of information and ambivalence are characteristic of the disintegration status)
  3. reintegration (A combination of selective perception and negative out-group distortion is characteristic.)
  4. pseudo-independence
  5. immersion-emersion
  6. autonomy (Flexibility and complexity are characteristic of the autonomy status.)

Each status is characterized by a different information-processing strategy (IPS), which are the methods the individual uses to reduce discomfort related to racial issues.

a. Incorrect Suppression of information and ambivalence are characteristic of the disintegration status.
b. Incorrect Obliviousness and denial are characteristic of the contact status.
c. Incorrect Flexibility and complexity are characteristic of the autonomy status.
d. CORRECT The combination of selective perception and negative out-group distortion is characteristic of the reintegration status. A person in this stage attempts to resolve moral conflicts about racism by idealizing White society and denigrating members of minority groups.

The correct answer is: selective perception and negative out-group distortion

How well did you know this?
1
Not at all
2
3
4
5
Perfectly
23
Q

The traditional Hawaiian practice of ho’oponopono is best described as a:
Select one:

A. primary prevention.
B. family intervention.
C. type of individual psychotherapy.
D. method for promoting relaxation.

A

Being a culturally competent practitioner requires being familiar with indigenous healing practices that might be appropriate for culturally diverse clients.

a. Incorrect See explanation for response b.
b. CORRECT Ho’oponopono (“setting it right”) is a traditional Hawaiian spiritual healing ritual for restoring harmony among family members by resolving a current conflict or other interpersonal problem. It is a structured process that is led by a senior family member or other respected elder and incorporates prayer, discussions aimed at identifying and resolving the problem, and a closing ceremony.
c. Incorrect See explanation for response b.
d. Incorrect See explanation for response b.

The correct answer is: family intervention.

How well did you know this?
1
Not at all
2
3
4
5
Perfectly
24
Q

From the perspective of structural family therapy, triangulation, parent-child coalition, and detouring are:
Select one:

A. methods for establishing clearer boundaries.
B. methods for maintaining homeostasis.
C. types of positive feedback.
D. attempts to increase intimacy.

A

As its name implies, structural family therapy focuses on the family structure, especially its boundaries. Also, like many other forms of family therapy, it is based on systems theory.

a. Incorrect Although boundaries are important in structural family therapy, the phenomena listed in the question do not always create “clearer boundaries.”
b. CORRECT The function of these three processes is to reduce conflict or stress and thereby maintain the status quo.
c. Incorrect These phenomena are probably better conceptualized as sources of negative feedback since their function is to maintain a state of homeostasis.
d. Incorrect This is not the function of these processes as defined by Minuchin.

The correct answer is: methods for maintaining homeostasis.

How well did you know this?
1
Not at all
2
3
4
5
Perfectly
25
Q

Research on Helms’s White Racial Identity Development Model has shown that White therapists are most effective in cross-cultural counseling situations when they are in which stage of identity development?
Select one:

A. the stage that matches the client’s stage of identity development
B. the stage that complements the client’s stage of identity development
C. the reintegration stage
D. the autonomy stage

A

Helms’s (1995) model distinguishes between six stages of identity development:

Contact
Disintegration
Reintegration
Pseudo-independence
Immersion-emersion, and 
Autonomy.

a. Incorrect See explanation for response d.
b. Incorrect See explanation for response d.
c. Incorrect See explanation for response d.
d. CORRECT The research on Helms’s model has found that, the more advanced the White therapist’s identity development, the greater his/her effectiveness when working with clients from culturally diverse groups.

The correct answer is: the autonomy stage

How well did you know this?
1
Not at all
2
3
4
5
Perfectly
26
Q

When working with a client belonging to an ethnic minority group, a White therapist interprets the client’s reluctance to disclose personal information as a sign of paranoia. According to Ridley (2005), this therapist is exhibiting which of the following?
Select one:

A. marginalization
B. overidentification
C. color blindness
D. cultural ambivalence

A

C. R. Ridley distinguishes between ten race-related defense mechanisms: color blindness, color consciousness, cultural transference, cultural countertransference, cultural ambivalence, pseudotransference, overidentification, identification with the oppressor, race-based misdiagnosis, and avoidance of race (Overcoming unintentional racism in counseling and therapy: A practitioner’s guide to intentional intervention, Thousand Oaks, CA, Sage Publications, 2005).

a. Incorrect Marginalization is not one of the race-related defense mechanisms identified by Ridley.
b. Incorrect Overidentification occurs when a minority therapist overidentifies with the experiences of a minority client and assumes that the client’s presenting problems are race-related.
c. CORRECT As defined by Ridley, color blindness refers to a therapist’s “illusion that minority clients are no different than non-minority clients” (p.67). He notes that “color blind” therapists tend to overlook the effects of racism and discrimination and, consequently, view deviations from White middle-class norms as pathological.
d. Incorrect According to Ridley, cultural ambivalence refers to a therapist’s ambivalent motivations for treating members of ethnic minority groups (e.g., needs for power and dominance versus needs for acceptance and approval).

The correct answer is: color blindness

How well did you know this?
1
Not at all
2
3
4
5
Perfectly
27
Q

The presence of which of the following symptoms would suggest a diagnosis of Conduct Disorder rather than a diagnosis of Oppositional Defiant Disorder?
Select one:

A. frequent lying and running away from home
B. low frustration tolerance and temper outbursts
C. drug use
D. onset of symptoms after age 12

A

Conduct Disorder involves a pattern of behavior in which the basic rights of others and major age-appropriate social norms or rules are violated.

a. CORRECT Lying and running away are characteristic of Conduct Disorder. Behaviors associated with Oppositional Defiant Disorder are less severe and include negativism, defiance, and hostility.
b. Incorrect Low tolerance for frustration and temper outbursts are characteristic of both disorders.
c. Incorrect The use of drugs does not help distinguish between the two disorder.
d. Incorrect The onset of both disorders is typically prepubertal.

The correct answer is: frequent lying and running away from home

How well did you know this?
1
Not at all
2
3
4
5
Perfectly
28
Q

The research has found that individual therapy, group therapy, and family therapy are effective treatments for Bulimia Nervosa. With regard to individual therapy, the research has shown that:
Select one:

A. behavior therapy is superior to either cognitive-behavior therapy or interpersonal therapy in terms of both short- and long-term effects.
B. cognitive-behavior therapy is superior to either behavior therapy or interpersonal therapy in terms of both short-term and long-term effects.
C. cognitive-behavior therapy is superior to either interpersonal therapy or behavior therapy in terms of short-term effects, but cognitive-behavior therapy and interpersonal therapy are about equally effective in terms of long-term effects.
D. behavior therapy, cognitive-behavior therapy, and interpersonal therapy are about equally effective in terms of short-term effects, but cognitive-behavior therapy is superior in terms of long-term effects.

A

Not surprisingly, cognitive-behavior therapy (which combines a variety of strategies and has been found effective for a number of disorders) has been found to be a beneficial treatment for Bulimia.

a. Incorrect See explanation for response c.
b. Incorrect See explanation for response c.
c. CORRECT C. G. Fairburn et al. (Psychotherapy and bulimia nervosa: Long-term effects in interpersonal therapy, behavior therapy, and cognitive-behavior therapy, Archives of General Psychiatry, 50, 419-428, 1993), for example, compared behavior therapy, cognitive-behavior therapy, and interpersonal therapy and found cognitive-behavior therapy to be superior in terms of short-term effects. However, a long-term follow-up revealed that cognitive-behavior therapy and interpersonal therapy had similar effects on both eating behaviors and attitudes toward weight and shape.

d. Incorrect See explanation for response c.
The correct answer is: cognitive-behavior therapy is superior to either interpersonal therapy or behavior therapy in terms of short-term effects, but cognitive-behavior therapy and interpersonal therapy are about equally effective in terms of long-term effects.

How well did you know this?
1
Not at all
2
3
4
5
Perfectly
29
Q

The psychoanalyst Adolph Stern provided the first organized clinical description of the borderline patient. Of the ten basic characteristics Stern delineated, which of the following did he consider to be the most primary?
Select one:

A. inhibited aggression
B. fixation
C. emotional dysregulation
D. narcissism

A

Stern described the borderline patient in terms of 10 basic characteristics. Only one of these is listed in the responses.

a. Incorrect See explanation for response d.
b. Incorrect See explanation for response d.
c. Incorrect See explanation for response d.
d. CORRECT Stern considered the difficulties experienced by the borderline patient to be secondary to narcissism and viewed narcissism as arising from a serious disturbance in the early mother-child relationship. (Note that other psychoanalytically-oriented theorists have identified other factors as being primary: Kernberg, for example, emphasizes the role of excessive aggression.)

The correct answer is: narcissism

How well did you know this?
1
Not at all
2
3
4
5
Perfectly
30
Q

When using the DSM-5, level of severity of Intellectual Disability is based on:
Select one:

A. The individual’s score on a standardized intelligence test.
B. The individual’s adaptive functioning in conceptual, social, and practical domains.
C. The degree of discrepancy between the individual’s cognitive and adaptive functioning.
D. The degree of discrepancy between the individual’s IQ and academic achievement.

A

Answer B is correct: The DSM-5 distinguishes between four levels of severity for Intellectual Disability - mild, moderate, severe, and profound - and bases them on level of adaptive functioning in conceptual, social, and practical domains.

The correct answer is: The individual’s adaptive functioning in conceptual, social, and practical domains.

How well did you know this?
1
Not at all
2
3
4
5
Perfectly
31
Q

The clinical course of AIDS dementia complex (ADC) is described in the literature in terms of six stages. For example, a person with AIDS who has unequivocal evidence of functional, intellectual, or motor impairment but is able to perform all but the most demanding aspects of activities of daily living and can walk without assistance is in which of the following stages?
Select one:

A. Stage 0.5 (equivocal/subclinical)
B. Stage 1 (mild)
C. Stage 2 (moderate)
D. Stage 3 (severe)

A

AIDS dementia complex (ADC) has been estimated to affect up to one-third of adults and one-half of children with AIDS.

a. Incorrect Stage 0.5 is characterized by minimal or equivocal signs of impairment with no deficits in work or activities of daily living.
b. CORRECT The symptoms listed in the question are characteristic of Stage 1.
c. Incorrect A person in Stage 2 cannot work or perform demanding activities of daily living and may require assistance when walking.
d. Incorrect A person in Stage 3 has significant intellectual impairments and cannot walk unassisted. Additional information about the stages of ADC is provided in the Abnormal Psychology chapter of the written study materials.

The correct answer is: Stage 1 (mild)

How well did you know this?
1
Not at all
2
3
4
5
Perfectly
32
Q

For a diagnosis of Bulimia Nervosa, an individual has to exhibit which of the following?
Select one:

A. purging following binge eating for at least one month
B. binge eating for at least two months
C. lack of control over eating plus either purging or excessive exercise for at least two months
D. binge eating and inappropriate compensatory behavior for at least three months

A

The key features of Bulimia Nervosa are binge eating and recurrent inappropriate compensatory behavior (e.g., purging, diuretic use, excessive exercising) for at least three months.

a. Incorrect See explanation for response d.
b. Incorrect See explanation for response d.
c. Incorrect See explanation for response d.
d. CORRECT As noted above, this is consistent with the diagnostic criteria for this disorder.

The correct answer is: binge eating and inappropriate compensatory behavior for at least three months

How well did you know this?
1
Not at all
2
3
4
5
Perfectly
33
Q

Of the 6 to 10% of adults over 65 who have dementia, what percent have Alzheimer’s disease?
Select one:

A. 85 to 95
B. 65 to 75
C. 40 to 50
D. 20 to 35

A

Estimates of the prevalence rates for Alzheimer’s disease and other forms of dementia vary, with rates increasing as age increases.

a. Incorrect See explanation for response b.
b. CORRECT The studies indicate that, of adults over 65 with dementia, between two-thirds to three-fourths have Alzheimer’s disease. See, e.g., H. C. Hendrie, Epidemiology of dementia and Alzheimer’s disease, American Journal of Geriatric Psychiatry, 6(2 Suppl. 1), S3-18, 1998.
c. Incorrect See explanation for response b.

d. Incorrect See explanation for response b.
The correct answer is: 65 to 75

How well did you know this?
1
Not at all
2
3
4
5
Perfectly
34
Q

An electrolyte imbalance is a possible complication of Bulimia Nervosa. The danger of this complication lies in the fact that it can lead to:
Select one:

A. cardiac arrhythmia and arrest.
B. blood dyscracias.
C. permanent memory loss.
D. dehydration.

A

Of the conditions listed, only one is a consequence of an electrolyte imbalance.

a. CORRECT Electrolyte imbalances caused by vomiting and the use of laxatives and diuretics can have serious consequences, including, in extreme cases, cardiac arrhythmia and arrest.
b. Incorrect Blood dyscracias are not caused by electrolyte imbalances.
c. Incorrect Permanent memory loss has not be linked to electrolyte imbalances.
d. Incorrect Dehydration is one of the causes (not consequences) of an electrolyte imbalance.

The correct answer is: cardiac arrhythmia and arrest.

How well did you know this?
1
Not at all
2
3
4
5
Perfectly
35
Q

A graduate student is extremely anxious about a lecture he has to give to undergraduate students the next morning. To reduce his anxiety, he drinks two beers. According to Steele and Joseph (1990), the student’s anxiety will actually increase in this situation if he:
Select one:

A. relaxes and “does nothing.”
B. performs a simple task.
C. performs a moderately demanding task.
D. performs a very demanding task.

A

C. M. Steele and R. A. Joseph have developed an attention-allocation model to explain why alcohol sometimes reduces anxiety but other times has no effect or actually increases anxiety (Alcohol myopia: Its prized and dangerous effects, American Psychologist, 45(8), 921-933, 1990).

a. CORRECT According to Steele and Joseph, alcohol creates a “myopia” in that it restricts the drinker’s attention to the most salient aspect of the situation and reduces attention to other, less salient aspects. Consequently, if an intoxicated person engages in a distracting task, he/she will worry less about the anxiety-arousing stimulus (e.g., giving a lecture). Conversely, if the person does not engage in a distracting task, he/she will focus on the anxiety-arousing stimulus and actually become more anxious about it.
b. Incorrect See explanation for response a.
c. Incorrect See explanation for response a.

d. Incorrect See explanation for response a.
The correct answer is: relaxes and “does nothing.”

How well did you know this?
1
Not at all
2
3
4
5
Perfectly
36
Q

Longitudinal studies of individuals who receive a diagnosis of Borderline Personality Disorder in adolescence or early adulthood indicate that these individuals often exhibit a reduction or remission in symptoms over time. However, recovery varies for type of symptom, with _________ symptoms showing the least amount of improvement with increasing age.
Select one:

A. impulsive
B. interpersonal
C. affective
D. cognitive

A

Of the Personality Disorders, Borderline Personality Disorder (BPD) has been found to have the best prognosis, with the majority of patients showing a significant improvement in or remission of symptoms over time.

a. Incorrect See explanation for response c.
b. Incorrect See explanation for response c.
c. CORRECT Longitudinal studies have found that, by middle age or sooner, most individuals with BPD no longer meet the diagnostic criteria for the disorder. The resolution of symptoms varies, however: For example, in a six-year prospective study of 290 patients with BPD, Zanarini et al. found that impulsive symptoms resolved most quickly, affective symptoms were the most chronic, and cognitive and interpersonal symptoms were intermediate in terms of resolution (M. C. Zanarini, F. R. Frankenburg, J. Hennen, and K. R. Silk, The longitudinal course of borderline pathology: 6-year prospective follow-up of the phenomenology of borderline personality disorder, American Journal of Psychiatry, 160, 274-283, 2003).
d. Incorrect See explanation for response c.

The correct answer is: affective

How well did you know this?
1
Not at all
2
3
4
5
Perfectly
37
Q

During the first few sessions with a male client, you learn that he has trouble maintaining friendships and is disturbed by this since he doesn’t like being alone, has frequent mood shifts, gets very angry about even minor irritations, isn’t sure what he wants to do with his life, and has had three different jobs in the past two years. Based on this information, the best diagnosis is:
Select one:

A. Borderline Personality Disorder.
B. Narcissistic Personality Disorder.
C. Histrionic Personality Disorder.
D. Schizoid Personality Disorder.

A

To distinguish between different Personality Disorders, you need to be familiar with their essential features, and these are summarized in the Abnormal Psychology chapter of the written study materials.

a. CORRECT The pattern of instability in mood, relationships, and occupation is characteristic of Borderline Personality Disorder.
b. Incorrect The man isn’t exhibiting the grandiosity that characterizes Narcissistic Personality Disorder.
c. Incorrect Although shifts in mood are typical of Histrionic Personality Disorder, the man isn’t exhibiting attention-seeking behavior.
d. Incorrect Schizoid Personality Disorder is characterized by restricted emotions and detachment from interpersonal relationships.

The correct answer is: Borderline Personality Disorder.

How well did you know this?
1
Not at all
2
3
4
5
Perfectly
38
Q

Fairburn’s (2008) CBT-E (cognitive-behavioral therapy - enhanced) adds which of the following to traditional cognitive-behavioral therapy for individuals with Bulimia Nervosa?
Select one:

A. interventions aimed at boundary disturbances
B. interventions aimed at interpersonal relationships
C. interventions that foster insight into the cause of the disorder
D. interventions derived from Rogerian therapy

A

Cognitive-behavioral therapy is considered the most effective treatment for Bulimia Nervosa. However, several experts have suggested that the effectiveness of a traditional approach to CBT is improved if it includes a greater focus on other issues such as emotional responding and interpersonal relationships.

a. Incorrect See explanation for response b.
b. CORRECT C. G. Fairburn’s CBT-E incorporates interventions that target perfectionism, low self-esteem, interpersonal factors, and emotional factors (Cognitive behavior therapy and eating disorders, New York, Guilford Publ., 2008).
c. Incorrect See explanation for response b.

d. Incorrect See explanation for response b.
The correct answer is: interventions aimed at interpersonal relationships

How well did you know this?
1
Not at all
2
3
4
5
Perfectly
39
Q

The most effective intervention for cigarette smoking combines nicotine replacement therapy with:
Select one:

A. antidepressants and bibliotherapy.
B. support from a clinician and skills training.
C. covert sensitization and relapse prevention.
D. stimulus control and habit reversal training.

A

Reviews of the literature conclude that combined treatments are most effective for cigarette smoking, especially in terms of long-term consequences (e.g., American Psychiatric Assoc., 1996).

a. Incorrect See explanation for response b.
b. CORRECT There is evidence that NRT is most effective in terms of long-term effects when it is combined with guidance and support from a mental health professional and skills training that focuses on ways for avoiding and dealing with relapse.
c. Incorrect See explanation for response b.
d. Incorrect See explanation for response b.

The correct answer is: support from a clinician and skills training.

How well did you know this?
1
Not at all
2
3
4
5
Perfectly
40
Q

Which of the following is true about sleep terrors and sleepwalking?
Select one:

A. Both occur during REM sleep.
B. Both are usually followed by amnesia for the event.
C. Both are associated with an impaired homeostatic sleep drive.
D. Both are accompanied by difficulty awakening in the morning with morning confusion.

A

Answer B is correct: In the DSM-5, sleep terrors and sleepwalking are types of Non-Rapid Eye Movement Sleep Arousal Disorder. Both types occur during non-REM sleep and are characterized by amnesia for the episode.

Answer A: This is the opposite of what is true - both occur during non-REM sleep.

Answer C: An impaired homeostatic sleep drive has been linked to Circadian Rhythm Sleep-Wake Disorder.

Answer D: Difficulty awakening and morning confusion are associated with Circadian Rhythm Sleep-Wake Disorder.

The correct answer is: Both are usually followed by amnesia for the event.

How well did you know this?
1
Not at all
2
3
4
5
Perfectly
41
Q

When using the DSM-5, a clinician would code which of the following to indicate that a client has symptoms that do not meet the diagnostic criteria for a specific disorder but does not want to specify the reason why?
Select one:

A. [Disorder] not otherwise specified
B. [Disorder] provisional
C. other specified disorder
D. unspecified disorder

A

Answer D is correct: When using the DSM-5, diagnostic uncertainty about a client’s diagnosis is indicated by coding one of the following: Other specified disorder is coded when the clinician wants to indicate the reason why the client’s symptoms do not meet the criteria for a specific diagnosis (e.g., “other specified depressive disorder, recurrent brief depression”); and unspecified disorder is coded when the clinician does not want to indicate the reason why the client’s symptoms do not meet the criteria for a specific diagnosis (e.g., unspecified depressive disorder).

Answer A: See explanation for answer D.

Answer B: When using the DSM-5, the provisional specifier applies when “there is a strong presumption that the full criteria will ultimately be met for a disorder but not enough information is available to make a firm diagnosis” (p. 23).

Answer C: See explanation for answer D.

The correct answer is: unspecified disorder

How well did you know this?
1
Not at all
2
3
4
5
Perfectly
42
Q

Studies investigating anxiety over the lifespan suggest that, when compared to younger adults, older adults are:
Select one:

A. more likely to be misdiagnosed as having an anxiety disorder.
B. much less likely to benefit from pharmacotherapy or cognitive-behavioral therapy.
C. less likely to have comorbid symptoms of depression.
D. more likely to believe their symptoms are due to physical health problems.

A

Studies indicate that anxiety is the most common psychiatric disorder in older adults, with generalized anxiety disorder being the most prevalent anxiety disorder.

a. Incorrect In fact, underdiagnosis and undertreatment of anxiety disorders are more common among older (versus younger) adults.
b. Incorrect The research has generally found that older adults with anxiety benefit to about the same degree as younger adults from pharmacotherapy and CBT.
c. Incorrect Younger and older adults with anxiety frequently have comorbid symptoms of depression, but this may actually occur more often in older adults.
d. CORRECT Older adults are more likely than younger adults to attribute their anxiety symptoms to physical health problems and, as a result, to seek help from a medical (versus mental health) professional. See, e.g., M. A. Stanley and J. G. Beck, Anxiety disorders, Clinical Psychology Review, 2000, 20, 731-754.

The correct answer is: more likely to believe their symptoms are due to physical health problems.

How well did you know this?
1
Not at all
2
3
4
5
Perfectly
43
Q

Electroconvulsive shock therapy (ECT) is considered an effective intervention for severe depression that has not been responsive to other treatments or is accompanied by a high suicide risk. However, ECT also often produces adverse side effects including:
Select one:

A. patchy anterograde amnesia.
B. temporary retrograde amnesia.
C. patchy anterograde amnesia and temporary retrograde amnesia.
D. permanent retrograde amnesia.

A

The side effects of ECT vary from patient to patient but usually include some degree of memory loss.

a. Incorrect See explanation for response c.
b. Incorrect See explanation for response c.
c. CORRECT Memory loss most often involves patchy anterograde amnesia for three to six months post-ECT and retrograde amnesia for events that occurred within several months prior to ECT. In most cases memories of past events eventually return.
d. Incorrect Permanent retrograde amnesia is relatively uncommon, so this is not the best response.

The correct answer is: patchy anterograde amnesia and temporary retrograde amnesia.

How well did you know this?
1
Not at all
2
3
4
5
Perfectly
44
Q

Augustine, age 5, is in foster care and enrolled in a therapeutic nursery school. He and his older sister were removed from their home after they were found wandering in the park asking people for food and their parents were both found to have a drug problem. While observing Augustine at school, you notice that he impulsively approaches strangers, tends to be overly familiar with people he has just met, and quickly becomes attached to any adult who pays attention to him. Before assigning a DSM-5 diagnosis of ______________ to Augustine, you will want to confirm that his symptoms are attributable to ______________.
Select one:

A. Reactive Attachment Disorder, disinhibited type; developmental delays
B. Disinhibited Social Engagement Disorder; developmental delays
C. Reactive Attachment Disorder, disinhibited type; early neglect or deprivation
D. Disinhibited Social Engagement Disorder; early neglect or deprivation

A

Answer D is correct: Disinhibited Social Engagement Disorder involves a pattern of culturally inappropriate and overly familiar behavior with unfamiliar people with evidence that the behavior is related to the experience of extremely insufficient care.

Answer A: The presence or absence of developmental delays is not a diagnostic criterion for a diagnosis of Disinhibited Social Engagement Disorder or Reactive Attachment Disorder and, therefore, would not be useful for confirming either diagnosis, which is what this question is asking about.

Answers B and C: See explanations for answers A and D.

The correct answer is: Disinhibited Social Engagement Disorder; early neglect or deprivation

How well did you know this?
1
Not at all
2
3
4
5
Perfectly
45
Q

In terms of sexual orientation, most men who receive a diagnosis of Transvestic Disorder identify themselves as:
Select one:

A. solely or predominantly heterosexual.
B. solely or predominantly homosexual.
C. unequivocally bisexual.
D. unequivocally asexual.

A

Answer A is correct: According to the DSM-5 the majority of men with Transvestic Disorder identify themselves as heterosexual, although some have occasional sexual relations with other men, especially when cross-dressed.

Answers B, C, and D: See explanation for answer A.

The correct answer is: solely or predominantly heterosexual.

How well did you know this?
1
Not at all
2
3
4
5
Perfectly
46
Q

Hypnagogic hallucinations are:
Select one:

A. misperceptions of real stimuli.
B. sensations perceived in the wrong sensory modality.
C. false perceptions that occur when falling asleep.
D. false perceptions associated with hallucinogenic use.

A

Hypnagogic hallucinations are not necessarily indicative of a mental disorder but are associated with Narcolepsy.

a. Incorrect This sounds more like an illusion.
b. Incorrect This is referred to as synesthesia.
c. CORRECT This is the definition of hypnagogic hallucinations.
d. Incorrect These are simply hallucinations.

The correct answer is: false perceptions that occur when falling asleep.

How well did you know this?
1
Not at all
2
3
4
5
Perfectly
47
Q

Research on the comorbidity of the eating disorders suggests that the rates of OCD, Social Phobia, and Specific Phobia are:
Select one:

A. higher in individuals with Anorexia Nervosa and Bulimia Nervosa than in other individuals and that the onset of the anxiety disorder is most often prior to the onset of the eating disorder.
B. higher in individuals with Anorexia Nervosa or Bulimia Nervosa than in other individuals and the onset of the anxiety disorder is most often simultaneous with the onset of the eating disorder.
C. higher in individuals with Anorexia Nervosa or Bulimia Nervosa than in other individuals and the onset of the anxiety disorder is most often after the onset of the eating disorder.
D. similar in individuals with Anorexia Nervosa or Bulimia Nervosa as the rates for other individuals and the onset is most often simultaneous with the onset of the eating disorder.

A

The research has generally found that the rates of several anxiety disorders are higher among individuals with an eating disorder than among individuals in the general population.

a. CORRECT The majority of studies have found that the onset of the anxiety disorder precedes the onset of the eating disorder. See, e.g., W. H. Kaye et al., Comorbidity of anxiety disorders with anorexia nervosa and bulimia nervosa, American Journal of Psychiatry, 161, 2215-2221, 2004.
b. Incorrect See explanation above.
c. Incorrect See explanation above.
d. Incorrect See explanation above.

The correct answer is: higher in individuals with Anorexia Nervosa and Bulimia Nervosa than in other individuals and that the onset of the anxiety disorder is most often prior to the onset of the eating disorder.

How well did you know this?
1
Not at all
2
3
4
5
Perfectly
48
Q

Your new client, Elwood E., age 28, says that, for as long as he can remember, he has had trouble finishing projects because of his tendency to repeatedly check for mistakes and desire to achieve perfection in whatever he does. He says that, because of these tendencies, he received several “incompletes” when he was in college and was fired from his last job. Elwood also reports that he has constant thoughts and impulses that he knows are inappropriate, that make him very anxious, but that he cannot control. He says he’s afraid he’s going to hurt one of his family members by forgetting to turn off the stove or by accidentally leaving a door or window unlocked at night and that, for this reason, he spends a great deal of time checking and rechecking the stove, doors, and windows. Based on these symptoms, the most likely diagnosis or diagnoses for Elwood is/are:
Select one:

A. Obsessive-Compulsive Disorder and Generalized Anxiety Disorder.
B. Obsessive-Compulsive Disorder and Obsessive-Compulsive Personality Disorder.
C. Obsessive-Compulsive Personality Disorder and Impulse Control Disorder NOS.
D. Obsessive-Compulsive Personality Disorder only.

A

Elwood is exhibiting symptoms characteristic of two disorders.

a. Incorrect A diagnosis of Generalized Anxiety Disorder (GAD) requires the presence of excessive anxiety and worry about a number of events. Elwood’s concerns about safety sound more like true obsessions than symptoms of GAD. In addition, Elwood’s other symptoms suggest that he has Obsessive-Compulsive Personality Disorder.
b. CORRECT Elwood’s preoccupation with perfection and orderliness and the duration of these symptoms (“for as long as he can remember”) are characteristic of Obsessive-Compulsive Personality Disorder. Elwood also has obsessions and compulsions which are not characteristic of Obsessive-Compulsive Personality Disorder but are symptoms of Obsessive-Compulsive Disorder.
c. Incorrect Elwood’s symptoms do not meet the diagnostic criteria for an impulse control disorder. Also, the behaviors associated with this disorder (unlike Elwood’s behaviors) are not performed in response to an obsession.
d. Incorrect See explanation for response b.

The correct answer is: Obsessive-Compulsive Disorder and Obsessive-Compulsive Personality Disorder.

How well did you know this?
1
Not at all
2
3
4
5
Perfectly
49
Q

A meta-analysis of the research on expressed emotion and relapse by Butzlaff and Hooley (1998) found that high levels of expressed emotion by family members:
Select one:

A. are predictive of relapse for patients with schizophrenia but not for patients with a mood or eating disorder.
B. are predictive of relapse for patients with a mood disorder or schizophrenia but not for those with an eating disorder.
C. are predictive of relapse for patients with an eating disorder or a mood disorder but not for patients with schizophrenia.
D. may be somewhat more predictive of relapse for patients with a mood or eating disorder than for patients with schizophrenia.

A

Expressed emotion has been well-established as a predictor of relapse in patients with schizophrenia, and recent studies have linked it to negative outcomes for a number of other disorders.

a. Incorrect See explanation for response d.
b. Incorrect See explanation for response d.
c. Incorrect See explanation for response d.
d. CORRECT R. L. Butzlaff and J. M. Hooley looked at studies investigating the impact of expressed emotion on outcomes for schizophrenia, mood disorders, and eating disorders and found that high expressed emotion by family members was more strongly predictive of relapse for mood and eating disorders than for schizophrenia, although all effect sizes for all three were significant. Specifically, they obtained weighted mean effect sizes for mood disorders, eating disorders, and schizophrenia of, respectively, .39, .51, and .31. (Expressed emotion and psychiatric relapse: A meta-analysis, Archives of General Psychiatry 55, 547-552, 1998.)

The correct answer is: may be somewhat more predictive of relapse for patients with a mood or eating disorder than for patients with schizophrenia.

How well did you know this?
1
Not at all
2
3
4
5
Perfectly
50
Q

A nine-year old child with Panic Disorder:
Select one:

A. has been misdiagnosed because Panic Disorder does not occur in preadolescent children.
B. is most likely to manifest his disorder as crying, freezing, and clinging to his parents.
C. is most likely to manifest his disorder as shortness of breath, chest pain, and heart palpitations and saying that he “feels like he’s going crazy.”
D. is most likely to manifest his disorder as shortness of breath, chest pain, tachycardia, and school refusal.

A

There is some controversy regarding the ability of children to experience the cognitive symptoms of panic; however, cases of the disorder in children are described in the literature.

a. Incorrect See explanation for response d.
b. Incorrect These symptoms are characteristic for children aged 3 through 5.
c. Incorrect These symptoms are characteristic for adolescents.
d. CORRECT These are the symptoms listed for children aged 6 through 12 in M. L. Wolraich (ed.), The classification of child and adolescent mental diagnosis in primary care: Diagnostic and statistical manual for primary care (DSM-PC), Elk Grove Village, IL, American Academy of Pediatrics, 1996.

The correct answer is: is most likely to manifest his disorder as shortness of breath, chest pain, tachycardia, and school refusal.

How well did you know this?
1
Not at all
2
3
4
5
Perfectly
51
Q

A caller to a suicide helpline is most likely to be which of the following?
Select one:

A. an African-American male
B. an African-American female
C. a White male
D. a White female

A

The few studies that have been conducted to identify the types of callers to suicide helplines indicate that they are consistent with the groups who are at high risk for a suicide attempt. a. Incorrect See explanation for response d.

b. Incorrect See explanation for response d.
c. Incorrect See explanation for response d.
d. CORRECT Young White females are the most frequent callers to suicide helplines. This is not too surprising since whites make up the largest proportion of the population, and females are at a higher risk than males for both depression and a suicide attempt.

The correct answer is: a White female

How well did you know this?
1
Not at all
2
3
4
5
Perfectly
52
Q

A female client says she frequently misperceives things and this has made her anxious about leaving home. For example, she often thinks she sees small animals at work when, in fact, the “animals” are actually inanimate objects such as books, coffee mugs, and office supplies. The woman’s misperceptions are best described as:
Select one:

A. illusions.
B. delusions.
C. hallucinations.
D. depersonalization.

A

For the exam, you want to be familiar with the definitions of the terms listed in the responses to this question.

a. CORRECT An illusion is a misperception of reality (e.g., misperceiving a coffee mug as a rodent).
b. Incorrect A delusion is a false belief about reality that is firmly held regardless of evidence to the contrary.
c. Incorrect While an illusion is elicited by an actual stimulus, an hallucination is a sensory perception in the absence of an external stimulus. Since this women is “misperceiving things,” this suggests she is experiencing illusions rather than hallucinations.
d. Incorrect Depersonalization is an alteration in the sense of self – i.e., the sense that one is detached from one’s mind or body.

The correct answer is: illusions.

How well did you know this?
1
Not at all
2
3
4
5
Perfectly
53
Q

Childhood-onset Obsessive-Compulsive Disorder:
Select one:

A. is about equally common in boys and girls.
B. is four times more common in girls than boys.
C. is two times more common in girls than boys.
D. is more common in boys than girls.

A

For the exam, you want to be familiar with the prevalence rates and gender ratios for several disorders including OCD. The rates and ratios you’re most likely to encounter on exam questions are included in the Abnormal Psychology chapter of the written study materials.

a. Incorrect See explanation for response d.
b. Incorrect See explanation for response d.
c. Incorrect See explanation for response d.
d. CORRECT While OCD is about equally common in male and female adults, because of its earlier onset in males, it is more common in male children than in female children.

The correct answer is: is more common in boys than girls.

How well did you know this?
1
Not at all
2
3
4
5
Perfectly
54
Q

Research on the impact of cultural identification on substance abuse among Native American youth has found that the risk for abuse is:
Select one:

A. lowest for those who are acculturated into the non-Native American (mainstream) culture.
B. lowest for those who can adapt to both Native American and non-Native American cultures.
C. highest for those who strongly identify with the Native American culture and reject the non-Native American culture.
D. highest for those who do not strongly identify with either the Native American or non-Native American culture.

A

The studies have established a link between risk for substance abuse and cultural identity for Native American youth.

a. Incorrect See explanation for response b.
b. CORRECT The studies have shown that either strong identification with one’s own Native American tribe or a bicultural identity is associated with a lower risk for substance use than a strong identification with the mainstream culture only. See, e.g., E. R. Oetting, F. Beauvais, and J. Velarde, Marijuana use by reservation Native American youth, Listening Post (Indian Health Service), 4, 25-28, 1982.
c. Incorrect Strong identification with the Native American culture is associated with a lower risk for substance abuse.
d. Incorrect The highest risk seems to be for those who reject the Native American culture and are highly acculturated into the mainstream.

The correct answer is: lowest for those who can adapt to both Native American and non-Native American cultures.

How well did you know this?
1
Not at all
2
3
4
5
Perfectly
55
Q

Alcohol-Induced Sleep Disorder most often involves which of the following symptoms?
Select one:

A. increased wakefulness, restless sleep, and vivid dreams
B. abnormal physiological and behavioral symptoms during sleep
C. excessively long nocturnal sleep and excessive sleepiness during the day
D. difficulty falling and staying asleep with a reduction in REM (dream) sleep

A

According to the DSM, Alcohol-Induced Sleep Disorder is usually of the insomnia type.

a. CORRECT Although alcohol initially produces sleepiness, this is followed by insomnia, restless sleep, and increased REM sleep, often with vivid, anxiety-laden dreams.
b. Incorrect See explanation above.
c. Incorrect See explanation above.
d. Incorrect See explanation above.

The correct answer is: increased wakefulness, restless sleep, and vivid dreams

How well did you know this?
1
Not at all
2
3
4
5
Perfectly
56
Q

Research by Kaye, Gendall, and Strober (1998) suggests that food restriction associated with Anorexia Nervosa reduces __________ levels.
Select one:

A. serotonin
B. acetylcholine
C. dopamine
D. glutamate

A

Kaye et al. found that anorexia is related to a higher-than-normal levels serotonin in the brain.

a. CORRECT Kaye proposes that high levels of serotonin cause anxiety and that starvation reduces tryptophan, which then reduces brain levels of serotonin and temporarily relieves anxiety. See, e.g., W Kay et al., Serotonin neuronal function and selective serotonin reuptake inhibitors in anorexia and bulimia, Biological Psychiatry, 44(9), 825-838, 1998.
b. Incorrect See explanation for response a.
c. Incorrect See explanation for response a.
d. Incorrect See explanation for response a.

The correct answer is: serotonin

How well did you know this?
1
Not at all
2
3
4
5
Perfectly
57
Q

With regard to information revealed by a client in the context of the client-therapist relationship, who is the “holder of the privilege”?
Select one:

A. the client
B. the therapist
C. the court
D. both the client and the therapist

A

The psychotherapist-patient privilege refers to the legal right of the client to have his/her psychotherapy records and disclosure withheld in a legal proceeding. This privilege is state law in all 50 states, though the laws related to privilege (e.g., exceptions to privilege) vary somewhat from state to state.

a. CORRECT In all states, the client is ordinarily the holder of the privilege, which means that it is up to the client to decide whether to exercise or waive the privilege.
b. Incorrect See explanation for response a.
c. Incorrect See explanation for response a.
d. Incorrect See explanation for response a.

The correct answer is: the client

How well did you know this?
1
Not at all
2
3
4
5
Perfectly
58
Q

The primary function of the psychology licensing board is best described by which of the following?
Select one:

A. establishment of minimal standards of competence
B. limitation of access to the profession
C. protection of the public through the collection of fees for licensure
D. the establishment, monitoring, and enforcement of ethical principles

A

Although the various state and provincial boards have different requirements for licensure, they all set the minimum requirements for licensure.

a. CORRECT Licensing boards establish and monitor entry-level qualifications required to offer services to the public for a fee under the title “psychologist”; by doing so, the boards helps ensure competence.
b. Incorrect The licensing boards do not function to limit access to the profession.
c. Incorrect Although licensing is designed to protect the public, this protection is not accomplished through the collection of fees.
d. Incorrect The APA, not the licensing boards, establishes, monitors, and enforces the ethical principles.

The correct answer is: establishment of minimal standards of competence

How well did you know this?
1
Not at all
2
3
4
5
Perfectly
59
Q

A psychologist is hired by an industrial development corporation to administer tests to help select key personnel. After being hired, the psychologist learns that the corporation is interested in screening potential employees for “homosexual tendencies,” and he is asked by company management to administer a projective test for the purpose of assessing sexual orientation. The psychologist should:
Select one:

A. administer the test to applicants only after obtaining signed informed consents from job applicants.
B. administer the test and make recommendations but do not give management the actual test results.
C. administer the test and give the results to management since it is not his responsibility to decide what information should be used in making employment decisions.
D. refuse to administer the test and explain to management that it is inappropriate to utilize projective tests for this purpose.

A

Ethical guidelines require psychologists to use tests only for purposes for which the tests have been validated. For example, Standard 9.02(a) of the APA’s Ethics Code requires psychologists to use tests only for purposes that are “appropriate in light of the research on or evidence of the usefulness and proper application of the techniques.”

a. Incorrect Consent is not the issue in this situation. Obtaining an informed consent would not resolve the unethical use of a test.
b. Incorrect Use of a projective test to make employment decisions is not considered ethical due to a lack of validity evidence, and the psychologist, therefore, should not administer the test.
c. Incorrect It is the psychologist’s responsibility to ensure that tests and test results are not misused.
d. CORRECT Use of projective tests for this purpose is unethical because (1) projective tests have not been validated as job selection tests and (2) homosexuality has not been shown to be related to job performance. Therefore, using a projective test in the selection process to identify homosexual tendencies violates ethical guidelines.

The correct answer is: refuse to administer the test and explain to management that it is inappropriate to utilize projective tests for this purpose.

How well did you know this?
1
Not at all
2
3
4
5
Perfectly
60
Q

A psychologist is asked to evaluate a prisoner for the purpose of making a recommendation about the prisoner’s readiness for parole. To be consistent with ethical guidelines, the psychologist should:
Select one:

A. refuse to do the evaluation since making a recommendation in this situation is prohibited.
B. conduct the evaluation but base his recommendation on the results of standardized tests only.
C. conduct the evaluation as long as the prisoner has been told the purpose of the evaluation and has been warned about the limits on confidentiality.
D. conduct the evaluation after obtaining a consent from the prisoner’s legal representative and report the results without making a recommendation.

A

Special precaution is required when conducting evaluations and making recommendations in forensic settings.

a. Incorrect Making a recommendation is not prohibited by APA guidelines.
b. Incorrect The techniques used by psychologists must be valid, but there is nothing that says a psychologist can only use standardized tests.
c. CORRECT This response is most consistent with the requirements of Standard 9.03(a) of the Ethics Code and Paragraph 6.03 of the Specialty Guidelines for Forensic Psychology (APA, 2012).
d. Incorrect Again, a recommendation is not prohibited, so this response is not as good as response c.

The correct answer is: conduct the evaluation as long as the prisoner has been told the purpose of the evaluation and has been warned about the limits on confidentiality.

How well did you know this?
1
Not at all
2
3
4
5
Perfectly
61
Q

Dr. Bill sets his clients’ fees on the basis of a “sliding scale” that is based on their current income. This practice is:
Select one:

A. acceptable but not explicitly mentioned in the Ethics Code.
B. unacceptable but not explicitly mentioned in the Ethics Code.
C. explicitly recommended in the Ethics Code.
D. explicitly prohibited in the Ethics Code.

A

Sliding scale fees are not explicitly addressed in the Ethics Code.

a. CORRECT Sliding scale fees are generally considered acceptable as long as they are fair and serve the best interests of the client.
b. Incorrect See explanation above.
c. Incorrect See explanation above.
d. Incorrect See explanation above.

The correct answer is: acceptable but not explicitly mentioned in the Ethics Code.

How well did you know this?
1
Not at all
2
3
4
5
Perfectly
62
Q

Employee X is thinking about filing a sexual harassment suit against her boss, Supervisor Y. He makes a habit of complimenting Employee X on how she looks, often with a “wolf whistle” and this makes Employee X feel very uncomfortable. Employee X decides to tell Supervisor Y that she finds his behavior offensive. Supervisor Y says he “means no harm” but that he’ll stop if that’s what she wants. This situation:
Select one:

A. represents a sexual harassment suit because Supervisor Y is in a position of authority.
B. represents a sexual harassment suit because Supervisor Y’s comments have created a hostile work environment for Employee X.
C. represents a sexual harassment suit only if it is determined that a “reasonable woman” would find Supervisor Y’s behavior offensive.
D. does not represent a sexual harassment suit as long as Supervisor Y actually stops making comments about Employee X’s appearance.

A

Sexual harassment laws and policies continue to be reinterpreted but, in this case, there does not seem to be grounds for a sexual harassment suit. For additional information on this issue, see the discussion of Standard 3.02 of the Ethics Code in the chapter on ethics and professional issues in the written study materials.

a. Incorrect When the harasser is in a position of power, this increases the likelihood that a claimant will win a suit. However, the harasser’s status is not all that is taken into consideration.
b. Incorrect If Supervisor Y continues with his comments after being asked to stop, then this might be the case.
c. Incorrect The “reasonable woman” standard is used to determine if a person’s actions constitute sexual harassment but, as noted above, if the supervisor stops his behavior, it is unlikely that the employee would have grounds for a suit against him.
d. CORRECT If the harassment is not severe and the person stops when asked, it is unlikely that the act would be found to constitute sexual harassment.

The correct answer is: does not represent a sexual harassment suit as long as Supervisor Y actually stops making comments about Employee X’s appearance.

How well did you know this?
1
Not at all
2
3
4
5
Perfectly
63
Q

Dr. Tout, a psychologist, has been asked to serve as a fact witness by the plaintiff’s attorney. The plaintiff was in a car accident three months ago and is claiming that he has suffered from persistent emotional problems as a result of the accident. He has been seeing Dr. Tout in therapy for these problems for several weeks. During her testimony in court, Dr. Tout is asked by the plaintiff’s attorney whether, in her opinion, the patient’s emotional difficulties are directly related to the accident. Dr. Tout should:
Select one:

A. comply with the request as long as she also presents any information regarding possible limitations of her opinion.
B. comply with request only if her evaluation of the defendant has provided her with sufficient information for an informed opinion.
C. comply with the request only if the defendant has waived his right to confidentiality.
D. not comply with the request unless she is ordered to do so by the court.

A

The distinction between a fact witness and an expert witness is an important one. As their names imply, a fact witness testifies only to the facts, while an expert witness can offer an opinion. Note that, in this question, Dr. Tout has been asked to serve as a fact witness.

a. Incorrect See explanation for response d.
b. Incorrect See explanation for response d.
c. Incorrect See explanation for response d.
d. CORRECT This issue is addressed by T. P. Remley in Preparing for court appearances, American Counseling Association, Alexandria, VA, 1991. Remley notes that a fact witness should not render an opinion at a hearing or trial and, if asked to give an opinion, should note that he/she is not in a position to do so because of potential bias, insufficient information on which to base an opinion, etc. (A psychologist may be required to give an opinion when ordered to do so by the court. However, even in this situation, the psychologist should qualify the opinion with a statement about its potential limitations.)

The correct answer is: not comply with the request unless she is ordered to do so by the court.

How well did you know this?
1
Not at all
2
3
4
5
Perfectly
64
Q

To encourage college students to participate in her research study, Dr. LaPlace offers students free admittance to a popular four-hour workshop (“Finding a Meaningful Relationship and Keeping It”) that is offered by a colleague of hers. This policy:
Select one:

A. is unacceptable since it is coercive and, therefore, violates the provisions of the Ethics Code.
B. is unacceptable since it represents a multiple relationship and, therefore, violates the provisions of the Ethics Code.
C. is unacceptable since it represents a “conflict of interest” and, therefore, violates the provisions of the Ethics Code.
D. may be acceptable as long as Dr. LaPlace is careful to clarify any risks, obligations, and so on with the students.

A

The Ethics Code does not prohibit the use of inducements for encouraging people to participate in a research study but does place some limitations on their use.

a. Incorrect The situation described in this question cannot really be considered coercive: It would be coercive if the research was being conducted in a prison and prisoners were led to believe that they would receive special privileges if they participate in the research study.
b. Incorrect There may be cases when this would be true, but there isn’t any information in the question to assume that a multiple (dual) relationship is relevant to this situation.
c. Incorrect A conflict of interest is not relevant to this situation.
d. CORRECT Standard 8.06(b) of the Code states that in offering professional services as an inducement to potential research participants, “psychologists clarify the nature of the services, as well as the risks, obligations, and limitations.

The correct answer is: may be acceptable as long as Dr. LaPlace is careful to clarify any risks, obligations, and so on with the students.

How well did you know this?
1
Not at all
2
3
4
5
Perfectly
65
Q

You are working as a school psychologist. A teacher asks you to evaluate one of her students. After talking with the teacher and the student, you begin to suspect that the teacher is the one with the problem. As an ethical psychologist, you would:
Select one:

A. file a formal report with the School Board.
B. discuss the problem with the child’s parents immediately.
C. talk to the teacher about your suspicions.
D. report the teacher to the local ethics committee.

A

Although this problem is not dealt with directly by the Ethics Code, it does recommend an informal resolution with other psychologists whenever appropriate. Presumably, this would apply to other professionals as well.

a. Incorrect See explanation for response c.
b. Incorrect See explanation for response c.
c. CORRECT This is the most “conservative” of the responses given and most consistent with the “spirit” of the Ethics Code, which requires psychologists to attempt to informally resolve problems first rather than immediately contacting the ethics committee or other authority.
d. Incorrect See explanation for response c.

The correct answer is: talk to the teacher about your suspicions.

How well did you know this?
1
Not at all
2
3
4
5
Perfectly
66
Q

An intern at a large mental health clinic decides to work at another agency and makes appointments to see several of her previous clients at her new place of employment. Her actions are:
Select one:

A. commendable since it makes it possible for the clients to have continuity in therapy.
B. ethical as long as adequate supervision is available at the agency.
C. unethical because she has previously worked with the clients.
D. unethical because the clients are already receiving services from the clinic.

A

Standard 10.04 of the APA’s Ethics Code states that, “in deciding whether to offer or provide services to those already receiving mental health services elsewhere, psychologists carefully consider the treatment issues and the potential client’s/patient’s welfare. Psychologists discuss these issues with the client/patient … in order to minimize the risk of confusion and conflict, consult with the other service providers when appropriate, and proceed with caution and sensitivity to the therapeutic issues.”

a. Incorrect As suggested by this Standard, interns should not take clients from a previous place of employment without considering the professional relationships involved, the therapeutic issues, and the welfare of the clients.
b. Incorrect Interns should not see clients without supervision. However, even if appropriate supervision is available, an intern should not offer his/her services to clients of a previous place of employment.
c. Incorrect This response really doesn’t make sense.
d. CORRECT This response is most in line with Standard 10.04. By going ahead and making the appointments, the intern is not taking steps to minimize the risk of confusion and conflict and is not consulting with the other service providers.

The correct answer is: unethical because the clients are already receiving services from the clinic.

How well did you know this?
1
Not at all
2
3
4
5
Perfectly
67
Q

You receive a voicemail from a well-respected licensed psychologist in your community who is currently seeing one of your former clients. She says that she has obtained the client’s consent and wants you to forward the client’s record to her. You should:
Select one:

A. forward a photocopy (not the original) of the record to her.
B. forward only information that you feel is relevant and not obsolete.
C. contact the client to obtain a release directly from him.
D. wait until you hear directly from the client before taking any action.

A

Even though the psychologist is licensed and well-respected, you would not want to breach confidentiality by forwarding client information to her without knowing specifically what the client has consented to (or if, in fact, his consent was actually obtained).

a. Incorrect See explanation for response c.
b. Incorrect See explanation for response c.
c. CORRECT This is the best option of those given. You would want to verify that the client has signed a release and discuss the release and its potential consequences with him before providing the psychologist with the requested information.
d. Incorrect See explanation for response c.

The correct answer is: contact the client to obtain a release directly from him.

How well did you know this?
1
Not at all
2
3
4
5
Perfectly
68
Q

According to the APA’s General Guidelines for Providers of Psychological Services, the term “psychological services” applies to all of the following except:
Select one:

A. administration and scoring of psychological tests.
B. teaching psychology at a university.
C. supervising delivery of psychological services.
D. conducting psychotherapy in private practice.

A

As noted in the ethics and professional issues chapter of the written study materials, teaching psychology at a university is not listed as a psychological service in the General Guidelines for Providers of Psychological Services.

a. Incorrect This activity is defined as being a psychological service.
b. CORRECT As noted above, this activity is not defined as a psychological service.
c. Incorrect This activity is considered a psychological service.
d. Incorrect This activity is considered a psychological service.

The correct answer is: teaching psychology at a university.

How well did you know this?
1
Not at all
2
3
4
5
Perfectly
69
Q
Shortly after a school psychologist begins working for a small rural school district, she finds that there are a number of children in a class for students with an Intellectual Disability who are emotionally disturbed but who appear to have an average or above average level of intelligence. The county has no class for emotionally disturbed children. The psychologist should:
Select one:

A. report the situation immediately to the psychology ethics committee in her state.
B. report the situation immediately to the teacher’s standards and practices commission in her state.
C. threaten to quit if the children are not placed in a regular classroom.
D. evaluate each student and recommend an appropriate action regardless of available options.

A

This question presents a rather complex ethical dilemma. Before responding too quickly, consider the psychologist’s responsibilities and the options that are available. For ethics questions, the most conservative course of action is often the most appropriate one.

a. Incorrect At first glance, reporting may seem appropriate. However, the psychology ethics committee does not respond to complaints about the unethical activities of teachers and schools.
b. Incorrect There is no indication that the inappropriateness of this situation is the responsibility of a particular teacher.
c. Incorrect A psychologist should always try to correct a situation before quitting. Insisting that a situation be resolved and threatening to quit is not necessarily the most ethical or useful response.
d. CORRECT The psychologist’s first intervention should be to assess each child and make appropriate recommendations. If appropriate actions are not taken by the school, the psychologist will then want to consider other interventions (e.g., talking to the school board, consulting with the ethics committee).

The correct answer is: evaluate each student and recommend an appropriate action regardless of available options.

How well did you know this?
1
Not at all
2
3
4
5
Perfectly
70
Q

A psychology intern administers tests to clients of a mental health clinic, but her supervisor scores the tests and interprets their results. When preparing the psychological report, the supervisor does not indicate that the intern administered the tests. This is:
Select one:

A. ethical as long as the client was informed that the examiner is a psychology intern and was given the supervisors name.
B. ethical since the supervisor is ultimately responsible for the accuracy of the psychological report.
C. unethical since the intern should not be administering psychological tests.
D. unethical since the intern should be listed as the examiner in the report.

A

Answer D is correct: This issue is not directly addressed in the Ethics Code or the Standards for Educational and Psychological Testing. Therefore, to answer this question, you must consider which response is most consistent with the “spirit” of these documents. This response is most consistent with the provisions of the APA guidelines. See, for example, Standard 5.01(b) of the Ethics Code, which prohibits psychologists from making misleading statements, and Standard 6.06, which requires psychologists to provide accurate information to payors and funding sources.

The correct answer is: unethical since the intern should be listed as the examiner in the report.

How well did you know this?
1
Not at all
2
3
4
5
Perfectly
71
Q

A licensed psychologist who is working in an isolated rural community finds that some of his clients have problems that are beyond his training and expertise. The psychologist should:
Select one:

A. continue seeing the clients only if there is no other psychologist available.
B. use only those interventions he feels competent to use in treating the clients.
C. obtain appropriate consultation by phone.
D. refuse to see the clients until he receives adequate training.

A

The Ethics Code requires psychologists to provide only services that are within “the boundaries of their competence.” However, it is acceptable to provide new services when certain conditions are met (Standard 2.01). Note that none of the responses to this question is optimal, so you have to pick the best one of those given.

a. Incorrect The lack of another psychologist is certainly a consideration in this situation, but, even in the absence of another psychologist, seeing the clients without getting necessary supervision or consultation would not be acceptable.
b. Incorrect This would not be acceptable if more effective treatments for the clients’ problems were available.
c. CORRECT It is perfectly acceptable for psychologists to acquire new skills during the course of their practice as long as they obtain adequate training, supervision, or consultation. Of course, consultation might not be sufficient in certain situations, but, of the responses given, this is the best choice. Note that nothing in the Code suggests that consultation must be in-person.
d. Incorrect Not providing services at all would not be the best course of action in most situations if the clients had no other professional they could see.

The correct answer is: obtain appropriate consultation by phone.

How well did you know this?
1
Not at all
2
3
4
5
Perfectly
72
Q

The court offers a defendant the choice of jail or an in-patient alcohol treatment program after he is found guilty of driving while intoxicated. As the clinician who will be working with the man at the treatment center, you should keep in mind that:
Select one:

A. a waiver of confidentiality is not required because treatment has been court-ordered.
B. a waiver of confidentiality is not required because the man gave up his right to confidentiality by committing the crime.
C. a waiver of confidentiality is not required because the man gave up his right to confidentiality by choosing the treatment program instead of jail.
D. a waiver of confidentiality is required and should be obtained before you release any information about the man’s treatment to the court.

A

Even though the treatment is court-ordered, a waiver of confidentiality is required before any information about the man’s treatment is released to the court.

a. Incorrect See explanation for response d.
b. Incorrect See explanation for response d.
c. Incorrect See explanation for response d.
d. CORRECT The usual procedure is for the individual to sign a waiver of confidentiality when he/she chooses the treatment option. However, you would want to check to make sure this has occurred and, if not, you would want to obtain a signed waiver. See, e.g., L. C. Sobell and M. B. Sobell, Preserving client rights, New York: The Free Press, 1981.

The correct answer is: a waiver of confidentiality is required and should be obtained before you release any information about the man’s treatment to the court.

How well did you know this?
1
Not at all
2
3
4
5
Perfectly
73
Q

With regard to the termination of professional services, the Ethics Code requires psychologists to:
Select one:

A. offer to help the client locate alternative services.
B. take actions that protect the best interests of the client.
C. provide pretermination counseling and suggest alternative services.
D. take action that is consistent with the reason(s) for the termination.

A

Standard 10.10(c) of the Ethics Code applies in this situation. It advises that certain steps be taken, regardless of the reason for the termination.

a. Incorrect See explanation for response c.
b. Incorrect See explanation for response c.
c. CORRECT None of the responses given is wrong but this one comes closest to the actual language of the Ethics Code and, therefore, is the best answer.
d. Incorrect See explanation for response c.

The correct answer is: provide pretermination counseling and suggest alternative services.

How well did you know this?
1
Not at all
2
3
4
5
Perfectly
74
Q

A client you have been seeing for eight months says he wants to quit therapy. He feels that the original problems he came to therapy for have all been resolved. You disagree and feel that there is good reason for the man to continue seeing you. You should:
Select one:

A. get the client to agree to a few more sessions.
B. discuss his reasons for wanting to terminate.
C. discuss his reasons for wanting to terminate and your reasons for thinking he should continue.
D. let him terminate but let him know he can come back if he desires.

A

Obviously you can’t force a client to continue seeing you, but you should discuss your reservations about his termination.

a. Incorrect See explanation for response c.
b. Incorrect See explanation for response c.
c. CORRECT If you believe it would not be good for the client to quit at this time, you should discuss this with him. If he still wants to leave, then you’d want to assure him that he can come back and/or give him appropriate referrals.
d. Incorrect See explanation for response c.

The correct answer is: discuss his reasons for wanting to terminate and your reasons for thinking he should continue.

How well did you know this?
1
Not at all
2
3
4
5
Perfectly
75
Q

Dr. Bascom, a newly-licensed psychologist, has been hired by a community mental health center. Several months after beginning her job, Dr. Bascom starts treating a college student who experiences severe test anxiety. Although Dr. Bascom took a graduate course in behavior therapy and has had some clinical experience with behavioral techniques, she has never worked with anyone with test anxiety. As an ethical psychologist, Dr. Bascom should:
Select one:

A. review the literature on test anxiety before beginning treatment.
B. seek consultation from an experienced colleague and continue working with the client.
C. refer the student to another psychologist who has experience in treating test anxiety.
D. continue to see the student since she has adequate experience in behavioral techniques.

A

In a clinical setting, when a psychologist is faced with an unfamiliar situation, the question of whether to refer the client to another therapist or seek supervision arises. Standard 2.01(a) of the APA’s Ethics Code states that “Psychologists provide services, teach, and conduct research with populations and in areas only within the boundaries of their competence, based on their education, training, supervised experience, consultation, study, or professional experience.” This principle suggests that, in deciding whether to seek supervision or make a referral, psychologists must consider their training and experience.

a. Incorrect While the psychologist does need further training in treating test anxiety, a literature review undertaken on one’s own is not sufficient study or training.
b. CORRECT Seeking consultation is probably the best course of action in this situation since Dr. Bascom does have experience and training with behavioral techniques. Such a course of action is consistent with Standard 2.01(a), which states that psychologists provide services involving new techniques “based on their education, training, supervised experience, consultation, study, or professional experience.”
c. Incorrect This is not an incorrect course of action, but it is not the best choice in this situation. While acceptable, a referral is not necessary since the psychologist has the training and experience necessary to use behavioral techniques. If the psychologist did not have such training and experience, a referral would be a better course of action.
d. Incorrect Since the psychologist has not used behavioral techniques specifically to treat test anxiety before, it would be better to obtain supervision.

The correct answer is: seek consultation from an experienced colleague and continue working with the client.

How well did you know this?
1
Not at all
2
3
4
5
Perfectly
76
Q

During the first session with a client, you learn that he has a history of offenses that you find difficult to deal with and feel, therefore, that you would not like working with him. As an ethical psychologist you should:
Select one:

A. disregard your personal preferences and accept the young man into therapy.
B. refer the young man to another professional.
C. accept the young man into therapy but seek supervision.
D. tell the young man that you cannot accept him into therapy.

A

The therapeutic alliance is considered a crucial factor in therapy outcome. If a therapist believes that some characteristic of a client would interfere with the development of that alliance, it would not benefit the client to continue treatment.

a. Incorrect It is unethical for a psychologist to accept a client into therapy if the psychologist’s personal preferences may negatively affect the therapeutic process.
b. CORRECT It is not a violation of the Ethics Code to NOT accept a client into treatment when the psychologist feels that he/she would be unable to form a therapeutic relationship with the client – and, to do so, would be consistent with Ethics Code Standard 3.04 (Avoiding Harm). Therefore, the best (and most ethical) course of action in this situation is to refer the client to another professional (see Standard 2.06).
c. Incorrect Supervision may not be adequate to overcome the personal preferences of the psychologist. Therefore, answer b is a better response.
d. Incorrect Although it is acceptable to refer a client to another therapist, referrals must be handled in a professional manner. Abrupt termination is unethical.

The correct answer is: refer the young man to another professional.

How well did you know this?
1
Not at all
2
3
4
5
Perfectly
77
Q

If a psychology licensure candidate is believed to have cheated on the psychology licensing exam, he/she will:
Select one:

A. be allowed to re-take the exam but will be monitored during the process.
B. be barred from re-taking the exam for a period of no less than three years.
C. be given an opportunity to provide evidence to the licensing board why his/her test score should not be cancelled.
D. be required by the Ethics Committee to appear at a formal hearing on the matter within six months.

A

This situation is addressed in the Standards for Educational and Psychological Testing (AERA, APA, & NCME, 1999).

a. Incorrect See explanation for response c.
b. Incorrect See explanation for response c.
c. CORRECT Standard 8.11 of the Standards states that, in cases of misconduct by a test taker (e.g., suspected cheating), the test taker should be notified and “given a timely opportunity to provide evidence that the score should not be cancelled or withheld.” In addition, Standard 8.13 states that, in this situation, the test taker is entitled to “some form of procedural due process protection.” In other words, the test taker must be given an opportunity to respond to the charge and be provided with information about the procedures that will be followed before any action against him/her is taken.
d. Incorrect See explanation for response c.

The correct answer is: be given an opportunity to provide evidence to the licensing board why his/her test score should not be cancelled.

How well did you know this?
1
Not at all
2
3
4
5
Perfectly
78
Q

For a claim of malpractice:
Select one:

A. the therapist must have had malevolent intentions and there must be a causal connection between the therapist’s acts and the harm suffered by the client.
B. the therapist must have had malevolent intentions and/or there must be some harm to the client as a result of the therapist’s acts.
C. the therapist may or may not have had malevolent intentions but there must be a causal connection between the therapist’s acts and the harm suffered by the client.
D. the therapist must have violated normal “standards of practice” or must have had malevolent intentions.

A

A complaint of malpractice requires that (1) the psychologist had a professional relationship with the client that established a legal duty of care; (2) there must be a demonstrable standard of care that the psychologist breached; (3) the client must have suffered harm or injury; and (4) the psychologist’s breach of duty must be the proximate cause of that harm or injury.

a. Incorrect See explanation for response c.
b. Incorrect See explanation for response c.
c. CORRECT The psychologist’s actions do not have to be malevolent but there must be evidence that the psychologist’s actions are the cause of the harm suffered by the client.
d. Incorrect See explanation for response c.

The correct answer is: the therapist may or may not have had malevolent intentions but there must be a causal connection between the therapist’s acts and the harm suffered by the client.

How well did you know this?
1
Not at all
2
3
4
5
Perfectly
79
Q

You agree to see an adolescent in therapy. Before you begin therapy, the parents tell you they will want information about how their son is progressing. As an ethical psychologist, you should:
Select one:

A. tell the boy you’ll have to tell his parents everything.
B. tell the boy’s parents that you can’t tell them anything.
C. agree in advance with all parties what, if anything, will be revealed to the parents.
D. proceed with caution.

A

Ethically, a psychologist has the obligation to protect the confidentiality of his/her client, but legally, in most situations, parents have the right to information obtained in therapy about their minor child. This dilemma is acknowledged by the Ethics Code, which requires that psychologists consider the preferences and best interests of individuals who are legally incapable of giving informed consent.

a. Incorrect In most circumstances, this course of action will not be consistent with the adolescent’s best interest.
b. Incorrect If you do this, you will be acting in conflict with the law. Legally, in most situations, parents have the right to information obtained in therapy by a psychologist about their minor children. Ethically, however, in order to safeguard the child’s interests, psychologists should try to encourage parents to respect confidentiality.
c. CORRECT This course of action best meets the psychologist’s ethical and legal obligations. An advance agreement safeguards the interests of the adolescent, who would be made aware of the types of information which may not be kept confidential.
d. Incorrect Certainly, you should proceed with caution in this situation, as you should in any situation in which you are faced with a possible ethical dilemma. However, this is a very general response to a situation that is more specific. When answering these types of questions, you should choose responses with a level of specificity that matches the question.

The correct answer is: agree in advance with all parties what, if anything, will be revealed to the parents.

How well did you know this?
1
Not at all
2
3
4
5
Perfectly
80
Q

A client you have been seeing in therapy for three months tells you that her former therapist made repeated sexual advances toward her. She tells you that she does not want you to say anything about it to anyone and that she just wants to “get over it.” As an ethical psychologist, you should:
Select one:

A. inform the client that you are ethically obligated to make a report to the state licensing board.
B. convince the client that she should make a report to the appropriate authorities.
C. tell the client that you will maintain confidentiality.
D. tell the client that you must file a complaint with APA but that you will not reveal her name.

A

It is important to keep in mind that client confidentiality nearly always takes priority over other concerns.

a. Incorrect In this situation, you are not required to make a report since the client has not waived confidentiality.
b. Incorrect “Convincing” the client would not be the best course of action.
c. CORRECT You would not want to make a report unless you had permission from the client.
d. Incorrect Although you could file a complaint without naming the client, the Ethics Committee would be unable to take action in most cases.

The correct answer is: tell the client that you will maintain confidentiality.

How well did you know this?
1
Not at all
2
3
4
5
Perfectly
81
Q

A psychologist working for the criminal justice system is asked by the court to evaluate a young man accused of committing a felony. She is told by an attorney who has attempted to interview the man that he has difficulty attending to general directives and that his perceptions of time, place, and people appear to be confused. The psychologist will be required to testify in court regarding her assessment of the young man. In terms of confidentiality, the psychologist is:
Select one:

A. legally bound to maintain confidentiality unless she obtains a written release from the young mans attorney.
B. legally bound to maintain confidentiality unless she obtains a written release from the young man.
C. not legally bound to maintain confidentiality but ethically required to inform the young man of the limits of confidentiality.
D. not legally bound to maintain confidentiality or ethically required to discuss the limits of confidentiality due to the nature of the case.

A

Standard 4.02(a) of the Ethics Code mandates psychologists to discuss “the relevant limits of confidentiality.” Note that, in this case, the psychologist has been asked by the court to evaluate the defendant.

a. Incorrect A psychologist who is asked by the court to evaluate a defendant is not required legally or ethically to obtain the attorney’s permission to conduct the evaluation.
b. Incorrect When the court requests a psychologist to conduct an evaluation, the psychologist is not required to obtain a written release from the individual being evaluated.
c. CORRECT When working within the criminal justice system, a psychologist is serving both the offender and the criminal justice system. In such situations, a psychologist is not legally bound to maintain confidentiality; however, in accordance with Standard 4.02, the psychologist should inform the offender of the limits of confidentiality.
d. Incorrect To be consistent with the requirements of the Ethics Code, psychologists should always discuss the limits of confidentiality.

The correct answer is: not legally bound to maintain confidentiality but ethically required to inform the young man of the limits of confidentiality.

How well did you know this?
1
Not at all
2
3
4
5
Perfectly
82
Q

If a health care provider is faced with a conflict between state law and the HIPAA requirements:
Select one:

A. state law would preempt the HIPAA requirements.
B. the health care provider should respond based on the best interests of the client.
C. the HIPAA requirement would preempt state law when it provides the client with greater privacy protection or control over access to his/her records.
D. the health care provider has the discretion to resolve the issue in any appropriate manner.

A

The HIPAA Privacy Rule provides a Federal floor of privacy protections for individuals’ individually identifiable health information where that information is held by a covered entity or by a business associate of the covered entity. State laws that are contrary to the Privacy Rule are preempted by the Federal requirements, unless a specific exception applies. These exceptions include if the State law (1) relates to the privacy of individually identifiable health information and provides greater privacy protections or privacy rights with respect to such information, (2) provides for the reporting of disease or injury, child abuse, birth, or death, or for public health surveillance, investigation, or intervention, or (3) requires certain health plan reporting, such as for management or financial audits. In these circumstances, a covered entity is not required to comply with a contrary provision of the Privacy Rule [See California Office Of HIPAA Implementation’s Frequently Asked Questions at http://www.ohi.ca.gov/calohi/#].

a. Incorrect See explanation above.
b. Incorrect See explanation above.
c. CORRECT See explanation above.
d. Incorrect See explanation above.

The correct answer is: the HIPAA requirement would preempt state law when it provides the client with greater privacy protection or control over access to his/her records.

How well did you know this?
1
Not at all
2
3
4
5
Perfectly
83
Q

The function of the state licensing boards is probably best described as:
Select one:

A. ensuring that psychologists provide clients with effective services.
B. ensuring that only competent psychologists become and remain licensed.
C. setting entry-level qualifications for licensure and monitoring the conduct of licensed psychologists.
D. defining the services that can be legitimately provided by licensed psychologists.

A

As you are likely aware, the state licensing boards set the minimum requirements for licensure.

a. Incorrect Although this may be a desired outcome of the licensing requirements, it can’t be assumed that licensure will result in the provision of effective services.
b. Incorrect See explanations for responses a and c.
c. CORRECT This is the most accurate description of the function of the licensing boards.
d. Incorrect See explanation for response c.

The correct answer is: setting entry-level qualifications for licensure and monitoring the conduct of licensed psychologists.

How well did you know this?
1
Not at all
2
3
4
5
Perfectly
84
Q

Dr. Blue, a psychologist, is hired by a company to administer tests for the purpose of evaluating current employees to determine if they should be considered for promotion. In this situation:
Select one:

A. an informed consent from examinees should be obtained.
B. an informed consent is not required since the employer is Dr. Blue’s “client.”
C. an informed consent from examinees is not required, although they should be reminded about the limits of confidentiality.
D. it is up to Dr. Blue to decide whether any information about the test should be given to examinees.

A

This issue is addressed in APA’s Statement on the Disclosure of Test Data and in Standards for Educational and Psychological Testing.

a. CORRECT In most situations, psychologists obtain informed consents prior to testing from the person being evaluated or his/her legal guardian. Although there are exceptions in certain employment situations, this response is the best of those given.
b. Incorrect While it is not always necessary to get an informed consent in employment situations, this response is not as good as response a because the fact that the employer is the client does not automatically eliminate the need for an informed consent. (And, even when it isn’t necessary to obtain a consent, the individual should ordinarily be given some information about the purpose of the test, etc.)
c. Incorrect See explanation for response a.
d. Incorrect See explanation for response a.

The correct answer is: an informed consent from examinees should be obtained.

How well did you know this?
1
Not at all
2
3
4
5
Perfectly
85
Q

A client says she is upset because her doctor has suggested she undergo tests for a possible hyperactive thyroid. She says she believes that doctors are always looking for excuses to perform unnecessary procedures in order to make more money. You should:
Select one:

A. read up on hyperthyroidism so that you can give her an informed opinion.
B. have her sign a release of information and contact her physician.
C. refer her to an endocrinologist to discuss the purpose of the tests.
D. explore the source of her hostility toward doctors.

A

This woman needs to find out if tests for hyperthyroidism are necessary. A psychologist cannot ethically render a professional opinion on this matter.

a. Incorrect No matter how much you read up on hyperthyroidism, you will not, as a psychologist, be qualified to offer a professional opinion. Thus, this course of action would be a direct violation of the above-cited principle.
b. Incorrect While this is not an unethical or incorrect course of action, it does not address the woman’s needs as well as answer c. The woman needs to know if the physician’s recommendation was warranted; the physician’s own opinion on this matter is not likely to be objective.
c. CORRECT Of the choices offered, a referral to an endocrinologist most directly meets the woman’s needs and addresses the issue of the psychologist’s competence, which is the ethical issue that underlies this question. An endocrinologist is qualified to discuss the tests with the woman.
d. Incorrect There is no indication in this question that the woman has general feelings of hostility toward doctors.

The correct answer is: refer her to an endocrinologist to discuss the purpose of the tests.

How well did you know this?
1
Not at all
2
3
4
5
Perfectly
86
Q

Brousseau and Driver’s (1994) notion of “career concept” refers to an individual’s:
Select one:

A. work-related values.
B. career-related identity.
C. work-related personality characteristics.
D. career decisions.

A

Career concept is a key concept in Brousseau and Driver’s model of career development.

a. Incorrect See explanation for response d.
b. Incorrect See explanation for response d.
c. Incorrect See explanation for response d.
d. CORRECT Career concept refers to an individual’s career decisions, which vary along three dimensions: frequency of job change; direction of change; and type of change in job content. Brousseau and Driver distinguish between four career concepts - steady state, linear, spiral, and transitory. See, e.g., K. R. Brousseau and M. J. Driver, Enhancing informed choice: A career-concepts approach to career advisement, Selections, Spring, 24-31, 1994.

The correct answer is: career decisions.

How well did you know this?
1
Not at all
2
3
4
5
Perfectly
87
Q

According to Donald Super, “career maturity” refers to:
Select one:

A. the final stage of career development.
B. the ability to accomplish the tasks of each stage of career development.
C. the ability to make realistic career choices.
D. the acquisition of skills needed to successfully perform one’s chosen job.

A

Super proposes that career development involves several stages.

a. Incorrect See explanation for response b.
b. CORRECT Career maturity refers to successful accomplishment of the tasks of each stage of career development.
c. Incorrect See explanation for response b.
d. Incorrect See explanation for response b.

The correct answer is: the ability to accomplish the tasks of each stage of career development.

How well did you know this?
1
Not at all
2
3
4
5
Perfectly
88
Q

Social loafing is most likely to occur when members of a team are less productive than individual members would have been working alone because:
Select one:

A. they believe their lack of effort will not be recognized by others.
B. they experience a diminished sense of self-awareness while in the presence of others.
C. the mere presence of others produces a level of arousal that interferes with productivity.
D. they experience an enhancement of pre-existing tendencies.

A

Social loafing is one of the possible negative outcomes of group participation.

a. CORRECT Social loafing is the tendency of people to exert less effort when working as a member of a group than they would exert when working alone. It has been attributed to the belief that one’s effort (or lack of effort) will not be recognized or that group outcomes do not depend on one’s effort.
b. Incorrect See explanation for response a.
c. Incorrect See explanation for response a.
d. Incorrect See explanation for response a.

The correct answer is: they believe their lack of effort will not be recognized by others.

How well did you know this?
1
Not at all
2
3
4
5
Perfectly
89
Q

Which of the following is specifically excluded as a “disability” in the Americans with Disabilities Act?
Select one:

A. HIV/AIDS
B. learning disabilities
C. transvestism
D. diabetes

A

Disability is defined in the Americans with Disabilities Act (ADA) as “a physical or mental impairment that substantially limits one or more of the major life activities” of an individual.

a. Incorrect HIV/AIDS is included as a disability.
b. Incorrect Dyslexia and other learning disabilities are included in the definition of disability.
c. CORRECT The ADA lists several conditions that do NOT qualify as disabilities. These include transvestism, exhibitionism, gender identity disorder not resulting from a physical impairment, compulsive gambling, kleptomania, and use of illegal drugs.
d. Incorrect Diabetes is included as a disability.

The correct answer is: transvestism

How well did you know this?
1
Not at all
2
3
4
5
Perfectly
90
Q

A supervisor administers a test on the material covered in a training program to employees who have just completed the program in order to assess its effectiveness. Six months later, the same supervisor rates the employees on their job performance and correlates the two sets of scores. She obtains a correlation coefficient of .65. Most likely, the magnitude of the coefficient has been biased by which of the following?
Select one:

A. shrinkage
B. criterion contamination
C. a contrast effect
D. demand characteristics

A

In the situation described in this question, the same person is rating employees on the predictor and criterion.

a. Incorrect Shrinkage is associated with cross-validation.
b. CORRECT When knowledge of a person’s performance on the predictor affects how he/she is rated on the criterion, criterion contamination has occurred and is likely to inflate the criterion-related validity coefficient. A coefficient of .65 is relatively high and may reflect criterion contamination.
c. Incorrect The contrast effect occurs when the ability of previous interviewees or applicants affects how a subsequent interviewee or applicant is evaluated.
d. Incorrect Demand characteristics are cues in the environment that inform research participants what behaviors are expected from them.

The correct answer is: criterion contamination

How well did you know this?
1
Not at all
2
3
4
5
Perfectly
91
Q

According to Super’s lifespan model of career development, a major task of the ________ stage is to update one’s skills through training.
Select one:

A. growth
B. disengagement
C. exploration
D. maintenance

A

Super’s (1980) model of career development distinguishes between five stages: growth (ages 4-13), exploration (14-24), establishment (ages 25-44), maintenance (ages 45-65), and disengagement (ages 65+).

a. Incorrect A primary task for the initial growth stage is to develop an understanding of the meaning and purpose of work.
b. Incorrect A primary task of the final disengagement phase is to gradually separate from paid employment.
c. Incorrect The tasks of the exploration phase include becoming aware of one’s abilities and interests and learning about the different types of work.
d. CORRECT During the maintenance stage, the primary tasks are staying competitive with younger workers by obtaining necessary training and developing plans for retirement.

The correct answer is: maintenance

92
Q

An evaluator wants to assess the benefits received by client participation in a program. Which approach to program evaluation should they use?
Select one:

A. Goals-based
B. Process-based
C. Outcomes-based
D. Assistance-based

A

The correct answer is C. Outcomes-based program evaluation focuses on identifying and evaluating the benefits gained by participation in a program, and is typically used for evaluation of non-profit programs. Goals-based evaluations focus on how well the program is achieving set objectives, so answer A is incorrect. Process-based evaluations regard the effectiveness of the programs systems and structures, so answer B is incorrect. Answer D, assistance-based, is not an approach to program evaluation and thus is incorrect.

The correct answer is: Outcomes-based

93
Q

When using the survey feedback method in an organization, the purpose of the feedback is to:
Select one:

A. inform individual employees about their individual areas of strength and weakness.
B. inform employees about their progress toward meeting organizational goals.
C. provide employees with information about the organization’s strengths and weaknesses.
D. provide managers with information about employees’ strengths and weaknesses.

A

Survey feedback is an organizational development technique.

a. Incorrect See explanation for response c.
b. Incorrect See explanation for response c.
c. CORRECT The rationale underlying the use of survey feedback is that, for organizational change to be effective, employees must be aware of the organization’s current strengths and weaknesses. The technique entails collecting information about the organization (especially its problems) and providing that information to employees who then work together to identify solutions to the problems.
d. Incorrect See explanation for response c.

The correct answer is: provide employees with information about the organization’s strengths and weaknesses.

94
Q

When applying Bandura’s social learning theory to training in the workplace, you would use which of the following techniques?
Select one:

A. vestibule training
B. job rotation
C. behavioral modeling
D. miniature job training

A

Knowing that Bandura’s research linked learning to observation would have enabled you to select the correct answer to this question.

a. Incorrect Vestibule training involves the use of a simulation of the actual work environment.
b. Incorrect Job rotation involves “rotating” through several different jobs.
c. CORRECT Bandura’s social learning theory suggests that behavioral modeling would be an effective job training technique.
d. Incorrect Miniature job training involves providing a job applicant with some training to determine if he/she will benefit from it.

The correct answer is: behavioral modeling

95
Q

In organizations, quality circles (QCs):
Select one:

A. consist of workers who make work-related decisions that were previously made by management.
B. consist of workers who make suggestions to management about solutions to work-related problems.
C. include a designated internal change agent who is responsible for initiating, guiding, and evaluating a change effort.
D. include an external change agent who helps employees adapt to change.

A

QCs consist of a small team of employees who work together on a particular job and meet voluntarily to discuss solutions to job productivity problems.

a. Incorrect See explanation for response b.
b. CORRECT A feature that distinguishes QCs from other types of work teams is that members of QCs do not make final decisions about solutions to problems but, instead, recommend their solutions to management.
c. Incorrect See explanation for response b.
d. Incorrect See explanation for response b.

The correct answer is: consist of workers who make suggestions to management about solutions to work-related problems.

96
Q

The “risky shift phenomenon” is a potential problem when:
Select one:

A. employees are low in both motivation and ability.
B. employees work together to derive a work-related decision.
C. a subjective rating scale is used to evaluate employee performance.
D. a controversial message is delivered by a highly credible communicator.

A

The research has shown that group decision-making is associated with a number of potential liabilities including the risky shift phenomenon.

a. Incorrect See explanation for response b.
b. CORRECT The risky shift is the tendency for decisions made by a group to be less conservative (more risky) than the decisions made by individuals working alone. For additional information on the risky shift and related phenomena, see the Industrial-Organizational Psychology chapter of the AATBS written study materials.
c. Incorrect See explanation for response b.
d. Incorrect See explanation for response b.

The correct answer is: employees work together to derive a work-related decision.

97
Q

Lawler’s (1973) model of facet satisfaction:
Select one:

A. is similar to equity theory and assumes that job satisfaction is affected by comparisons of one’s own inputs and outcomes to the inputs and outcomes of others.
B. is similar to need hierarchy theory and assumes that job satisfaction is related to the fulfillment of one’s most prepotent needs.
C. is similar to two factor theory and views satisfaction and dissatisfaction as independent factors.
D. is similar to goal setting theory and proposes that participation in the setting of work objectives is crucial for job satisfaction.

A

Lawler’s model of facet satisfaction is based on the premise that job satisfaction is a product of comparisons of one’s job inputs, the inputs of others, and the job’s characteristics to one’s outcomes and the outcomes of others.

a. CORRECT Like equity theory, Lawler’s model predicts that workers compare their own input/outcome ratios to the input/outcome ratios of comparable others. When the ratios are the same, the worker is satisfied; when the worker’s own ratio is larger than that of comparable others, the worker may feel guilt or other discomfort; and when the worker’s ratio is less than that of comparable others, he/she is dissatisfied.
b. Incorrect See explanation for response a.
c. Incorrect See explanation for response a.
d. Incorrect See explanation for response a.

The correct answer is: is similar to equity theory and assumes that job satisfaction is affected by comparisons of one’s own inputs and outcomes to the inputs and outcomes of others.

98
Q

Which of the following individuals has proposed that organizational culture acts, at least in part, as a defense mechanism that helps organizational members avoid uncertainty and anxiety?
Select one:

A. Herbert Simon
B. Edgar Schein
C. Leon Festinger
D. Geert Hofstede

A

Knowing that Edgar Schein has written extensively on organizational culture would have helped you choose the correct answer to this question.

a. Incorrect See explanation for response b.
b. CORRECT In his book Organizational Culture and Leadership (1992), Schein describes the basic assumptions that underlie an organization’s culture as cognitive defense mechanisms. Additional information about Schein’s work on organizational culture is provided in the Industrial-Organizational Psychology chapter of the written study materails.
c. Incorrect See explanation for response b.
d. Incorrect See explanation for response b.

The correct answer is: Edgar Schein

99
Q

In terms of incremental validity, which of the following situations most supports the use of a new predictor?
Select one:

A. moderate base rate with many applicants and few job openings
B. low base rate with many applicants and few job openings
C. moderate base rate with few applicants and many job openings
D. low base rate with few applicants and many job openings

A

The degree to which a new selection technique will increase decision-making accuracy depends on several factors including the base rate (proportion of correct decisions without the new technique) and the selection ratio (ratio of applicants to job openings).

a. CORRECT A moderate base rate suggests that there’s room for improvement and, therefore, that a new predictor is likely to increase decision-making accuracy. Also, the situation is optimal when there are many applicants to choose from (a low selection ratio).
b. Incorrect A low base rate suggests that something else besides selection is the problem - e.g., the company’s standards of success are too high or employees need training in order to do the job satisfactorily.
c. Incorrect See explanation above.
d. Incorrect See explanation above.

The correct answer is: moderate base rate with many applicants and few job openings

100
Q

Vestibule training would be most useful:
Select one:

A. for teaching basic skills to unskilled workers.
B. when on-the-job training would be too dangerous.
C. when training costs must be kept to a minimum.
D. for teaching complex cognitive skills.

A

Vestibule training is a type of off-the-job training that makes use of a physical replication or simulation of the actual job environment.

a. Incorrect See explanation for response b.
b. CORRECT A vestibule is especially useful when on-the-job training would be too dangerous.
c. Incorrect Setting up a vestibule can be very costly, so the need to minimize costs would not be an indicator for this type of training.
d. Incorrect See explanation for response b.

The correct answer is: when on-the-job training would be too dangerous.

101
Q

In his theory of career development, Donald Super uses which of the following to pictorially depict the various roles that a person participates in during the course of his/her life?
Select one:

A. rainbow
B. arch
C. ladder
D. web

A

Super uses several figures to illustrate and integrate aspects of career development.

a. CORRECT Super’s Life Career Rainbow illustrates the various roles people play (e.g., student, parent, worker) and how these roles correspond to a person’s life stages.
b. Incorrect Super’s arch illustrates the personal and environmental factors that determine a person’s career path.
c. Incorrect The ladder depicts the stages in Super’s career development model.
d. Incorrect The web illustrates the bases of career maturity.

The correct answer is: rainbow

102
Q
Kurt Lewin (1951) described the process of organizational change as involving three stages. These stages are:
Select one:

A. plan, act, and evaluate.
B. unfreezing, changing, and refreezing.
C. entry, planning, and action.
D. collecting data, providing feedback, and implementing/developing action plans.

A

Lewin’s (1951) model of organizational change was among the first to be proposed and involves three stages.

a. Incorrect These are not the stages identified by Lewin.
b. CORRECT This answer correctly identifies Lewin’s three stages.
c. Incorrect Huse (1980) subsequently modified Lewin’s model of organizational change by adding steps to the three stages. This answer identifies three of the steps (entry and planning are steps in the unfreezing stage, while action is the primary step in the changing stage).
d. Incorrect These are the three steps of survey feedback.

The correct answer is: unfreezing, changing, and refreezing.

103
Q

Transformational leaders:
Select one:

A. are likely to have a “dark side.”
B. use their legitimate status to gain compliance.
C. appeal to the self-interest of their followers.
D. seek to empower their followers.

A

Transformational leaders recognize the need for change (transformation) and are able to communicate that vision to their followers and gain their commitment to it.

a. Incorrect Some experts describe a charismatic leader as having a “dark side,” but it’s not a characteristic ordinarily associated with transformational leaders.
b. Incorrect This sounds more like transactional leaders (who are contrasted with transformational leaders).
c. Incorrect This also sounds more like a transactional leader.
d. CORRECT One characteristic of transformational leaders is that they attempt to raise the level of consciousness, awareness, and commitment of their followers and to strengthen their ability to control their own destinies.

The correct answer is: seek to empower their followers.

104
Q

Calculation of the utility of a training program in terms of dollar value includes which of the following?
Select one:

A. T x d x C
B. T - d x C
C. T x C - d
D. T + C + d

A

Utility refers to the cost effectiveness of training and can be calculated using the following formula: $U = T x N x d x SD - N x C where:
$U = utility measured in terms of dollar value
T = number of years duration of the training program’s effects on performance
N = number of people trained
d = effect size of the training program
SD = standard deviation of job performance in dollars
C = per person cost of training

a. CORRECT As shown above, T x d x C is part of the formula for calculating utility.
b. Incorrect See formula above.
c. Incorrect See formula above.
d. Incorrect See formula above.

The correct answer is: T x d x C

105
Q

The “paired comparison” technique is:
Select one:

A. a method of job analysis.
B. a method of job evaluation.
C. a relative measure of job performance.
D. an absolute measure of job performance.

A

The paired comparison technique is a performance appraisal technique.

a. Incorrect See explanation for response c.
b. Incorrect See explanation for response c.
c. CORRECT When using the paired comparison technique, all employees are compared to each other in pairs for each dimension of job performance.
d. Incorrect See explanation for response c.

The correct answer is: a relative measure of job performance.

106
Q

You would be concerned about a selection test’s differential validity for males and females when you discover that:
Select one:

A. the Y-intercept for the regression line is higher for males than for females.
B. the X-intercept for the regression line is higher for males than for females.
C. the slope of the regression lines for males and females are different.
D. females consistently obtain lower scores than males on the predictor.

A

Differential validity refers to different validity coefficients for different groups of people.

a. INCORRECT See explanation for response c.
b. INCORRECT See explanation for response c.
c. CORRECT The slope of the regression line is directly related to the magnitude of the validity coefficient. If the slopes differ for males and females, this indicates that the validity coefficients are different for males and females.
d. Incorrect See explanation for response c.

The correct answer is: the slope of the regression lines for males and females are different.

107
Q

Equity theory predicts that a husband will feel best about his marriage if:
Select one:

A. he and his wife give and receive the same amount in their relationship.
B. he perceives that he and his wife give and receive the same amount in their relationship.
C. in terms of their relationship, his give/receive ratio is the same as or greater than the give/receive ratio of his wife.
D. in terms of their relationship, he perceives that his give/receive ratio is the same as the give/receive ratio of his wife.

A

Equity theory is a cognitive theory of motivation that emphasizes the impact of the individual’s beliefs about the equity of the situation in terms of inputs and outcomes. When a person perceives the situation to be equitable, he/she will be satisfied and motivated.

a. Incorrect See explanation for response d.
b. Incorrect See explanation for response d.
c. Incorrect See explanation for response d.
d. CORRECT According to equity theory, a person compares his/her input/outcome ratio to that of others. If he/she perceives the ratios to be similar, this has positive effects on satisfaction and motivation.

The correct answer is: in terms of their relationship, he perceives that his give/receive ratio is the same as the give/receive ratio of his wife.

108
Q

Transformational leaders:
Select one:

A. appeal to the self-interest of their followers.
B. help followers recognize that their goals are compatible with the organization’s goals.
C. induce followers to transcend self-interest for the sake of the organization.
D. redefine the organization’s goals to better coincide with the self-interest of followers.

A

Transformational leaders motivate followers to accept and support organizational change that is in the best interests of the organization.

a. Incorrect This is more characteristic of transactional leaders.
b. Incorrect This is less consistent with descriptions of transformational leaders than response c.
c. CORRECT Transformational leaders motivate workers to achieve transcendental goals that go beyond each worker’s immediate self-interests.
d. Incorrect This is not true about transformational leaders.

The correct answer is: induce followers to transcend self-interest for the sake of the organization.

109
Q

An aerospace corporation uses the procedure known as multiple cutoff in its employee selection process. As a result, which of the following is most likely to occur?
Select one:

A. An applicant who obtains very high scores on tests of mathematical and mechanical intelligence will not be hired because he fails a test of verbal intelligence by several points.
B. An applicant who scores low on a test of verbal intelligence obtains very high scores on tests of mechanical and mathematical intelligence, so he is hired as an airline mechanic.
C. An applicant’s score on tests of verbal, mathematical, and mechanical intelligence place him in the lowest of fifteen criterion groups, so he is not hired.
D. An applicant’s score on a test of mechanical intelligence compares favorably to the scores of successful employees, so he is hired as an airline mechanic.

A

Multiple cutoff is a method for combining predictor scores that involves establishing minimum cut-off scores for each predictor. If an examinee falls below the cut-off for even one predictor, he or she will be rejected.

a. CORRECT The applicant in this case was not hired because he did not pass all of the tests used for prediction. As noted above, when multiple cutoff is used, all of the predictor tests must be passed.
b. Incorrect The multiple cutoff technique is noncompensatory; that is, a high score on one measure does not compensate for a low score on another measure.
c. Incorrect In this case, scores on two or more predictors are combined to predict membership in three or more criterion groups. This procedure is known as multiple discriminant analysis.
d. Incorrect In this case, a score on a test is interpreted in terms of a comparison to the scores of successful employees and is not a description of the multiple-cutoff technique.

The correct answer is: An applicant who obtains very high scores on tests of mathematical and mechanical intelligence will not be hired because he fails a test of verbal intelligence by several points.

110
Q

According to Albert Bandura, modeling (observational learning):
Select one:

A. teaches new behaviors.
B. disinhibits and facilitates behaviors already in one’s repertoire.
C. provides cues as to the appropriateness of a particular behavior.
D. all of the above.

A

Bandura’s research found that modeling is useful for the acquisition of new behaviors and for increasing or decreasing behaviors already in an individual’s repertoire.

a. Incorrect See explanation for response d.
b. Incorrect See explanation for response d.
c. Incorrect See explanation for response d.
d. CORRECT According to Bandura, modeling serves a number of functions including the ones listed in responses a, b, and c. (And, if you think about it, this makes sense: We learn a lot from observing others.)

The correct answer is: all of the above.

111
Q

Tolman’s research with rats in mazes confirmed that:
Select one:

A. noncontingent reinforcement can lead to superstitious behavior.
B. reinforcement is not necessary for learning to occur.
C. observational learning occurs in non-human animals.
D. learning is constrained by biological predispositions.

A

For the licensing exam, you want to have Tolman associated with latent learning.

a. Incorrect The effect of noncontingent (accidental) reinforcement on behavior was identified by Skinner.
b. CORRECT Tolman’s research indicated that learning of a new behavior (but not necessarily performance of that behavior) occurs without reinforcement. Specifically, his studies of rats in mazes demonstrated that rats formed cognitive maps of the maze even without reinforcement.
c. Incorrect Observational learning was not part of Tolman’s research.
d. Incorrect Biological predispositions were not addressed by Tolman’s research.

The correct answer is: reinforcement is not necessary for learning to occur.

112
Q

For the treatment of migraine headaches:
Select one:

A. autogenic training is more effective than thermal biofeedback.
B. thermal biofeedback is more effective than autogenic training.
C. thermal biofeedback plus autogenic training is more effective than either treatment alone.
D. thermal biofeedback plus autogenic training is no more effective than either treatment alone.

A

For the exam, you want to be familiar with the research on the use of biofeedback for the disorders addressed in the Learning Theory chapter of the written study materials.

a. Incorrect See explanation for response c.
b. Incorrect See explanation for response c.
c. CORRECT Several studies have found that this combination of treatments is the best approach for migraine headaches.
d. Incorrect See explanation for response c.

The correct answer is: thermal biofeedback plus autogenic training is more effective than either treatment alone.

113
Q

Secondary reinforcers are:
Select one:

A. stimuli that increase a behavior through their accidental pairing with that behavior.
B. stimuli that gain their reinforcing value through their association with an unconditioned reinforcer.
C. high-frequency behaviors that are used to reinforce low-frequency behaviors.
D. reinforcers that are removed (rather than applied) following a behavior.

A

Reinforcers are classified as primary or secondary.

a. Incorrect This answer does not describe secondary reinforcers.
b. CORRECT Primary (unconditioned) reinforcers are inherently reinforcing, while secondary (conditioned) reinforcers acquire their reinforcing value through their association with one or more primary reinforcers. Money is an example of a secondary reinforcer.
c. Incorrect When using the Premack Principle, a high-frequency behavior is used as a reinforcer for a low-frequency behavior to increase the low-frequency behavior.
d. Incorrect This answer describes negative reinforcement.

The correct answer is: stimuli that gain their reinforcing value through their association with an unconditioned reinforcer.

114
Q

A first-grade student frequently misbehaves in class. Whenever he does so, his teacher has him stand in the corner for ten minutes. During that time, he often turns around and makes faces at his classmates, and they respond by laughing and talking to him. The boy’s misbehavior is being maintained by which of the following?
Select one:

A. habituation
B. satiation
C. positive reinforcement
D. negative reinforcement

A

In this situation, the boy’s misbehavior is continuing because of the attention he receives from his classmates when he misbehaves.

a. Incorrect Habituation occurs when a punishment loses its effectiveness.
b. Incorrect Satiation occurs when a reinforcer loses its effectiveness.
c. CORRECT Positive reinforcement occurs when a behavior increases because of a stimulus that is applied following the behavior. In this situation, the boy’s misbehavior is being maintained by the attention of his peers.
d. Incorrect Negative reinforcement occurs when a behavior increases because of a stimulus that is removed following the behavior.

The correct answer is: positive reinforcement

115
Q

When using the technique known as “covert sensitization,” the client:
Select one:

A. is consistently reinforced for engaging in behaviors other than the target (undesirable) behavior during a prespecified period of time.
B. imagines a person other than him/herself engaging in variety of alternative (desirable) behaviors.
C. repeatedly engages in the target (undesirable) behavior while reinforcement is consistently withheld.
D. imagines he/she is engaging in the target (undesirable) behavior and then imagines an aversive consequence for doing so.

A

Covert sensitization is a type of aversion therapy that uses imagery.

a. Incorrect This procedure describes the technique known as overcorrection.
b. Incorrect This answer describes covert modeling.
c. Incorrect This answer describes operant extinction.
d. CORRECT Covert sensitization uses counterconditioning in imagination (versus in vivo) in order to reduce or eliminate an undesirable behavior.

The correct answer is: imagines he/she is engaging in the target (undesirable) behavior and then imagines an aversive consequence for doing so.

116
Q

Procedural memory is to declarative memory as:
Select one:

A. echoic is to iconic.
B. fact is to experience.
C. explicit is to implicit.
D. skill is to fact.

A

Long-term memory can be conceptualized in terms of two basic types or categories - procedural and declarative.

a. Incorrect See explanation for response d.
b. Incorrect See explanation for response d.
c. Incorrect See explanation for response d.
d. CORRECT As its name implies, procedural memory contains memories for procedures - that is, perceptual, cognitive, and motor skills. Declarative memory (which is subdivided into semantic and episodic memory) contains memories for facts and events.

The correct answer is: skill is to fact.

117
Q

For Kohler and other Gestalt psychologists, learning is the result of which of the following?
Select one:

A. the development of mental models
B. the formation of associations between stimuli and responses
C. the perception of relationships among elements of the problem
D. observation and imitation of the behavior of others

A

Kohler was one of the founders of Gestalt psychology, which is based on the assumption that “the whole is greater than the sum of its parts.” He is probably best known for his work on insight.

a. Incorrect See explanation for response c.
b. Incorrect See explanation for response c.
c. CORRECT Kohler described insight as a type of relational thinking - i.e., the sudden understanding of relationships among elements of the problem situation.
d. Incorrect See explanation for response c.

The correct answer is: the perception of relationships among elements of the problem

118
Q

As defined by Beck, “schemas” are:
Select one:

A. cognitive structures.
B. logical errors.
C. innate predispositions.
D. automatic interpretations.

A

For Beck and others, schemas are cognitive structures that represent knowledge about events, objects, or other phenomena. Schemas structure the way we think about the stimuli we encounter.

a. CORRECT Beck believes that cognitive schemas are the product of biological and environmental phenomena and develop early in life.
b. Incorrect Logical errors are important to Beck’s theory but are not synonymous with cognitive schemas.
c. Incorrect Cognitive schemas are predispositions and do have a biological component, but this is not the best response.
d. Incorrect This sounds more like automatic thoughts, which are also part of Beck’s theory but are not synonymous with cognitive schemas.

The correct answer is: cognitive structures.

119
Q

The gradual decline in the intensity, frequency, or duration of a response to the repeated presentation of the same punishment or other stimulus is referred to as:
Select one:

A. satiation.
B. inhibition.
C. fading.
D. habituation.

A

This question describes a situation in which an individual is no longer responding to a punishment or other stimulus because of repeated exposure to it.

a. Incorrect See explanation for response d.
b. Incorrect See explanation for response d.
c. Incorrect See explanation for response d.
d. CORRECT This is the definition of habituation.

The correct answer is: habituation.

120
Q

Self-monitoring is used in cognitive-behavioral therapies:
Select one:

A. to facilitate diagnosis.
B. as both an assessment tool and an intervention.
C. to foster feelings of self-efficacy.
D. to explore the underlying meaning of events.

A

Self-monitoring is what it sounds like – in involves monitoring one’s own behavior.

a. Incorrect See explanation for response b.
b. CORRECT Self-monitoring has been found useful not only as an assessment technique but also as an intervention because it tends to alter the target behavior in the desired direction.
c. Incorrect See explanation for response b.
d. Incorrect See explanation for response b.

The correct answer is: as both an assessment tool and an intervention.

121
Q

A psychologist investigating short-term memory shows the letter “T” to each research subject for a brief period and, after removing the letter, asks the subject what he or she has seen. The psychologist finds that many subjects report seeing the wrong letter, with the most common error of recall being which of the following letters?
Select one:

A. L
B. D
C. S
D. F

A

This is a difficult question that requires you to know that information in short-term memory is believed to be primarily coded acoustically.

a. Incorrect See explanation for response b.
b. CORRECT Studies similar to the one described in this question have found that errors in short-term memory usually involve a confusion of letters that sound alike, thereby supporting the theory that information is stored acoustically in short-term memory. Of the letters listed in the responses, only “D” sounds like “T”.
c. Incorrect See explanation for response b.
d. Incorrect See explanation for response b.

The correct answer is: D

122
Q

Implicit memory is involved in:
Select one:

A. remembering where you were on your last birthday.
B. buttoning your shirt when you get dressed in the morning.
C. solving a simple arithmetic problem “in your head.”
D. studying for the licensing exam.

A

Implicit memory is memory that does not require conscious effort.

a. Incorrect See explanation for response b.
b. CORRECT Of the memories listed in the responses, buttoning a shirt is most likely to involve implicit memory. (In fact, most procedural memories are implicit and some investigators use the terms interchangeably.)
c. Incorrect See explanation for response b.
d. Incorrect See explanation for response b.

The correct answer is: buttoning your shirt when you get dressed in the morning.

123
Q

As defined by Aaron Beck, automatic thoughts are:
Select one:

A. more accessible and less stable than voluntary thoughts.
B. spontaneously triggered by a specific circumstance and accompanied by an emotional reaction.
C. systematic errors in reasoning that are evident during times of psychological distress.
D. always irrational.

A

Beck distinguishes between a number of cognitive phenomena including voluntary thoughts, automatic thoughts, cognitive schemas, and cognitive distortions.

a. Incorrect Automatic thoughts are less accessible and more stable than voluntary thoughts.
b. CORRECT This response is consistent with Beck’s definition of automatic thoughts.
c. Incorrect This describes cognitive distortions.
d. Incorrect Automatic thoughts may be rational or irrational in the sense that they may be accurate or inaccurate. Treatment focuses on irrational (inaccurate) automatic thoughts.

The correct answer is: spontaneously triggered by a specific circumstance and accompanied by an emotional reaction.

124
Q

Which of the following is best conceptualized as a type of aversive counterconditioning?
Select one:

A. implosive therapy
B. overcorrection
C. response cost
D. covert sensitization

A

Aversive counterconditioning is used to eliminate an undesirable behavior that produces a pleasant or positive response (e.g., sexual arousal) by pairing stimuli associated with that behavior with stimuli that produce an unpleasant response. By doing so, the pleasant response is replaced by the unpleasant response and the behavior is reduced or eliminated.

a. Incorrect Implosive therapy is used to eliminate anxiety and involves exposing the individual to the anxiety-arousing stimulus in imagination for an extended period of time.
b. Incorrect Overcorrection is usually classified as a type of punishment and involves having the individual engage in certain behaviors as a penalty for having performed the target behavior - e.g., “fixing” the consequences of the target behavior and/or engaging in alternative desirable behaviors.
c. Incorrect Response cost is a type of negative punishment that involves removing a stimulus (e.g., a privilege) following a behavior whenever it occurs in order to reduce the frequency of that behavior.
d. CORRECT When using covert sensitization to eliminate an undesirable behavior, the individual imagines an unpleasant (aversive) consequence while imagining that he/she is engaging in the behavior.

The correct answer is: covert sensitization

125
Q

A researcher conducting a study to investigate the phenomenon known as “stimulus generalization” would most likely do which of the following after establishing a conditioned response in her participants?
Select one:

A. Present new stimuli that are similar in varying degrees to the CS.
B. Repeatedly present the CS after a neutral stimulus.
C. Repeatedly present the CS without the US.
D. Pair presentation of the CS with a second US.

A

Stimulus generalization occurs when stimuli similar to the CS elicit a response that is similar to the CR.

a. CORRECT This procedure would be used to determine the extent of stimulus generalization following classical conditioning.
b. Incorrect This answer describes higher-order conditioning.
c. Incorrect This answer describes extinction.
d. Incorrect The procedure described in this answer would not be useful for investigating stimulus generalization.

The correct answer is: Present new stimuli that are similar in varying degrees to the CS.

126
Q

The best way to keep two sets of six digits in working memory is to:
Select one:

A. visualize the digits in a string.
B. convert the 12 digits to “chunks.”
C. connect each digit with a visual image.
D. repeat the digits first forward and then backward several times.

A

Working memory is another name for short-term memory. Two processes are useful for maintaining information in short-term memory: chunking and rehearsal (especially elaborative rehearsal).

a. Incorrect See explanation for response b.
b. CORRECT The number of digits exceeds the number of units that can be held in short-term memory. Therefore, it would be necessary to “chunk” the digits into a smaller number of units; e.g., four chunks of three digits each.
c. Incorrect This might be useful after the digits have been chunked to transfer information from short-term to long-term memory.
d. Incorrect See explanation for response b.

The correct answer is: convert the 12 digits to “chunks.”

127
Q

When using in vivo exposure with response prevention:
Select one:

A. the CS is presented repeatedly without the US.
B. the US is presented repeatedly without the CS.
C. the CS and US are alternately presented.
D. the CS and US are presented simultaneously.

A

In vivo exposure with response prevention (also known as flooding) involves exposing an individual to an anxiety-arousing stimulus while preventing the person from making his/her usual avoidance response.

a. CORRECT The assumption underlying the use of in vivo exposure as a treatment for anxiety is that anxiety is a classically-conditioned response in which the anxiety-arousing stimulus (the conditioned stimulus) attained its anxiety-arousing properties by being paired in the past with a stimulus that naturally produced anxiety (the unconditioned stimulus). In vivo exposure involves extinguishing the classically conditioned anxiety response by repeatedly exposing the person to the conditioned stimulus (CS) without the unconditioned stimulus (US).
b. Incorrect See explanation above.
c. Incorrect See explanation above.
d. Incorrect See explanation above.

The correct answer is: the CS is presented repeatedly without the US.

128
Q

Baddeley and Hitch’s (1974) multi-component model of working memory includes all of the following components except:
Select one:

A. a phonological loop.
B. a visuo-spatial sketchpad.
C. a central executive.
D. a semantic-abstract processor.

A

Working memory is the aspect of short-term memory that is involved in the manipulation and storage of information. Baddeley and Hitch’s (1974) multi-component model of working memory describes it as consisting of three components.

a. Incorrect The phonological loop is one of the components of working memory identified by Baddeley and Hitch. Its role is to rehearse verbal information in order to retain it in short-term memory.
b. Incorrect The visuo-spatial sketchpad is also one of the components of working memory and is involved in the processing and short-term storage of visuospatial information.
c. Incorrect As described by Baddeley and Hitch, the central executive component of working memory directs the activities of the other two components and controls the transmission of information to other parts of the cognitive system.
d. CORRECT A semantic-abstract processor was not identified by Baddeley and Hitch as a component of working memory.

The correct answer is: a semantic-abstract processor.

129
Q

Dr. Agape uses a dismantling strategy to identify the “active ingredient” of systematic desensitization. He is most likely to find that which of the following is the critical component?
Select one:

A. stimulus control
B. reciprocal inhibition
C. extinction
D. behavior rehearsal

A

Research using a dismantling strategy involves separating and comparing the various components of a treatment to identify the component(s) most responsible for the treatment’s effects.

a. Incorrect See explanation for response c.
b. Incorrect See explanation for response c.
c. CORRECT Systematic desensitization was originally developed as an application of reciprocal inhibition. However, studies using the dismantling strategy have found that its effects are due to repeated exposure to the CS without the US (i.e., to classical extinction).
d. Incorrect See explanation for response c.

The correct answer is: extinction

130
Q

The aspect of memory that is capable of storing a very large amount of information for a very brief period of time is referred to as ________ memory.
Select one:

A. short-term
B. sensory
C. primary
D. secondary

A

According to the multistore model of memory, memory has three components – sensory, short-term, and long-term. Additional information about these components is provided in the Learning Theory chapter of the written study materials.

a. Incorrect Short-term memory has a limited capacity and a limited duration.
b. CORRECT Sensory memory contains information collected from all senses but retains that information for a very brief period (probably .5 second).
c. Incorrect Some authors use the term primary memory as a name for short-term memory.
d. Incorrect The term secondary memory refers to long-term memory which has a very large capacity and a very long duration.

The correct answer is: sensory

131
Q

A psychologist determines that, for a six-year child, Behavior A is more probable than Behavior B. The psychologist tells the parents to make the child’s opportunity to engage in Behavior A contingent on Behavior B. The psychologist is using which of the following techniques?
Select one:

A. response cost
B. shaping
C. DRO
D. Premack principle

A

In this situation, the opportunity to engage in the more frequent behavior is being made contingent on performance of the less frequent behavior in order to increase the less frequent behavior.

a. Incorrect See explanation for response d.
b. Incorrect See explanation for response d.
c. Incorrect See explanation for response d.
d. CORRECT The Premack Principle is a type of positive reinforcement in which the reinforcer is a frequently occurring behavior. If you allow yourself to talk on the phone to friends (or watch TV) only after studying for at least one hour, you are using the Premack Principle.

The correct answer is: Premack principle

132
Q

Research on the effects of divorce on the parent-child relationship has generally shown that, during the initial period following divorce, the relationship between the custodial parent and his/her child often changes. Specifically, during this period, the custodial mother most often:
Select one:

A. provides harsher but more inconsistent punishment.
B. becomes overindulgent and overpermissive.
C. spends more time with her child.
D. is more concerned about the effectiveness of her parenting skills.

A

According to Wallerstein and Blakeslee (1990), the period following divorce is typically characterized by a “diminished capacity to parent.”

a. CORRECT Mothers, immediately after divorce, are often more punitive but their punishments are commonly inconsistent.
b. Incorrect This is not what has been reported in the literature about custodial mothers.
c. Incorrect Both parents tend to become more self-involved following a divorce and, therefore, typically spend less time with their children.
d. Incorrect This is not what has been found in follow-up studies of divorced families.

The correct answer is: provides harsher but more inconsistent punishment.

133
Q

Research on parenting styles suggests that parents who are _____________ are most likely to have adolescents who are high in academic achievement.
Select one:

A. high in both demandingness and warmth
B. high in demandingness but low in warmth
C. low in demandingness and warmth
D. low in demandingness but high in warmth

A

Two dimensions are often used to categorize parenting styles - demandingness/control and warmth/support.

a. CORRECT Research by Baumrind and others has confirmed that an authoritative parenting style (i.e., a style characterized by a high degree of warmth and demandingess) is most predictive of academic success in adolescence.
b. Incorrect See explanation above.
c. Incorrect See explanation above.
d. Incorrect See explanation above.

The correct answer is: high in both demandingness and warmth

134
Q

Konrad Lorenz’s (1957) research on “imprinting” in ducklings and goslings had the greatest impact on the study of ___________ in human infants.
Select one:

A. fear and aggression
B. gender differences
C. sociability
D. attachment

A

Imprinting refers to the social bonding that occurs during a critical period following birth in some non-human species.

a. Incorrect See explanation for response d.
b. Incorrect See explanation for response d.
c. Incorrect See explanation for response d.
d. CORRECT The ethologist Lorenz found that certain birds become attached (bonded) to the first moving object they encounter during the first few days following birth. This finding led to a search for a similar “critical period” for attachment in humans.

The correct answer is: attachment

135
Q

A(n) __________ would be most likely to agree that the basic structure of memory, attention, and other aspects of cognition are similar throughout the lifespan and that increasing complexity of these abilities during childhood are due to increases in capacity and efficiency.
Select one:

A. Piagetian
B. Vygotskian
C. information processing theorist
D. ecological systems theorist

A

Experts in the area of cognitive development have different explanations for the cognitive changes that occur during childhood.

a. Incorrect Piaget viewed cognitive development as the result of qualitative changes that occur as the consequence of biological maturation and interactions with the environment.
b. Incorrect Vygotsky focused on the impact of sociocultural factors in cognitive development.
c. CORRECT As an example, according to information processing theorists (e.g., Case, 1985; Fischer, 1980), increases in abstract reasoning during adolescence are due to increases in the underlying capacities to attend to information and to hold information in memory.
d. Incorrect Ecological systems theorists (e.g., Bronfenbrenner) emphasize the impact of external factors on development.

The correct answer is: information processing theorist

136
Q

A mother is most likely to accurately recall which of the following about her child?
Select one:

A. birth weight
B. ages when first crawled and walked alone
C. height at certain ages
D. age when read first word

A

Certain facts about one’s child seem to be recalled most easily and accurately. (Even if you are not familiar with the research in this area, you may have been able to derive the answer from your own experiences.)

a. CORRECT Although mothers may remember specific information such as grades, height, etc., they most consistently and accurately remember their children’s birth weights.
b. Incorrect See explanation for response a.
c. Incorrect See explanation for response a.
d. Incorrect See explanation for response a.

The correct answer is: birth weight

137
Q

A 12-month old baby who has been classified as “insecure/ambivalent” is reunited with his mother in Ainsworth’s “Strange Situation.” Most likely, this baby will:

Select one:
A. hit or push his mother when she approaches and continue to cry after she picks him up.
B. run away from his mother when she approaches but then cling to her when she picks him up.
C. greet his mother but try to avoid her when she attempts to pick him up.
D. ignore his mother intially but welcome her attempts to pick him up.

A

Research using the Strange Situation has identified four distinct attachment patterns - secure, insecure/avoidant, insecure/ambivalent, and disorganized-disoriented. Information about these patterns is provided in the Lifespan Development chapter of the written study materials.

a. CORRECT These behaviors are characteristic of children with a insecure/ambivalent pattern.
b. Incorrect See explanation for response a.
c. Incorrect See explanation for response a.
d. Incorrect See explanation for response a.

The correct answer is: hit or push his mother when she approaches and continue to cry after she picks him up.

138
Q

A mother of a newborn is most likely to say that, based on the nature of her baby’s cry, she can tell if her baby is:
Select one:

A. hungry, tired, or uncomfortable.
B. hungry, thirsty, or tired.
C. hungry, angry, or in pain.
D. hungry, lonely, or irritable.

A

Researchers have distinguished between three types of cries in infants. Mothers and other caretakers have confirmed that each cry is associated with a different meaning.

a. Incorrect See explanation for response c.
b. Incorrect See explanation for response c.
c. CORRECT Even very young infants seem to be able to communicate that they are either hungry, angry or in pain by the nature of their cries.
d. Incorrect See explanation for response c.

The correct answer is: hungry, angry, or in pain.

139
Q

Eric Erikson proposed that an adolescent who doesn’t successfully resolve the psychosocial conflict of identity versus identity confusion due to uncertainty about his/her sexual identity is likely to exhibit which of the following maladaptive responses?
Select one:

A. compulsion
B. inertia
C. disdain
D. repudiation

A

Erikson described the outcomes of each stage of psychosocial development in terms of basic strengths (virtues) and maladaptive responses (danger).

a. Incorrect Compulsion is the maladaptive outcome of inadequate resolution of the autonomy vs. shame/doubt conflict.
b. Incorrect This is the maladaptive outcome for the industry vs. inferiority conflict.
c. Incorrect Disdain is the result of inadequate resolution of the integrity vs. despair conflict.
d. CORRECT Repudiation is the maladaptive outcome for inadequate resolution of the identity vs. identity confusion conflict.

The correct answer is: repudiation

140
Q

Harry Harlow’s research confirmed the importance of which of the following in the development of attachment in infant monkeys?
Select one:

A. oral gratification
B. tactile comfort
C. interactional synchrony
D. pheromones

A

Harlow is best known for his research on early attachment and its consequences in infant monkeys.

a. Incorrect See explanation for response b.
b. CORRECT Harlow and his colleagues found that infant monkeys preferred physical contact with a soft terrycloth surrogate mother to contact with a hard wire surrogate that provided food. They concluded that these results indicate that contact (tactile) comfort is most critical for attachment.
c. Incorrect See explanation for response b.
d. Incorrect See explanation for response b.

The correct answer is: tactile comfort

141
Q

Research using brain imaging techniques has confirmed that repeated exposure to alcohol during prenatal development is most likely to negatively impact the:
Select one:

A. hypothalamus, thalamus, and parietal lobes.
B. medulla, thalamus, and temporal lobes.
C. basal ganglia, hippocampus, and frontal lobes.
D. brainstem, amygdala, and frontal lobes.

A

Familiarity with the symptoms of fetal alcohol syndrome (FAS) would have helped you identify the correct answer to this question.

a. Incorrect See explanation for response c.
b. Incorrect See explanation for response c.
c. CORRECT FAS is characterized by a combination of behavioral and cognitive symptoms including an intellectual disability, impaired attention and memory, hyperactivity, impaired motor coordination, impulsivity, and poor judgment. The areas of the brain listed in this answer mediate some of these functions and are often adversely affected by prenatal exposure to alcohol. Other areas that are commonly affected include the corpus callosum, cerebellum, and hypothalamus.
d. Incorrect See explanation for response c.

The correct answer is: basal ganglia, hippocampus, and frontal lobes.

142
Q
Diana Baumrind (1971) and her colleagues describe parenting styles in terms of two dimensions - demandingness and responsiveness. For example, the authoritative style is characterized by:
Select one:

A. high demandingness, high responsiveness.
B. high demandingness, low responsiveness.
C. low demandingness, high responsiveness.
D. low demandingness, low responsiveness.

A

Researchers who classify parenting styles in terms demandingness and responsiveness distinguish between four styles, which represent different combinations of levels of these characteristics.

a. CORRECT A combination of high demandingness and high responsiveness is characteristic of authoritative parents. Note, however, that the demands of authoritative parents are reasonable and involve setting and enforcing appropriate limits.
b. Incorrect A combination of high demandingness and low responsiveness is characteristic of authoritarian parents.
c. Incorrect A combination of low demandingness and high responsiveness is characteristic of permissive parents.
d. Incorrect A combination of low demandingness and low responsiveness is characteristic of uninvolved parents.

The correct answer is: high demandingness, high responsiveness.

143
Q

Longitudinal research by John M. Gottman (1994) has linked criticism, defensiveness, contempt, and stonewalling to a high risk for:
Select one:

A. divorce in the first seven years following marriage.
B. child abuse.
C. delinquency in girls in early adolescence.
D. acting out behaviors by boys following the divorce of their parents.

A

Knowing that Gottman and his colleagues have investigated risk factors for divorce would have helped you identify the correct response to this question.

a. CORRECT Gottman and his colleagues (1994) have conducted longitudinal research investigating the interactions between married couples that predict divorce. They have found that the combination of criticism, defensiveness, contempt, and stonewalling (which they refer to as the “four horseman of the Apocalypse”) is associated with a high risk for early divorce, while the suppression of both positive and negative affect predicts later divorce.
b. Incorrect See explanation above.
c. Incorrect See explanation above.
d. Incorrect See explanation above.

The correct answer is: divorce in the first seven years following marriage.

144
Q

In Piaget’s model of cognitive development, which stage is associated with the development of object permanence?
Select one:

A. sensorimotor stage
B. preoperational stage
C. concrete operational stage
D. formal operational stage

A

Object permanence is the ability of a child to recognize that an object continues to exist when the object is no longer visible.

a. CORRECT Object permanence is an accomplishment associated with the sensorimotor stage. Basic skills are developed during this stage.
b. Incorrect The preoperational stage begins the emergence of semiotic functions.
c. Incorrect Children in the concrete operational stage are capable of mental operations, which are logical rules for transforming and manipulating information.
d. Incorrect During the formal operational stage, a person is able to think abstractly, relativistically, and hypothetically.

The correct answer is: sensorimotor stage

145
Q

In general, peer pressure reaches its peak in intensity during:
Select one:

A. preadolescence (ages 10 to 13).
B. early adolescence (ages 13 to 15).
C. middle adolescence (ages 15 to 17).
D. late adolescence (ages 17 to 19).

A

The age at which conformity to peer pressure peaks varies somewhat for different types of behavior but, for most behaviors, it is most intense at around age 14 or 15.

a. Incorrect See explanation for response b.
b. CORRECT Peer pressure increases during preadolescence, peaks during early adolescence (ages 13 to 15), and then gradually declines.
c. Incorrect See explanation for response b.
d. Incorrect See explanation for response b.

The correct answer is: early adolescence (ages 13 to 15).

146
Q

With regard to empathy in childhood and adolescence, the research has generally found that:
Select one:

A. females exhibit higher levels of empathy than do males but that females and males both benefit from empathy modeling and training.
B. females exhibit higher levels of empathy than do males and that males do not benefit from empathy modeling and training.
C. females and males exhibit similar levels of empathy but that females benefit more from empathy modeling and training.
D. males exhibit higher levels of empathy than do females but that females benefit more from empathy modeling and training.

A

The research on gender differences in empathy is not entirely consistent but, in general, indicates that females of all ages exhibit higher levels of empathy (especially affective empathy) than do males.

a. CORRECT There is evidence that the gender difference in empathy is due, at least in part, to differences in childrearing practices and that, when males and females are exposed to similar empathy modeling and training, the gender gap in empathy decreases. See, e.g., L. A. Haynes and A. W. Avery, Training adolescents in self-disclosure and empathy skills, Journal of Community Psychology, 26(6), 526-530, 1979.
b. Incorrect See explanation above.
c. Incorrect See explanation above.
d. Incorrect See explanation above.

The correct answer is: females exhibit higher levels of empathy than do males but that females and males both benefit from empathy modeling and training.

147
Q

Research investigating the impact of early attachment has most consistently linked the ________ attachment pattern to aggressive behavior problems in childhood.
Select one:

A. ambivalent
B. organized avoidant
C. disorganized
D. disorganized secure

A

The studies have linked early attachment to a number of behaviors, including aggression in childhood.

a. Incorrect The ambivalent attachment pattern is relatively rare, and the research has not linked it to aggressiveness in childhood.
b. Incorrect Although some studies found a link between the avoidant attachment pattern and aggression for boys (but not for girls), other studies have not found a consistent link between the avoidant pattern and agression for boys or girls. In addition, while a disorganized avoidant pattern (a pattern that combines behaviors associated with the disorganized and avoidant types) has been linked an increased risk for aggression, an organized avoidant pattern has not.
c. CORRECT The research has most consistently linked aggression to the disorganized attachment pattern. See, e.g., K. Lyons-Ruth, Attachment relationships among children with aggressive behavior problems: The role of early disorganized attachment patterns, Journal of Consulting and Clinical Psychology, 64(1), 64-73, 1996.
d. Incorrect Disorganized secure is not one of the attachment patterns identified by the researchers.

The correct answer is: disorganized

148
Q

Which of the following is NOT characteristic of Piaget’s preoperational stage of development?
Select one:

A. magical thinking
B. insight learning
C. finalism
D. decentration

A

The preoperational stage of development spans the period from ages two through seven.

a. Incorrect Magical thinking, the belief that one has magical powers, is a result of egocentrism and precausal reasoning.
b. Incorrect Piaget believed that a major achievement of the preoperational stage is insight learning (learning based on intuition).
c. Incorrect Finalism is the belief that every event has a purpose and is characteristic of the preoperational stage.
d. CORRECT In the preoperational stage, children exhibit centration, or an inability to mentally hold two dimensions at the same time. Decentration occurs in the concrete operational stage and contributes to the ability to conserve.

The correct answer is: decentration

149
Q

Studies comparing bilingual and monolingual children on measures of inhibitory control and cognitive flexibility have generally found that:
Select one:

A. monolingual children outperform bilingual children on both measures.
B. bilingual children outperform monolingual children on both measures.
C. monolingual children outperform bilingual children on a measure of inhibitory control but bilingual children outperform monolingual children on a measure of cognitive flexibility.
D. bilingual children outperform monolingual children on a measure of inhibitory control but monolingual children outperform bilingual children on a measure of cognitive flexibility.

A

The studies have not provided entirely consistent results but have generally confirmed that bilingualism has several advantages.

a. Incorrect See explanation for response b.
b. CORRECT Several studies have found that bilingual children do better than monolingual children on measures of inhibitory control and cognitive flexibility. See, e.g., N. Akhtar and J. A. Menjivar, Cognitive and linguistic correlates of early exposure to more than one language, in J. B. Benson (Ed.), Advances in child development and behavior (pp. 41-78), London, Academic Press, 2012.
c. Incorrect See explanation for response b.
d. Incorrect See explanation for response b.

The correct answer is: bilingual children outperform monolingual children on both measures.

150
Q

A woman is told that she has a serious illness for which there is no cure and that it is likely that she will die within six months. From the perspective of Kubler-Ross’ stages that dying people pass through, you would expect the woman’s first reaction to finding out about her prognosis to be:
Select one:

A. Why me?
B. The test results must be inaccurate.
C. I must be being punished for something I did.
D. I hate my life anyway, so what difference does it make?

A

Kubler-Ross’ stages, in order, are: denial, anger, bargaining, depression, and acceptance.

a. Incorrect This is more characteristic of the second stage (anger).
b. CORRECT This statement best reflects denial, the usual initial response to learning of one’s impending death.
c. Incorrect This statement best reflects the fourth stage (depression).
d. Incorrect This is also more characteristic of the fourth stage.

The correct answer is: The test results must be inaccurate.

151
Q

Vygotsky proposed that which of the following creates a “zone of proximal development” for young children?
Select one:

A. unconditional positive regard
B. make-believe play
C. attachment
D. extended family

A

The zone of proximal development is a key concept in Vygotsky’s theory of cognitive development. It is defined as a type of support that is supplied by adults and others who are more cognitively competent than the child and that allows the child to accomplish activities with assistance that he/she will eventually be able to perform independently.

a. Incorrect See explanation for response b.
b. CORRECT Although Vygotsky focused primarily on adults and more competent peers as providers of support (“scaffolding”) in the zone of proximate development, he also viewed make-believe (symbolic) play as a support system that allows children to act and think in more cognitively complex ways.
c. Incorrect See explanation for response b.
d. Incorrect See explanation for response b.

The correct answer is: make-believe play

152
Q

At __________ months of age, infants begin to search for a hidden object but reach for the object in the last place they found it even when they have seen the object moved to another location.
Select one:

A. 4 to 8
B. 8 to 12
C. 12 to 18
D. 18 to 24

A

The behavior described in this question is referred to as the “A-not-B error.”

a. Incorrect During this stage, infants reach for a partially hidden object but stop if the object disappears from view.
b. CORRECT At about eight months of age, children first provide evidence that they know that objects exist when they are out of view. However, they commit the A-not-B error, which continues to about 12 months of age.
c. Incorrect See explanation for response b.
d. Incorrect See explanation for response b.

The correct answer is: 8 to 12

153
Q

According to Piaget, centration, or the tendency to focus on only one aspect of a situation or object while ignoring all other aspects, is characteristic of the __________ stage.
Select one:

A. preconventional
B. conventional
C. preoperational
D. concrete operational

A

Piaget describes centration as one of the reasons why young children cannot conserve.

a. Incorrect Preconventional is not one of Piaget’s stages of cognitive development.
b. Incorrect Conventional is not one of Piaget’s stages of cognitive development.
c. CORRECT During the preoperational stage, several limitations (including centration) underlie the child’s inability to conserve.
d. Incorrect Decentration - or the ability to consider more than one dimension or aspect at a time - is characteristic of the concrete operational stage.

The correct answer is: preoperational

154
Q

Phallic is to initiative vs. guilt as latency is to:
Select one:

A. autonomy vs. shame and doubt.
B. identity vs. role confusion.
C. industry vs. inferiority.
D. generativity vs. stagnation.

A

The terms presented in this question represent stages in Freud’s and Erikson’s theories of development.

a. Incorrect See explanation for response c.
b. Incorrect See explanation for response c.
c. CORRECT Freud’s phallic and Erikson’s initiative vs. guilt stages occur at about ages 3 to 6, while Freud’s latency and Erikson’s industry vs. inferiority stages occur at about ages 6 to 12.
d. Incorrect See explanation for response c.

The correct answer is: industry vs. inferiority.

155
Q

The parent of a one-year-old says her daughter shows signs of understanding some of what is said to her. However, the girl is still babbling and the closest she has come to a word is “dadada.” You:
Select one:

A. advise the parent that this is within the range of normal language development.
B. tell the parent that most children don’t speak their first words until 16 or 17 months of age.
C. refer the parent and her daughter to a speech pathologist for an evaluation of the child’s language ability.
D. consider the possibility of a diagnosis of Expressive Language Disorder.

A

At 12 months of age, children still babble, although their babbling consists primarily of identical sound sequences (“dadada”). While many children have spoken their first words by this age, it is not uncommon for first words to appear in subsequent months.

a. CORRECT The child’s language development is “on track.”
b. Incorrect Most children utter their first words prior to 17 or 18 months.
c. Incorrect The child’s language skills do not indicate developmental delays, so this is unnecessary.
d. Incorrect The child’s language abilities are not indicative of a disorder.

The correct answer is: advise the parent that this is within the range of normal language development.

156
Q

The National Campaign on Teen Pregnancy (Kirby, 2001) concluded that which of the following has the strongest evidence of success for reducing teen pregnancy rates?
Select one:

A. abstinence-only programs
B. service learning programs
C. school condom distribution programs
D. community-wide programs and initiatives

A

Although teen pregnancy rates have declined in the United States in recent years, the U.S. continues to have the highest teen pregnancy and birth rates among comparable industrialized nations.

a. Incorrect See explanation for response b.
b. CORRECT According to the NCTP, there is now evidence that some programs do, in fact, reduce pregnancy rates. These include service learning programs (e.g., Teen Outreach and Reach for Health Community Service Learning) and comprehensive multi-modal programs (i.e., the Children’s Aid Society-Carrera Program). Other authors have similarly concluded that multimodel programs that address a variety of developmental issues (including sexual behaviors) are most effective. Other programs - e.g., abstinence-only, school condom distribution, and community-wide programs - have not been found to have a substantial impact on pregnancy rates [Kirby, C. (2001). Emerging answers: Research findings on programs to reduce teen pregnancy. Summary. Retrieved August 11, 2002 from: www.teenpregnancy.org.]
c. Incorrect See explanation for response b.
d. Incorrect See explanation for response b.

The correct answer is: service learning programs

157
Q

The expression of some of our traits is restricted by our genetic make-up. In other words, for these traits, genotype restricts phenotype to a small number of outcomes. This is referred to as:
Select one:

A. equilibration.
B. canalization.
C. niche-picking.
D. epigenesis.

A

For the exam, you want to be familiar with all of the terms listed in the answers to this question. Additional information about them is provided in the Lifespan Development chapter of the study materials.

a. Incorrect Equilibration is the term Piaget used to describe the drive toward cognitive equilibrium, which he considered to be the motivating force for cognitive development.
b. CORRECT Canalization occurs when phenotype is restricted by genotype to a small number of developmental outcomes.
c. Incorrect Niche-picking is an alternative term for active genotype-environment correlation and refers to the tendency of individuals to seek experiences that are consistent with their genetic predispositions.
d. Incorrect Epigenesis refers to the bidirectional and continuous influences of heredity and environment on developmental outcomes.

The correct answer is: canalization.

158
Q

Developmental research has most consistently linked prolonged emotional stress in an expectant mother to which of the following?
Select one:

A. sudden infant death syndrome
B. suppressed immune system in the newborn
C. increased crying and irritability in the newborn
D. decreased activity levels in the newborn

A

Developmental psychologists have found that a pregnant woman’s emotional state can have lasting effects on the fetus.

a. Incorrect See explanation for response c.
b. Incorrect See explanation for response c.
c. CORRECT Of the responses given, increased crying and irritability in the newborn have been most consistently linked to severe or prolonged stress in the expectant mother.
d. Incorrect Prolonged stress in the expectant mother is associated with increased activity levels in the newborn.

The correct answer is: increased crying and irritability in the newborn

159
Q

Prader-Willi syndrome is characterized by an intellectual disability and extreme obesity. It is caused by which of the following chromosomal abnormalities?
Select one:

A. an extra chromosome
B. a missing chromosome
C. a chromosomal translocation
D. a chromosomal deletion

A

Approximately 1 in 200 babies is born with a chromosomal abnormality.

a. Incorrect Down Syndrome and Klinefelter Syndrome are two disorders attributable to the presence of an extra chromosome.
b. Incorrect Turner Syndrome is an example of a disorder caused by the absence of a chromosome. It occurs only in females and involves the absence of one X chromosome.
c. Incorrect Translocation occurs when a section of a chromosome is attached to another chromosome.
d. CORRECT Deletion occurs when part of a chromosome is missing. Prader-Willi syndrome and cru-du-chat are two disorders caused by a chromosomal deletion.

The correct answer is: a chromosomal deletion

160
Q

Difficulty in repeating words just spoken by another person and recalling the name of a familiar object are characteristic symptoms of:
Select one:

A. Wernicke’s aphasia only.
B. Wernicke’s and conduction aphasia only.
C. Wernicke’s and Broca’s aphasia only.
D. Wernicke’s, Broca’s, and conduction aphasia.

A

The three aphasias listed in the responses - Wernicke’s, Broca’s and conduction - share several characteristics. These characteristics are described in the Physiological Psychology and Psychopharmacology chapter of the written study materials.

a. Incorrect See explanation for response d.
b. Incorrect See explanation for response d.
c. Incorrect See explanation for response d.
d. CORRECT All three types of aphasia involve the two symptoms listed in the question (difficulty in repeating words and recalling the names of familiar objects). Note that conduction aphasia is due to damage to nerve fibers that connect Broca’s area to Wernicke’s area.

The correct answer is: Wernicke’s, Broca’s, and conduction aphasia.

161
Q

Weber’s law explains why:
Select one:

A. you perceive objects that are close together as a single stimulus or unity.
B. you can be heard when you whisper in a very quiet room but must shout to be heard in a very loud, noisy room.
C. you perform better in the presence of others on simple tasks but worse in the presence of others on difficult or complex tasks.
D. you feel less pain after banging your elbow when you vigorously rub your elbow or apply heat to it.

A

Weber’s law is one of the psychophysical laws that explains the relationship between physical stimuli and their psychological effects.

a. Incorrect This is the prediction made by the Gestaltian “law of proximity.”
b. CORRECT According to Weber’s law, the “just-noticeable difference” in a stimulus is proportional to the magnitude of the original stimulus. It explains why a whisper is audible in a quiet room but not in a very noisy room.
c. Incorrect This answer describes social facilitation (performing better in the presence of others on simple tasks) and social inhibition (performing worse in the presence of others on complex tasks).
d. Incorrect This answer illustrates the prediction made by the gate-control theory of pain.

The correct answer is: you can be heard when you whisper in a very quiet room but must shout to be heard in a very loud, noisy room.

162
Q

Lithium is most effective for:
Select one:

A. Bipolar Disorder with rapid cycling.
B. classic Bipolar Disorder.
C. Bipolar Disorder with prominent anxiety symptoms.
D. Bipolar Disorder with prominent depressive symptoms.

A

Lithium has traditionally been the treatment-of-choice for Bipolar Disorder. Recent studies suggest, however, that carbamazepine and other anticonvulsant drugs are more effective for certain types of Bipolar Disorder.

a. Incorrect See explanation for response b.
b. CORRECT Lithium is still considered the best choice for classic Bipolar Disorder, while carbamazepine may be more effective for rapid cycling and for dysphoric mania (mania with prominent anxiety or depressive symptoms). Additional information about these drugs is provided in the Physiological Psychology and Psychopharmacology chapter of the written study materials.
c. Incorrect See explanation for response b.
d. Incorrect See explanation for response b.

The correct answer is: classic Bipolar Disorder.

163
Q

The diagnosis of a spinal cord injury generally relies on which of the following?
Select one:

A. X-ray and MRI or CT
B. X-ray and CPAP
C. actigraphy and MRI or CT
D. actigraphy and PET or SPECT

A

The procedure used to diagnose a spinal cord injury varies somewhat from facility to facility but ordinarily includes an X-ray and either an MRI or CT.

a. CORRECT An X-ray is often the initial assessment and is used to identify fractures or other injuries to the spine. An MRI or CT is then used to obtain information on injury to the soft tissues of the spine.
b. Incorrect CPAP (continuous positive airway pressure) is a treatment for sleep apnea.
c. Incorrect Actigraphy is used to monitor body movement and is sometimes used in the assessment of sleep disorders.
d. Incorrect See explanations above.

The correct answer is: X-ray and MRI or CT

164
Q

A patient who has been taking antipsychotic medication for a lengthy period of time is being slowly taken off the drug due to the development of tardive dyskinesia. Soon after the drug is completely stopped, the patient exhibits an increase in the symptoms of tardive dyskinesia. This:
Select one:

A. means that the patient should never have been given antipsychotic medication in the first place.
B. is common, and the initial increase in symptoms will likely be followed by a reduction.
C. suggests that the symptoms are not due to the antipsychotic medication but, instead, to unrelated brain pathology.
D. indicates that the patient will not benefit from withdrawal of the drug.

A

The best way to alleviate tardive dyskinesia resulting from long-term antipsychotic drug use is to gradually withdraw the drug.

a. Incorrect See explanation for response b.
b. CORRECT Many patients show a temporary increase in symptoms of tardive dyskinesia when an antipsychotic drug is withdrawn. However, over time, the symptoms gradually reduce in severity.
c. Incorrect See explanation for response b.
d. Incorrect See explanation for response b.

The correct answer is: is common, and the initial increase in symptoms will likely be followed by a reduction.

165
Q

Difficulties in concentration, forgetfulness, sensitivity to cold, unexplained weight gain, and constipation are symptomatic of:
Select one:

A. pernicious anemia.
B. hyperadrenalism.
C. hypoglycemia.
D. hypothyroidism.

A

For the exam, you should be aware that several physical conditions can mimic mental disorders.

a. Incorrect Pernicious anemia is associated with depression, feelings of guilt and worthlessness, anorexia and weight loss, and weakness.
b. Incorrect Hyperadrenalism involves depression, anxiety, and somatic delusions.
c. Incorrect Symptoms of hypoglycemia include weakness, headache, hunger, and anxiety.
d. CORRECT The symptoms listed in the question are all associated with hypothyroidism.

The correct answer is: hypothyroidism.

166
Q

As the result of head trauma, a young man experiences damage to the primary motor cortex in the left cerebral hemisphere. This is most likely to have which of the following effects?
Select one:

A. loss of voluntary movement in his right hand and fingers
B. loss of voluntary movement in his left hand and fingers
C. loss of involuntary movement on the right side of his body
D. loss of involuntary movement on the left side of his body

A

For most functions, the right side of the body is controlled by the left hemisphere and vice-versa.

a. CORRECT The motor cortex is involved in the regulation of voluntary movements, and damage to this area is most likely to affect voluntary movements in distal parts (e.g., hands and fingers) of the opposite side of the body.
b. Incorrect See explanation for response a.
c. Incorrect See explanation for response a.
d. Incorrect See explanation for response a.

The correct answer is: loss of voluntary movement in his right hand and fingers

167
Q

A client taking Parnate, an MAO-inhibitor, should be warned against eating all of the following except:
Select one:

A. yogurt.
B. soy sauce.
C. cottage cheese.
D. over-ripe avocados.

A

People taking MAO inhibitors must avoid foods high in tyramine.

a. Incorrect See explanation for response c.
b. Incorrect See explanation for response c.
c. CORRECT Although cheese should generally be avoided, it is aged and ripe cheeses that are a problem. Cottage cheese and cream cheese are usually acceptable. (The other foods listed contain tyramine and must be avoided.)
d. Incorrect See explanation for response c.

The correct answer is: cottage cheese.

168
Q

Anosognosia is best described as an impairment in:
Select one:

A. coordination and balance.
B. speech.
C. self-awareness.
D. visual perception.

A

Anosognosia is most often caused by brain trauma that affects the right hemisphere of the brain. It involves a lack of awareness of symptoms on the left side of one’s body.

a. Incorrect See explanation for response c.
b. Incorrect See explanation for response c.
c. CORRECT Anosognosia is defined as a deficit in self-awareness and, more specifically, awareness of one’s own symptoms.
d. Incorrect See explanation for response c.

The correct answer is: self-awareness.

169
Q

The primary motor cortex in the left hemisphere controls movement on the __________ side of the body.
Select one:

A. rostral
B. caudal
C. contralateral
D. ipsilateral

A

To choose the correct response to this question, you must be familiar with the concept of “contralateral representation.”

a. Incorrect Several terms are used to describe locations in the nervous system. Rostral means “toward the beak” and is a synonym for anterior.
b. Incorrect The term caudal means “toward the tail” and is a synonym for posterior.
c. CORRECT Contralateral means “located on the opposite side of the body.” Most functions of the body are controlled by the opposite side of the brain - e.g., voluntary motor movements on the right side of the body are controlled by motor centers in the left side of the brain.
d. Incorrect Ipsilateral means “located on the same side of the body.”

The correct answer is: contralateral

170
Q

For children with ADHD, the most common side effect of methylphenidate (Ritalin) is:
Select one:

A. appetite loss.
B. dizziness.
C. pruritus.
D. headache.

A

Methylphenidate (Ritalin) is associated with a number of adverse side effects. However, these symptoms can often be alleviated by reducing the dosage of the drug.

a. CORRECT In children, loss of appetite, abdominal pain, insomnia, and tachycardia are the most common side effects of Ritalin.
b. Incorrect This is a less common side effect of methylphenidate.
c. Incorrect This side effect is uncommon.
d. Incorrect This is also an uncommon side effect.

The correct answer is: appetite loss.

171
Q

Research by Baker and Green (2005) found that, when compared to older adults with chronic pain, younger adults with chronic pain tend to report:
Select one:

A. more pain intensity but fewer symptoms of depression.
B. less pain intensity and fewer symptoms of depression.
C. more pain intensity and more symptoms of depression.
D. less pain intensity but more symptoms of depression.

A

T. A. Baker and C. R. Green looked at the effects of age and race and on chronic pain [Inter-race differences among black and white Americans presenting for chronic pain management: The influence of age, physical health, and psychosocial factors, Pain Medicine, 6, 29-38, 2005].

a. Incorrect See explanation for response c.
b. Incorrect See explanation for response c.
c. CORRECT Baker and Green found that, among White and Black adults with chronic pain, adults under 50 reported more depressive symptoms, greater pain intensity, and more problems coping with pain than did those age 50 and over.
d. Incorrect See explanation for response c.

The correct answer is: more pain intensity and more symptoms of depression.

172
Q

As a consequence of head trauma caused by a serious car accident, a middle-aged woman experiences a loss of sensation for touch, temperature, and pain in her left hand. Most likely the damage responsible for this deficit is in her right ________ lobe.
Select one:

A. frontal
B. temporal
C. occipital
D. parietal

A

To answer this question, you need to know what part of the cerebral cortex mediates cutaneous sensation.

a. Incorrect See explanation for response d.
b. Incorrect See explanation for response d.
c. Incorrect See explanation for response d.
d. CORRECT Cutaneous sensation is mediated by the somatosensory cortex, which is located in the parietal lobe.

The correct answer is: parietal

173
Q

Which of the following statements is most consistent with the current understanding of postconcussional syndrome (PCS)?
Select one:

A. The more severe the head trauma, the more severe and the longer the duration of the symptoms of PCS.
B. Most patients who develop PCS never fully recover from the motor and other physical symptoms of the disorder.
C. The symptoms of PCS are often due to a combination of organic and psychological factors.
D. In most cases, PCS is purely psychological in nature and motivated by a desire for financial compensation.

A

Postconcussional syndrome (PCS) refers to a constellation of somatic and psychological symptoms associated with head trauma, especially mild head trauma. Symptoms include headache, dizziness, fatigue, impaired memory and concentration, depression, irritability, visual impairment, and sleep disturbances.

a. Incorrect PCS most commonly occurs in cases of mild traumatic brain injury (with up to 50% of people with mild brain injury having symptoms of PCS) and is rare in cases of moderate to severe brain injury.
b. Incorrect This is the opposite of what is true: Most patients with PCS experience full recovery, with symptoms resolving within three to six months of the trauma. See, e.g., L. M. Binder, A review of mild head trauma. Part 2: Clinical implications, Journal of Clinical and Experimental Neuropsychology, 19,-457, 1997.
c. CORRECT Although the etiology of PCS continues to be debated, the experts generally agree that it has both physiological and psychological origins. Psychological factors that contribute to PCS include the individual’s subjective interpretation of the injury, premorbid personality characteristics, desire for secondary gain, and access to social support. See, e.g., L. M. Ryan and D. M. Warden, Post concussion syndrome, International Review of Psychiatry, 15(4), 310-316, 2003.
d. Incorrect The majority of individuals with PCS are not seeking financial compensation (i.e., are not involved in litigation). Moreover, the deficits associated with the disorder are often more serious than the individual realizes, and there is evidence that, at least some symptoms, have a physiological cause.

The correct answer is: The symptoms of PCS are often due to a combination of organic and psychological factors.

174
Q

Administering Ritalin to a child to confirm or rule out a diagnosis of ADHD is:
Select one:

A. contraindicated since Ritalin increases activity levels and reduces attention in normal children.
B. not useful because only a minority of children with ADHD respond to Ritalin.
C. not useful since the effects of Ritalin are similar for children with and without ADHD.
D. useful, especially in cases where the child’s symptoms are highly variable.

A

Ritalin, a CNS stimulant, has been found to have a “paradoxical effect” in children and adults with and without ADHD.

a. Incorrect See explanation for response c.
b. Incorrect See explanation for response c.
c. CORRECT Using a drug is usually not the best strategy for confirming or ruling out a diagnosis. It is not useful in ADHD because, as noted above, individuals with or without the disorder have a similar response to the drug.
d. Incorrect See explanation for response c.

The correct answer is: not useful since the effects of Ritalin are similar for children with and without ADHD.

175
Q

In adults, the early motor symptoms of Huntington’s disease most often involve:
Select one:

A. tingling, numbness, and weakness in one limb.
B. repetitive movements in the extremities and face.
C. slowness and poverty of movement.
D. difficulty swallowing and speaking.

A

The early signs of Hungtington’s disease vary from individual to individual. For some, the initial symptoms are motor; for others, affective or cognitive.

a. Incorrect These symptoms are characteristic of multiple sclerosis.
b. CORRECT For many adults, early motor symptoms include rapid, repetitive, involuntary movements in the extremities and face - e.g., “piano-playing” movements of the fingers and facial grimaces.
c. Incorrect Slowness and poverty of movement are characteristic of Parkinson’s disease.
d. Incorrect Difficulty swallowing and speaking are characteristic of the later stages of Huntington’s disease.

The correct answer is: repetitive movements in the extremities and face.

176
Q

The most commonly-prescribed drugs for Tourette’s syndrome are drugs that:
Select one:

A. increase dopamine levels.
B. block dopamine transmission.
C. increase acetylcholine levels.
D. block acetylcholine transmission.

A

Knowing that haloperidol is a frequently-prescribed drug for Tourette’s syndrome would have helped you pick the correct answer to this question.

a. Incorrect See explanation for response b.
b. CORRECT Haloperidol is an antipsychotic drug that exerts its effects by blocking the transmission of dopamine. It has been found effective for alleviating the symptoms of Tourette’s syndrome in about 70% of patients.
c. Incorrect See explanation for response b.
d. Incorrect See explanation for response b.

The correct answer is: block dopamine transmission.

177
Q

There is evidence that, as a group, Asians have unique responses to psychotropic medications in terms of both therapeutic and side effects. Specifically, in comparison to Caucasian patients, Asian patients often:
Select one:

A. require lower doses to obtain the same therapeutic effects and experience more severe side effects from the same dose.
B. require lower doses to obtain the same therapeutic effects but experience less severe side effects from the same dose.
C. require higher doses to obtain the same therapeutic effects but experience more severe side effects from the same dose.
D. require higher doses to obtain the same therapeutic effects and experience less severe side effects from the same dose.

A

Studies have shown that, in some cases, the effects of the psychotropic drugs are related to race/ethnicity.

a. CORRECT As a group, Asians are more sensitive to psychotropic drugs in terms of both therapeutic and side effects. For this reason, the experts generally agree that the best course of action, when starting an Asian patient on a new drug, is to begin with a lower dosage than is recommended for Caucasian patients. See e.g., K. M. Lin and F. Cheung, Mental health issues for Asian Americans, Psychiatric Services, 50(6), 774-780, 1999.
b. Incorrect See explanation for response a
c. Incorrect See explanation for response a.
d. Incorrect See explanation for response a.

The correct answer is: require lower doses to obtain the same therapeutic effects and experience more severe side effects from the same dose.

178
Q

The suprachiasmatic nucleus (SCN) has been implicated in the control of circadian rhythms. The SCN is located in the:
Select one:

A. thalamus.
B. hypothalamus.
C. amygdala.
D. caudate nucleus.

A

Mention of the SCN may have “thrown” you, but what you need to know to answer this question is which region of the brain controls circadian rhythms.

a. Incorrect See explanation for response b.
b. CORRECT A good rule of thumb is, “when in doubt, go with the hypothalamus.” Although very small in size, the hypothalamus is involved in a variety of important functions including control of circadian rhythms.
c. Incorrect See explanation for response b.
d. Incorrect See explanation for response b.

The correct answer is: hypothalamus.

179
Q

_________ is most often used to initially diagnose a cerebral contusion because it is usually available in medical settings, is cost effective, and can be performed quickly.
Select one:

A. MRI
B. CT
C. SPECT
D. PET

A

A cerebral contusion is bruising of the brain that is caused by a head injury.

a. Incorrect An MRI is more sensitive and accurate than a CT scan but is often not available, is more costly, takes more time, and may require sedation of the patient. Consequently, it is used less often than a CT scan for the initial detection of a contusion.
b. CORRECT A CT scan is most commonly used to initially detect a contusion for the reasons stated in this question. See, e.g., B. Lee and A. Newberg, Neuroimagining in traumatic brain imaging, NeuroRx, 2(2), 372-383, 2005.
c. Incorrect See explanation above.
d. Incorrect See explanation above.
The correct answer is: CT

180
Q

A partial seizure is characterized by:
Select one:

A. a focal onset in one hemisphere of the brain with or without a loss of consciousness.
B. a focal onset in one hemisphere of the brain without a loss of consciousness.
C. a focal onset in one hemisphere of the brain with a loss of consciousness.
D. a simultaneous onset in both hemispheres of the brain without a loss of consciousness.

A

Epileptic seizures are categorized as either partial or generalized.

a. CORRECT Partial seizures begin in one hemisphere and affect movement and sensations beginning on one side of the body. Simple partial seizures do not affect consciousness, while complex partial seizures do alter consciousness.
b. Incorrect See explanation above.
c. Incorrect See explanation above.
d. Incorrect See explanation above.

The correct answer is: a focal onset in one hemisphere of the brain with or without a loss of consciousness.

181
Q

Increasing age is LEAST likely to have which of the following effects on a man’s sexual response cycle?
Select one:

A. The time to achieve an erection will increase.
B. Complete penile erection may not occur until just prior to orgasm.
C. The resolution phase and refractory period will both increase.
D. All of the above occur.

A

A number of predictable changes in the sexual response cycle accompany aging.

a. Incorrect This is a characteristic change. Unfortunately, this change, if unexpected, is often misinterpreted as a sign of impotence.
b. Incorrect This is also a characteristic change.
c. CORRECT Although the refractory period begins to increase in the 30s or 40s (and may last for several hours by age 60), the resolution phase actually decreases, with loss of erection following orgasm being quite rapid.
d. Incorrect Only response c does not accurately describe age-related changes in the sexual response cycle.

The correct answer is: The resolution phase and refractory period will both increase.

182
Q

For some patients with type 2 diabetes, acute hyperglycemia can produce:
Select one:

A. substantial mood and cognitive changes.
B. substantial mood changes only.
C. substantial cognitive changes only.
D. neither substantial mood nor substantial cognitive changes.

A

Diabetes is associated with rapid changes in blood glucose levels, and there is evidence that these fluctuations are associated with both mood and cognitive changes.

a. CORRECT Hyperglycemia refers to elevated blood glucose levels. There is evidence that, for some patients with type 2 diabetes, acute hyperglycemia produces substantial impairments in speed of information processing, working memory, and some aspects of attention as well as decreased happiness and alertness and increased agitation. [A. J. Sommerfield, I. J. Deary, and B. M. Frier, Acute hyperglycemia alters mood state and impairs cognitive performance in people with type 2 diabetes, Diabetes Care, 27(10), 2335-2340, 2004.]
b. Incorrect See explanation for response a.
c. Incorrect See explanation for response a.
d. Incorrect See explanation for response a.

The correct answer is: substantial mood and cognitive changes.

183
Q

The most consistent evidence for neurogenesis in human adults has been provided for which of the following areas of the brain?
Select one:

A. reticular formation
B. hippocampus
C. medulla oblongata
D. hypothalamus

A

Neurogenesis is the development of new neurons.

a. Incorrect See explanation for response b.
b. CORRECT Until relatively recently, it was believed that humans do not experience neurogenesis following the postnatal period. However, there is now consistent evidence that neurogenesis continues throughout the lifespan in the hippocampus and olfactory bulb. (See, e.g., P. S. Erikkson et al., Neurogenesis in the adult human hippocampus, Nature Medicine, 4, 1313-1317, 1998.) While there have been some reports of postnatal neurogenesis in the caudate nucleus, amygdala, neocortex, and substantia nigra, these findings have not been consistently replicated.
c. Incorrect See explanation for response b.
d. Incorrect See explanation for response b.
The correct answer is: hippocampus

184
Q

Visual agnosia is best conceptualized as:
Select one:

A. visual “not caring.”
B. visual “not knowing.”
C. visual repression.
D. visual blindness.

A

Visual agnosia is the loss of the ability to recognize visual stimuli even though visual acuity is intact.

a. Incorrect See explanation for response b.
b. CORRECT The term “agnosia” is derived from the Greek word “agnostos,” which means “unknown, unknowable, not knowing.”
c. Incorrect See explanation for response b.
d. Incorrect See explanation for response b.
The correct answer is: visual “not knowing.”

185
Q

The Working Memory Index of the WAIS-IV consists of which of the following subtests?
Select one:

A. Digit Span, Arithmetic, and Letter-Number Sequencing
B. Vocabulary, Similarities, Information, and Comprehension
C. Symbol Search, Coding, and Cancellation
D. Block Design, Matrix Reasoning, Visual Puzzles, Figure Weights, and Picture Completion

A

Answer A is correct: The WAIS-IV provides scores on four Indexes – Verbal Comprehension, Perceptual Reasoning, Working Memory, and Processing Speed. These are the subtests of the Working Memory Index.

Answer B: Verbal Comprehension subtests: Vocabulary, Similarities, Information, and Comprehension

Answer C: Processing Speed subtests: Symbol Search, Coding, and Cancellation

Answer D: Perceptual Reasoning subtests: Block Design, Matrix Reasoning, Visual Puzzles, Figure Weights, and Picture Completion

The correct answer is: Digit Span, Arithmetic, and Letter-Number Sequencing

186
Q

Arthur Jensen’s (1998) research on sources of variability in IQ indicated that the average IQ difference within families (as measured by the difference in IQs of full siblings living together) is _____ points.
Select one:

A. 4
B. 11
C. 18
D. 26

A

Unfortunately, the licensing exam will include three or four “distant galaxy” questions like this one that require you to be familiar with the details of a particular research study. When you encounter these questions in practice exams, try to memorize the information presented in those questions. When you encounter them on the licensing exam, take a deep breath and see if you have any knowledge that will help you make an “educated guess.”

a. Incorrect See explanation for response b.
b. CORRECT Jensen found that the average IQ difference between siblings in the same family was 11 points, while the average difference in IQs of the same person tested on two occasions a week apart was 4 points. (A. R. Jensen, The g factor: The science of mental ability, Westport, CT, Praeger, 1998.)
c. Incorrect See explanation for response b.
d. Incorrect See explanation for response b.

The correct answer is: 11

187
Q

Scoring and interpretation of the Thematic Apperception Test (TAT) usually involves considering which of the following?
Select one:

A. form quality and content
B. emotional control/lability
C. needs and press
D. general and specific attitudes

A

The TAT is a projective personality test that was derived from Henry Murray’s theory of needs.

a. Incorrect Form quality and content are two of the categories of Exner’s comprehensive scoring system for the Rorschach test.
b. Incorrect Emotional control/lability are not factors considered when scoring and interpreting TAT scores.
c. CORRECT Most scoring and interpretation systems for the TAT rely on Murray’s concepts of needs (internal determinants of behavior), press (environmental determinants of behavior), and thema (interactions between needs and press).
d. Incorrect General and specific attitudes are not factors considered when scoring and interpreting TAT responses.

The correct answer is: needs and press

188
Q

In the assessment of cognitive abilities with a standardized test, you would most likely “test the limits”:
Select one:

A. before administering the test using standardized procedures.
B. as an alternative to administering the test using standardized procedures.
C. after administering the test using standardized procedures.
D. whenever it seems appropriate to do so.

A

Testing the limits was one of the early methods used to obtain additional qualitative information about an examinee’s abilities.

a. Incorrect See explanation for response c.
b. Incorrect See explanation for response c.
c. CORRECT Testing the limits is done after the entire test has been administered under standardized conditions. It is used to obtain additional information about an examinee and may involve providing cues or asking questions.
d. Incorrect See explanation for response c.

The correct answer is: after administering the test using standardized procedures.

189
Q

Which of the following statements is true about the Stanford-Binet Intelligence Scales, 5th edition (SB5)?
Select one:

A. The Full Scale IQ, Verbal IQ, Nonverbal IQ, and Cognitive Indexes have a mean of 100 and standard deviation of 16.
B. The test is appropriate only for individuals ages 5 through 65.
C. It provides scores on four factor indexes – Working Memory, Verbal Comprehension, Processing Speed, and Perceptual Reasoning.
D. Its development was based on a hierarchical model of intelligence that begins with a global “g” factor.

A

Answer D is correct: For the exam, you want to be familiar with the major characteristics of the SB5, including its age range and the scores that it provides. This answer accurately describes the model underlying the SB5.

Answer A: For the scores listed in this answer, the mean is 100 and the standard deviation is 15.

Answer B: The age range is 2 to 85+.

Answer C: These are the Indexes for the WAIS-IV, not the SB5. The SB5 has five Cognitive Indexes – Fluid Reasoning, Knowledge, Quantitative Reasoning, Visual-Spatial Processing, and Working Memory.

The correct answer is: Its development was based on a hierarchical model of intelligence that begins with a global “g” factor.

190
Q

When conducting a functional behavioral assessment, a behavior’s function is described in terms of:
Select one:

A. the skills required to perform it.
B. the criteria used to measure it.
C. its antecedents and consequences.
D. its physical characteristics.

A

As its name suggests, a primary goal of a functional behavioral assessment is to determine the function of the target behavior.

a. Incorrect See explanation for response c.
b. Incorrect See explanation for response c.
c. CORRECT A functional behavioral assessment involves determining the environmental factors (antecedents and consequences) that contribute to its occurrence.
d. Incorrect See explanation for response c.

The correct answer is: its antecedents and consequences.

191
Q

The results of the Seattle Longitudinal Study found that:
Select one:

A. Measures of crystallized and fluid intelligence both begin to show substantial age-related declines in the mid- to late-40s.
B. Measures of crystallized intelligence show earlier age-related declines than measures of fluid intelligence.
C. Measures of perceptual speed and numeric ability begin to show age-related declines sooner than measures of vocabulary and verbal memory.
D. Measures of inductive reasoning, spatial orientation, and verbal memory begin to show age-relaed declines in the mid- to late-20s.

A

For the exam, you want to be familiar with the results of the Seattle Longitudinal Study, which are described in the Psychological Assessment chapter of the written study materials.

a. Incorrect See explanation for response c.
b. Incorrect See explanation for response c.
c. CORRECT The Seattle Longitudinal Study (which used a cross-sequential design that combined cross-sectional and longitudinal methodologies) found that, overall, intelligence remains relatively stable until the late 60s. However, of the mental abilities, perceptual speed and numeric ability are the first to decline.
d. Incorrect See explanation for response c.

The correct answer is: Measures of perceptual speed and numeric ability begin to show age-related declines sooner than measures of vocabulary and verbal memory.

192
Q

An adult with traumatic brain injury is likely to obtain the lowest scores on which of the following WAIS-IV subtests?
Select one:

A. Symbol Search, Coding, and Cancellation
B. Block Design, Visual Puzzles, and Picture Completion
C. Digit Span, Arithmetic, and Letter-Number Sequencing
D. Vocabulary, Similarities, and Information

A

Answer A is correct: The WAIS-IV Technical Manual provides score profiles for various clinical groups. It indicates that individuals in several clinical groups – e.g., traumatic brain injury, mild Alzheimer’s dementia, and ADHD – obtained the lowest score on the Processing Speed Index, which consists of the Symbol Search, Coding, and Cancellation subtests.

The correct answer is: Symbol Search, Coding, and Cancellation

193
Q

In the 1979 case of Larry P. v. Riles, the judge ruled that IQ tests could no longer be used to:
Select one:

A. make hiring and other employment decisions.
B. make college admission decisions.
C. determine if older African American adults should be assigned a legal guardian.
D. determine if African American children should be placed in special education classes.

A

In the Larry P. case, Judge Peckham concluded that the use of standard IQ tests resulted in the placement of a disproportionate number of African American children in special education classes.

a. Incorrect See explanation for response d.
b. Incorrect See explanation for response d.
c. Incorrect See explanation for response d.
d. CORRECT As noted above, Judge Peckham concluded that standard IQ tests should not be used to assess African American children because their use resulted in too many of these children being placed in “dead-end” (special education) classes.

The correct answer is: determine if African American children should be placed in special education classes.

194
Q

Berkowitz (1971) added which of the following to the original frustration-aggression hypothesis?
Select one:

A. the role of aggressive cues
B. the impact of the model’s attractiveness
C. the role of the individual’s level of frustration tolerance
D. the likelihood that aggressive behavior will be rewarded or punished

A

The frustration-aggression hypothesis was one of the first psychological theories of aggression and proposd that frustration triggers a readiness to act aggressively.

a. CORRECT Berkowitz extended the original theory by proposing that unjustified frustration leads to anger and that this anger is most likely to elicit aggression in the presence of aggressive cues.
b. Incorrect See explanation for response a.
c. Incorrect See explanation for response a.
d. Incorrect See explanation for response a.

The correct answer is: the role of aggressive cues

195
Q

Sherif et al.’s (1961) “Robber’s Cave” study is considered important because of the information it provided on:
Select one:

A. social facilitation and inhibition.
B. competition and cooperation.
C. bystander apathy.
D. conformity to group norms.

A

Sherif et al.’s (1961) study was one of the first that investigated factors that produce and alleviate intergroup competition.

a. Incorrect See explanation for response b.
b. CORRECT Sherif and his colleagues found that introducing a superordinate goal that could be accomplished only when members of two groups cooperated was the most effective way for reducing intergroup competition and hostility.
c. Incorrect See explanation for response b.
d. Incorrect See explanation for response b.

The correct answer is: competition and cooperation.

196
Q

__________ predicts that the decision to remain or leave a relationship depends on the relative costs and rewards of that relationship.
Select one:

A. Social comparison theory
B. Self-verification theory
C. Social exchange theory
D. Gain-loss theory

A

For the exam, you want to be familiar with the basic assumptions and predictions of the four theories listed in the answers to this question. Additional information about these theories is provided in the Social Psychology chapter of the written study materials.

a. Incorrect Social comparison theory predicts that, when people do not have objective standards for evaluating their own opinions and behaviors, they tend to do so by comparing their opinions and behaviors to those of others.
b. Incorrect Self-verification theory predicts that people prefer to interact with others who provide them with feedback that is consistent with their self-views.
c. CORRECT Social exchange theory predicts that we’re more likely to stay in a relationship when the rewards of the relationship exceed its costs.
d. Incorrect The gain-loss effect predicts that our liking for another person is maximized when that person’s initial evaluation of us is negative but subsequently becomes positive.

The correct answer is: Social exchange theory

197
Q

The impact of social roles on behavior was demonstrated by:
Select one:

A. Berkowitz’s research on the frustration-aggression hypothesis.
B. Latane and Darley’s research on bystander apathy.
C. Zimbardo’s prison study.
D. Sherif’s Robber’s Cave study.

A

For the exam, you want to be familiar with all of the studies listed in the answers to this question. These studies are described in the Social Psychology chapter of the written study materials.

a. Incorrect See explanation for response c.
b. Incorrect See explanation for response c.
c. CORRECT Zimbardo’s (1972) prison study demonatrated that, under certain conditions, people will behave in uncharacteristically irrational and unacceptable ways that are consistent with assigned social roles.
d. Incorrect See explanation for response c.

The correct answer is: Zimbardo’s prison study.

198
Q

Lewin’s field theory predicts that:
Select one:

A. human behavior is due more to physical than to psychological factors.
B. a leader’s power is directly affected by the characteristics of the task and environment.
C. human behavior is a function of both the person and his/her environment.
D. a group can be no more effective than its least effective member.

A

Lewin is probably best known for research on field theory.

a. Incorrect See explanation for response c.
b. Incorrect See explanation for response c.
c. CORRECT According to Lewin’s field theory, behavior is a function of the person and the environment. He expressed this relationship with the following formula: B = f(P, E).
d. Incorrect See explanation for response c.

The correct answer is: human behavior is a function of both the person and his/her environment.

199
Q

Which of the following are related to the psychology of oppression?
Select one:

A. Strength, learning, knowledge, and cultural individuality.
B. Perceptions, willpower, self-surrender, and power status.
C. Apathy, individualism, loss of identity, and group think.
D. Motivation, emotions, ambitions, and ideals.

A

The correct answer is D. The psychology of oppression consists of: motivation, agency, perception, emotions, ambitions, ideals, reasoning, memory, aesthetics, and morals that accept the oppressive social system, desire it, identify with it, take it for granted as normal and even as ideal, take pleasure in it, defend it, and reject alternatives to it. Answers A, B and C are incorrect as none of these are related to the psychology of oppression other than perception.

The correct answer is: Motivation, emotions, ambitions, and ideals.

200
Q

A manager engaged in long-range planning for his company is fairly confident about the strategic plan he has developed for the future but has some reservations about its acceptability. In this situation, the approach-avoidance conflict model predicts that, as it gets closer to the time to implement the plan:
Select one:

A. the positive aspects of the plan will increase in strength for the manager while the negative aspects decrease.
B. the negative aspects of the plan will increase in strength for the manager while the positive aspects decrease.
C. the positive and negative aspects of the plan will both increase in strength for the manager, but the negative aspects will increase more.
D. the positive and negative aspects of the plan will both decrease in strength for the manager, but the positive aspects will decrease more.

A

The approach-avoidance conflict is an intraindividual conflict that occurs when a goal or alternative has both positive and negative qualities. For information on other types of intraindividual conflict, see the Social Psychology chapter of the written study materials.

a. Incorrect See explanation for response c.
b. Incorrect See explanation for response c.
c. CORRECT As distance from the goal decreases, the strength of both the “approach gradient” and the “avoidance gradient” increases. However, the strength of the avoidance gradient increases more rapidly, meaning that, as you get closer to the goal, the more likely that you will choose to avoid it.
d. Incorrect See explanation for response c.

The correct answer is: the positive and negative aspects of the plan will both increase in strength for the manager, but the negative aspects will increase more.

201
Q

A worker whose job consists primarily of handling customer complaints will be happier if she blames the customer’s problems on:
Select one:

A. external, stable, and global factors.
B. internal, stable, and specific factors.
C. external, unstable, and global factors.
D. external, unstable, and specific factors.

A

Attribution theory has been applied to the learned helplessness model of depression, and studies looking at the attributions of depression people indicate that they tend to make internal, stable, global attributions, especially of negative events.

a. Incorrect See explanation for response d.
b. Incorrect See explanation for response d.
c. Incorrect See explanation for response d.
d. CORRECT This is the best answer since external, unstable, specific attributions are the exact opposite of those associated with depression. It also seems logical that one will experience a better sense of self-efficacy if the problems of customers are controllable and changeable, which they would be if they are due to external, unstable, and specific factors.

The correct answer is: external, unstable, and specific factors.

202
Q

According to the overjustification hypothesis:
Select one:

A. external rewards increase intrinsic interest.
B. intrinsic interest decreases the valuation of external rewards.
C. external rewards decrease intrinsic interest.
D. intrinsic interest increases the valuation of external rewards.

A

The overjustification hypothesis predicts that, when an external reward is given to a person for performing an intrinsically rewarding activity, the person’s intrinsic activity will decrease.

a. Incorrect See explanation for response c.
b. Incorrect See explanation for response c.
c. CORRECT Results of research demonstrating this phenomenon have been interpreted in terms of Bem’s self-perception theory, which proposes that people make attributions about their own attitudes and feelings by observing their own behaviors. This theory implies that a person observing herself doing something for an external reward would conclude that the external reward, and not intrinsic interest, is the cause of her behavior.
d. Incorrect See explanation for response c.

The correct answer is: external rewards decrease intrinsic interest.

203
Q

According to Bem’s (1972) self-perception theory:
Select one:

A. our attitudes and behaviors are unrelated.
B. our emotions shape (cause) our attitudes and behaviors.
C. we infer our attitudes from our behaviors.
D. we rely on our attitudes to choose a behavior.

A

Bem’s theory contradicts the more traditional view that attitudes determine behaviors.

a. Incorrect See explanation for response c.
b. Incorrect See explanation for response c.
c. CORRECT According to Bem, people infer their own attitudes and emotions from their behaviors.
d. Incorrect See explanation for response c.

The correct answer is: we infer our attitudes from our behaviors.

204
Q

When using protocol analysis to assess problem-solving in adolescents, a psychologist will ask each adolescent to:
Select one:

A. assemble a representative set of task assignments that the psychologist will review.
B. report the first answer he/she thinks of when presented with a problem.
C. describe how he/she determined the solution after working on a problem.
D. think aloud while solving a problem.

A

Protocol analysis involves analyzing the transcript (protocol) of a person’s verbalizations while solving a problem, making a decision, etc.

a. Incorrect This sounds like portfolio assessment.
b. Incorrect This sounds like free association.
c. Incorrect This is called “retrospective debriefing.”
d. CORRECT Having a person think-aloud while solving a problem, recording the person’s verbalizations, and then analyzing the transcript of that recording are the procedures used in protocol analysis.

The correct answer is: think aloud while solving a problem.

205
Q

When using structural equation modeling, the “fit” between the proposed causal model and the obtained data can be evaluated using the chi-square test for goodness-of-fit. However, one problem with using the chi-square test for this purpose is that:
Select one:

A. it is very sensitive to the sample size.
B. it is very insensitive to the sample size.
C. it is difficult to interpret when there are more than three predictors.
D. it is difficult to interpret when there are more than three criteria.

A

This question addresses a very advanced topic from the area of statistics. Fortunately, you won’t encounter very many questions of this difficulty level on the licensing exam.

a. CORRECT The chi-square test is very sensitive to sample size, with large samples producing statistically significant results even when there is a small difference between the proposed model and the obtained data. Because the goal in using the chi-square test in this situation is to NOT obtain a significant difference (i.e., you want the data to fit the model), a large sample can be problematic.
b. Incorrect See explanation for response a.
c. Incorrect See explanation for response a.
d. Incorrect See explanation for response a.

The correct answer is: it is very sensitive to the sample size.

206
Q

Dr. Stanley Stat conducts a meta-analysis of the research on the effects of a behavioral treatment for ADHD for adults and obtains an average effect size (Cohen’s d) of .50. Based on these results, he concludes that the effect size for adults is twice the effect size previously reported in a meta-analysis of the research on the effects of this treatment for adolescents. Apparently, the reported effect size for adolescents was:
Select one:

A. .25.
B. .70.
C. 1.0.
D. -.50.

A

Knowing that Cohen’s d indicates the difference between groups (usually treatment and no treatment groups) in terms of standard deviation units would have helped you identify the correct answer to this question.

a. CORRECT A standard deviation of .50 is twice the size of a standard deviation of .25.
b. Incorrect See explanation for response a.
c. Incorrect See explanation for response a.
d. Incorrect See explanation for response a.

The correct answer is: .25.

207
Q

The Bonferroni test helps control the experimentwise error rate by:
Select one:

A. controlling the total number of comparisons that can be made.
B. reducing the level of significance for each comparison.
C. permitting individual comparisons only after the omnibus test has produced significant results.
D. requiring that all comparisons be conducted as two-tailed tests.

A

For the exam, you want to have the Bonferroni test associated with planned (a priori) comparisons.

a. Incorrect See explanation for response b.
b. CORRECT The greater the number of planned comparisons, the greater the likelihood that a Type I error will be made. The Bonferroni test controls this possibility by reducing the level of alpha for each comparison.
c. Incorrect The Bonferroni test takes the place of the omnibus test (ANOVA) and post-hoc comparisons.
d. Incorrect See explanation for response b.

The correct answer is: reducing the level of significance for each comparison.

208
Q

Dr. Chang evaluates the relationship between a child’s behavior and rejection of the child by peers by observing the behavior of children previously classified as rejected or non-rejected in free-play situations over several days. During each observation period, Dr. Chang observes the first child on a list for 10 seconds and then records the child’s behavior, observes the next child on the list for 10 seconds and records his/her behavior, and so on until all children have been observed. Dr. Chang is using which of the following strategies?
Select one:

A. time sampling
B. time series
C. sequential act coding
D. multiple baseline

A

In the situation described in this question, Dr. Chang is observing and recording each child’s behavior during specific periods of time.

a. CORRECT Time sampling involves observing an individual or group of individuals during prespecified periods of time and, at the end of each observation, recording whether or not the target behavior(s) occurred. Time sampling is usually categorized as a type of interval recording.
b. Incorrect When using a time series design, the dependent (outcome) variable is assessed several times at regular intervals before and after administration of the independent variable.
c. Incorrect Sequential act coding involves recording events or behaviors in the order in which they occur.
d. Incorrect A multiple baseline design is single-case (single-subject) design that involves sequentially applying a treatment across baselines - i.e., across participants, behaviors, or settings.

The correct answer is: time sampling

209
Q

To determine the degree of association between two continuous variables that have been artificially dichotomized, you would use which of the following correlation coefficients?
Select one:

A. eta
B. biserial
C. tetrachoric
D. contingency

A

For the licensing exam, you want to be familiar with the uses of the four correlation coefficients listed in the answers to this question.

a. Incorrect Eta is used to determine the relationship between two continuous variables when the relationship is known to be nonlinear.
b. Incorrect The biserial correlation coefficient is used to determine the relationship between one continuous variable and one artificially dichotomized variable.
c. CORRECT The tetrachoric correlation coefficient is used to determine the relationship between two normally distributed continuous variables that have been artificially dichotomized. For example, it would be used to assess the degree of association between treatment outcome and symptom severity, when both variables were originally measured on a continuous scale but were then dichotomized so that outcome is categorized as either successful or unsuccessful and symptom severity is categorized as either mild or severe.
d. Incorrect The contingency correlation coefficient is used to determine the relationship between two nominal variables.

The correct answer is: tetrachoric

210
Q

To use two or more categorical variables to predict status on a single categorical variable, you would use which of the following?
Select one:

A. path analysis
B. logit analysis
C. multiple regression analysis
D. canonical correlation analysis

A

Answer B is correct: Logit analysis is the appropriate multivariate technique in this situation. If you’re familiar with the multivariate techniques described in the Statistics and Research Design chapter, you may have been able to identify the correct answer to this question through the process of elimination.

Answer A: Path analysis is used to test hypotheses about the causal relationships among three or more variables.

Answer C: Multiple regression requires the predictors and criterion to be measured on a continuous scale.

Answer D: Canonical correlation is used when there are multiple IVs and multiple DVs that are all measured on a continuous scale.

The correct answer is: logit analysis

211
Q

“Autocorrelation” is most likely to be a problem when using which of the following research designs?
Select one:

A. Solomon four-group
B. repeated measures
C. between groups
D. double-blind

A

Answer B is correct: Autocorrelation refers to the correlation between measurements of the dependent variable when the dependent variable is repeatedly administered to the same subjects.

The correct answer is: repeated measures

212
Q

A researcher wants to compare the effects of four different prevention programs on willingness to use safe-sex practices for sexually active male and female adolescents. Participants will be randomly assigned to one of the four programs and six months later will be asked to indicate if they engaged in safe sex all of the time, some of the time, or never. The appropriate statistical test for analyzing the obtained data is which of the following?
Select one:

A. one-way ANOVA
B. two-way ANOVA
C. multiple-sample chi-square test
D. single-sample chi-square test

A

Answer C is correct: This study has two independent variables (prevention program and gender) and one dependent variable (safe-sex practices). While the dependent variable is actually measured on an ordinal scale, no ordinal tests are listed in the responses. It is always possible to treat an ordinal variable as a nominal one but it is not acceptable to treat it as an interval or ratio variable. Of the tests listed in the answer, the multiple-sample chi-square is the best choice.

Answer A: The one-way ANOVA is used with one IV and one DV that is measured on an interval or ratio scale.

Answer B: The two-way ANOVA is used with two IVs and one DV that is measured on an interval or ratio scale.

Answer D: The single sample chi-square test is used when a study includes only one nominal variable. For additional information on the use of the chi-square tests, see the Statistics and Research Design chapter of the written study materials.

The correct answer is: multiple-sample chi-square test

213
Q

A Type I error occurs when, on the basis of a study’s results, a researcher:
Select one:

A. retains a false null hypothesis.
B. rejects a false null hypothesis.
C. retains a true null hypothesis.
D. rejects a true null hypothesis.

A

Answer D is correct: For the exam, you want to be familiar with Type I and Type II errors so that you can answer questions like this one. The Statistics and Research Design chapter of the written study materials includes a decision outcome table that might help you memorize the difference between these errors.

The correct answer is: rejects a true null hypothesis.

214
Q

To determine if there is a statistically significant pattern in the effect of time on memory for a list of nonsense syllables, you have subjects memorize a list of 15 syllables and then test their memory at 15-minute intervals for the next two hours. The best technique for analyzing the data you have collected is:
Select one:

A. multiple regression.
B. factorial ANOVA.
C. orthogonal analysis.
D. trend analysis.

A

Answer D is correct: In this situation, you want to determine if there is a pattern (or “trend”) in forgetting the list of nonsense syllables. Of the techniques listed, only trend analysis (a type of analysis of variance that is used when the IV is quantitative) would be useful for determining if there is a pattern or trend in subjects’ memory for the list of nonsense syllables over time.

The correct answer is: trend analysis.

215
Q

Which of the following is true about the standard error of the mean?
Select one:

A. It increases as the standard deviation decreases.
B. It is not affected by the size of the standard deviation.
C. It decreases as sample size increases.
D. It decreases as the number of samples increases.

A

The standard error of the mean is the standard deviation of the sampling distribution of the mean. It is equal to the population standard deviation divided by the square root of the sample size.

a. Incorrect As the population standard deviation increases or decreases, the standard error of the mean changes in the same direction: i.e., as one increases, the other increases.
b. Incorrect The standard error of the mean is affected by the standard deviation, so changes in the standard deviation will impact the standard error.
c. CORRECT As sample size increases, the standard error decreases.
d. Incorrect The standard error of the mean is affected by the size (not the number) of samples.

The correct answer is: It decreases as sample size increases.

216
Q

When using __________, examinees who obtain scores that fall within a specified range of scores are considered to have received identical scores.
Select one:

A.
top-down selection

B.
within-group norming

C.
separate cutoff scores

D.
banding

A

The procedures listed in the answers are methods of score adjustment that are used to take group differences into account when assigning or interpreting scores. They are described in the Test Construction chapter of the written study materials.

a. Incorrect See explanation for response d.
b. Incorrect See explanation for response d.
c. Incorrect See explanation for response d.
d. CORRECT Banding is being used, for example, when a band is defined as 91 to 100 points and examinees who receive scores of 91, 95, and 99 are treated the same.

The correct answer is: banding

217
Q

When using the split-half method to estimate the reliability of a 100-item speed test:
Select one:

A.
the split of the test into halves should be done in a random way.

B.
the test should be split so that the first 50 items are grouped together and the second 50 items are grouped together.

C.
the resulting reliability coefficient will overestimate the reliability of the test if the split is made on the basis of odd- versus even-numbered items.

D.
the resulting reliability coefficient will underestimate the reliability of the test if the split is made on the basis of odd- versus even-numbered items.

A

Internal consistency reliability is generally not used to assess the reliability of speed tests because it produces a spuriously high reliability coefficient.

a. Incorrect This would not overcome the bias introduced by using this method for assessing reliability.
b. Incorrect This procedure would not be appropriate since all of the items that were not completed would be in the same half of the test.
c. CORRECT For the exam, be sure to remember that split-half and other forms of internal consistency reliability overestimate the reliability of a speed test.
d. Incorrect This is the opposite of what is true.

The correct answer is: the resulting reliability coefficient will overestimate the reliability of the test if the split is made on the basis of odd- versus even-numbered items.

218
Q

When a test has high sensitivity, this means that there is a:
Select one:

A.
low chance of false negatives and high chance of false positives.

B.
low chance of false negatives and low chance of false positives.

C.
high chance of false negatives and high chance of false positives.

D.
high chance of false negatives and low chance of false positives.

A

Drawing a scattergram (like the one in the Test Construction chapter) would have helped you identify the correct answer to this question.

a. CORRECT Sensitivity refers to the proportion of people with the condition who are correctly identified by the test and is calculated by dividing the true positives by the true positives plus false negatives. When the sensitivity is high, this means that most of the people with the disorder will be identified as having the disorder by the test (i.e., there will be few false negatives) but that there will be some people without the disorder who will also be identified as having the disorder (i.e., there will be some false positives).
b. Incorrect See explanation above.
c. Incorrect See explanation above.
d. Incorrect Specificity refers to the proportion of people who do not have the condition who are correctly identified by the test. When a test has high specificity, there is a high chance of false negatives and low chance of false positives.

The correct answer is: low chance of false negatives and high chance of false positives.

219
Q

Item response theory is considered a useful technique for constructing all of the following types of tests except:
Select one:

A.
criterion-keyed tests.

B.
tailored (computerized) tests.

C.
diagnostic or employment screening tests.

D.
teacher-made (classroom) achievement tests.

A

Item response theory is theoretically and mathematically very complex, which limits its use.

a. Incorrect See explanation for response d.
b. Incorrect See explanation for response c.
c. Incorrect See explanation for response d.
d. CORRECT Use of item response theory for test construction requires constructing an “item response curve” for each item. To do so, a large sample of examinees is needed, which would probably not be available for classroom tests. The other types of tests listed have been identified as good candidates for item response theory.

The correct answer is: teacher-made (classroom) achievement tests.

220
Q

You conduct an item analysis of the items you plan to include in a new test of computer knowledge. You obtain a p value of .85 for item #47. This indicates that this item:
Select one:

A.
is difficult.

B.
is easy.

C.
has poor discrimination.

D.
has good discrimination.

A

In the context of test construction, “p” is an index of item difficulty and is calculated by dividing the total number of examinees who answered the item correctly by the total number of examinees.

a. Incorrect A low p value indicates a difficult item.
b. CORRECT The closer the p value is to 1.0, the easier the item since this means that a large proportion of examinees answered the item correctly.
c. Incorrect Discrimination is determined by calculating D (subtracting the proportion of people in the low-scoring group from the proportion in the high-scoring group).
d. Incorrect See explanation above.

The correct answer is: is easy.

221
Q

A screening test designed to identify individuals with traumatic brain injury identifies a number of individuals as having brain injury when, in fact, they do not. These individuals are:
Select one:

A.
false positives.

B.
true positives.

C.
false negatives.

D.
true negatives.

A

In the context of decision-making theory, individuals may be classified as false positives, true positives, false negatives, or true negatives based on their performance on the predictor and the criterion. (In the situation described in this question, the screening test is the predictor and the actual diagnosis/condition is the criterion.)

a. CORRECT False positives are individuals identified by the screening test as having brain injury when, in fact, they do not.
b. Incorrect True positives are individuals identified by the screening test as having brain injury when, in fact, they do.
c. Incorrect False negatives are individuals identified as not having brain injury when, in fact, they do.
d. Incorrect True negatives are individuals identified as not having brain injury when, in fact, they do not.

The correct answer is: false positives.

222
Q

Which of the following would NOT be useful for increasing a test’s reliability coefficient?
Select one:

A.
increasing the sample variance

B.
ensuring that the average p value is close to .50

C.
increasing the heterogeneity of the content domain

D.
increasing the number of items from 50 to 100

A

A test’s reliability coefficient, like any other correlation coefficient, is affected by a number of factors.

a. Incorrect Increasing sample variability (heterogeneity) maximizes the magnitude of the reliability coefficient.
b. Incorrect To maximize the range of scores on a test, the items should vary with regard to difficultly level, with the average difficulty level being about .50.
c. CORRECT This would have the effect of decreasing the reliability coefficient, especially the coefficient of internal consistency.
d. Incorrect Longer tests provide more reliable information.

The correct answer is: increasing the heterogeneity of the content domain

223
Q

When factors in a factor matrix are “oblique,” this means that:
Select one:

A.
the correlation coefficient for any two factors is equal to zero.

B.
the correlation coefficient for any two factors is greater than zero.

C.
the identified factors explain a statistically significant amount of variability in test scores.

D.
the identified factors do not explain a statistically significant amount of variability in test scores.

A

For the exam, you want to know the difference between orthogonal and oblique factors.

a. Incorrect In the context of factor analysis, orthogonal means “uncorrelated.”
b. CORRECT In the context of factor analysis, oblique means “correlated.”
c. Incorrect See explanation above.
d. Incorrect See explanation above.

The correct answer is: the correlation coefficient for any two factors is greater than zero.

224
Q

In the context of factor analysis, “specificity” refers to:
Select one:

A.
the proportion of variability in a test that has not been explained by the factor analysis.

B.
the proportion of variability in a test that has been explained by a single factor.

C.
the proportion of variability in a test that has been explained by all of the identified factors.

D.
the proportion of variability in a test that is attributable to measurement error.

A

As discussed in the Test Construction chapter of the written study materials, from the perspective of factor analysis, variability in test scores is due to a combination of communality, specificity, and error.

a. CORRECT In factor analysis, a test’s specificity is the variability that is due to factors that are specific to the test and not measured by any other test included in the analysis – i.e., variability that is not accounted for by the identified factors.
b. Incorrect See explanation for response a.
c. Incorrect See explanation for response a.
d. Incorrect See explanation for response a.

The correct answer is: the proportion of variability in a test that has not been explained by the factor analysis.

225
Q

When the results of test developer’s validity study confirm that her new measure of an attribute correlates highly with an established measure of the same attribute and does not correlate highly with established measures of unrelated attributes, the test developer has evidence of her test’s:
Select one:

A.
content validity.

B.
construct validity.

C.
internal validity.

D.
external validity.

A

For the exam, you want to be familiar with the major types of test validity - i.e., content, construct, criterion-related.

a. Incorrect Content validity is established primarily by having subject matter experts review test items to make sure they are a representative sample of the content domain(s) assessed by the test.
b. CORRECT A test has construct validity when it measures the construct (trait or attribute) it was designed to measure. It is established by gathering evidence by, for example, determining if the test has adequate convergent and divergent validity: A test has convergent validity when its scores have high correlations with scores on measures of the same and related constructs, and it has divergent validity when its scores have low correlations with scores on measures of unrelated constructs.
c. Incorrect Internal and external validity refer to the validity of research studies, not tests. A research study has adequate internal validity when its results allow a researcher to conclude that there is cause-effect relationship between the independent and dependent variables. A research study has adequate external validity when it allows the researcher to generalize conclusions about the cause-effect relationship to other people and conditions.
d. Incorrect See explanation for response c.

The correct answer is: construct validity.